Atheistforums.com

Science Section => Science General Discussion => Physics & Cosmology => Topic started by: Mousetrap on August 13, 2018, 08:21:38 AM

Title: Time dilation, length contraction, Relativity and the Bible!
Post by: Mousetrap on August 13, 2018, 08:21:38 AM
Quote from: Quote from: Hakurei Reimu on August 09, 2018, 10:04:36 AM

You, MT, are absolutely convinced in your ability to argue and gather facts, when everyone else sees that you're not doing abysmally. There is an explanation for this difference and that is the Dunning-Kruger effect â€" your ability to self-assess and your ability to perform are the same skill-set. If you are terrible at what you do, you will be terrible at assessing how good a job you're doing. The Dunning-Kruger effect is basically why certification is a thing.

You have been assessed by your more competent peers and have been found wanting. I in particular have given you real tests to assess your ability to do physics, and you have been unable to do them. I still await your calculations showing the viability of the "Biblical" nebular hypothesis.

John Lennox has the same affliction. He is not trained in the very fields that would allow him to competently assess Hawking's ideas. Now, he might be right, but that's a matter for other experts in the field to decide, not him. So far, none of his criticisms have stuck.
Quote from: MousetrapNow why dont you and I discuss...say...Time dilation and length contraction?
I do not agree that time can be changed and therefor have a huge problem with The theory of relativity, and special relativity.

I am not even going to pretend that I am a physicist, but I always wondered about the theory of Relativity and Special relativity.
But on the other hand, Hakurei Reimu is a well known and highly accredited scientist with numerous peer reviewed publications behind his name.
He even have the capability to prove John Lennox wrong, and that is one helova achievement.
And HR really hated the claims the MT made on the origins of the Universe.

I then thought it to be a valiant thing to do, and that is to ask, if one travels at the speed of light, does Time dilute?
Now why on Earth would I ask such a thing?
well, one can go on the WWW and listen to what scientists, universities, et all say on the topic, and they all are in agreement.
Time dilutes!

Bring into mind the Twin paradox, and one understands the topic.

Now, bring the Biblical view into time dilution, and it opposes any such a notion.
Quote from: KJVGen 1:14  And God said, Let there be lights in the firmament of the heaven to divide the day from the night; and let them be for signs, and for seasons, and for days, and years:

Daniel 7:25  And he shall speak great words against the most High, and shall wear out the saints of the most High, and think to change times and laws: and they shall be given into his hand until a time and times and the dividing of time.

Daniel can also be prophetically evaluated, but in reference to changing Times and Laws, can only be interpreated litterally.

I might be wrong, but I claim that Time can not change. No matter at what speed one travel.
Lets see, will HR propose time can be diluted?
If so, we have a nice test to see if the Biblical and scientific view on 'Time' differs from each other.
Title: Re: Time dilation, length contraction, Relativity and the Bible!
Post by: SGOS on August 13, 2018, 10:35:40 AM
It's time dilation, not time dilution.

https://en.wikipedia.org/wiki/Time_dilation
Title: Re: Time dilation, length contraction, Relativity and the Bible!
Post by: Baruch on August 13, 2018, 01:03:27 PM
Sorry, but this is off the deep end even for you ;-)  I hope you have your life preserver handy ;-))

There is very little definition of physics, even of Aristotle, in the Bible.  Jesus is more interested in living things in general, and people in particular ... he shows little interest in materialism.
Title: Re: Time dilation, length contraction, Relativity and the Bible!
Post by: Unbeliever on August 13, 2018, 01:22:17 PM
Quote from: Mousetrap on August 13, 2018, 08:21:38 AM
I might be wrong, but I claim that Time can not change. No matter at what speed one travel.

Why should we give a damn what you believe? You can claim anything you like, to your heart's content. Who gives a shit?
Title: Re: Time dilation, length contraction, Relativity and the Bible!
Post by: Hakurei Reimu on August 13, 2018, 06:38:29 PM
Quote from: Mousetrap on August 13, 2018, 08:21:38 AM
I am not even going to pretend that I am a physicist, but I always wondered about the theory of Relativity and Special relativity.
But on the other hand, Hakurei Reimu is a well known and highly accredited scientist with numerous peer reviewed publications behind his name.
He even have the capability to prove John Lennox wrong, and that is one helova achievement.
And HR really hated the claims the MT made on the origins of the Universe.
Stop this empty posturing. I don't claim any credentials other than what I have demonstrated. I don't claim to be a physicist. I just don't think that John Lennox is in any position to say Hawking was wrong when Hawking's academic peers and fellows in the field â€"all of whom should know Hawking's ideas and the state of the science better than anyoneâ€" aren't.

Unless Lennox's chosen field of expertise intersects in some way with Hawking's theories, his word is no better than anyone else's. Not even mine, and not even yours.

BTW, you still haven't answered my criticisms about your moronic theories of the origins of the Universe. I think it's time to call a spade a spade and a posturing blowhard a posturing blowhard.

Quote
<snip drabble about time "dilution", lol>

I might be wrong, but I claim that Time can not change. No matter at what speed one travel.
Yes, you are, in fact, wrong. We have observed time dilation happening in particle accelerators, and even in nature. Particle half-lives serve as natural timers, and we observe that more particles survive longer than they have any right to when their speeds are higher.

Experiment trumps assertions every single time.

We've observed time dilation experimentally. Deal with it.
Title: Re: Time dilation, length contraction, Relativity and the Bible!
Post by: Unbeliever on August 13, 2018, 06:42:08 PM
Einstein's "Time Dilation" Prediction Verified (https://www.scientificamerican.com/article/einsteins-time-dilation-prediction-verified/)

QuoteExperiments at a particle accelerator have confirmed the "time dilation" effect predicted by Albert Einstein's special theory of relativity
Title: Re: Time dilation, length contraction, Relativity and the Bible!
Post by: Baruch on August 13, 2018, 08:54:26 PM
Quote from: Unbeliever on August 13, 2018, 06:42:08 PM
Einstein's "Time Dilation" Prediction Verified (https://www.scientificamerican.com/article/einsteins-time-dilation-prediction-verified/)

Yes, particle radioactive half lives (a statistical thing) become longer at higher relative speeds.  With such small particles it is at least possible to accelerate them to nearly the speed of light.  Per quantum mechanics however (and Schrodinger's Cat) you can't predict the half life of individual unstable particles.
Title: Re: Time dilation, length contraction, Relativity and the Bible!
Post by: Mousetrap on August 14, 2018, 02:44:32 AM
Quote from: Hakurei Reimu on August 13, 2018, 06:38:29 PM
Stop this empty posturing. ...
Yes, you are, in fact, wrong. We have observed time dilation happening in particle accelerators, and even in nature. Particle half-lives serve as natural timers, and we observe that more particles survive longer than they have any right to when their speeds are higher.

We've observed time dilation experimentally. Deal with it.
Wow, I would have loved to have been there with you to also observe what you saw!
Title: Re: Time dilation, length contraction, Relativity and the Bible!
Post by: Mousetrap on August 14, 2018, 02:46:38 AM
Quote from: Unbeliever on August 13, 2018, 01:22:17 PM
Why should we give a damn what you believe? You can claim anything you like, to your heart's content. Who gives a shit?
Wont you love to see how I will be decimated when we start to discuss Time Dilation.
Sorry for the previous spelling mistake where I typed Dilution.
But I like that some atheists volunteer to fix my spelling.
Much appreciated to those who do.
Title: Re: Time dilation, length contraction, Relativity and the Bible!
Post by: Mousetrap on August 14, 2018, 03:26:19 AM
Quote from: Unbeliever on August 13, 2018, 06:42:08 PM
Einstein's "Time Dilation" Prediction Verified (https://www.scientificamerican.com/article/einsteins-time-dilation-prediction-verified/)
and they continue with an incorrect experimentation principle since 2002, to prove something that does not exist (https://pdfs.semanticscholar.org/505b/3ba592f5d9d19153c27bf8032fc6c60527c9.pdf)

Ok, so what we have now is L-ion sent in repetition and measured with Lasar to see where the electrons are at specific points. Now we call this a "Clock".
We push these ions at great speeds, one third C, with accelerators, using magnetism, and hope that it has zero effect on the frequency of the electrons, and if we find a variance, we call this Time dilation.

If we send a wheel at 1000 Km per hour, we will ignore the centrifugal forces on the rubber all around the wheel, measure the touching point on the ground with a pulsating tachograph, see that the clock is standing still, and call it evidence of Time dilation.

Lets get back to what SR and GR says, and show me what Lorentz is all about.
All these scientists are doing is to produce results that are non existent to get more grants to build bigger particle accelerators.

So, what does the Lorentz transformation do?
It only shows you what an observer will see when he looks at a moving object.

It does not verify that any clocks are running slower.
??? (https://www.kritik-relativitaetstheorie.de/2013/03/the-einstein-hoax-the-disastrous-intellectual-war-on-common-sense/)
Title: Re: Time dilation, length contraction, Relativity and the Bible!
Post by: Mousetrap on August 14, 2018, 05:29:33 AM
Lets start here.

(Gedanken experiment)

We move the moon a bit closer to the Earth so that it is exactly 300 000 Km from us.
WE place a huge clock on the Earth, and a month or so ago, we placed an exact replicaon the moon and we installed an exact replica on a rocket.
We man this rocket, and we had one of the clock installers on the Moon, staying there as an observer.
With special technology, at exactly 12H00, the rocket accelerated immediately to the speed of Light -1 Km per hour. (to satisfy those that say you can not travel faster than light.)
All these clocks were synchronized when they were built on Earth.

The questions to answer is:
1. When the Rocket left the Earth at 12H00, what did the clock on the Moon say time was seen from Earth?
2. When the Rocket reached 150 000 Km from the earth, what did the clock on the Moon say time was seen from Earth?
3. and what did the clock on the rocket say time was seen from Earth?
4. and what did the clock on the Earth say it was as seen from Earth.?

Title: Re: Time dilation, length contraction, Relativity and the Bible!
Post by: Baruch on August 14, 2018, 05:32:39 AM
There is no universal clock etc ... because the "Sensorium of G-d" from Newton, is fictional.
Title: Re: Time dilation, length contraction, Relativity and the Bible!
Post by: Mousetrap on August 14, 2018, 06:55:22 AM
Quote from: Baruch on August 14, 2018, 05:32:39 AM
There is no universal clock etc ... because the "Sensorium of G-d" from Newton, is fictional.
True.
However, if we take time as we measure it, eg Radio active decay, Sun Earth Greenwich median, we will still have a measurable medium to determine how far light travels in a second.

In this instance we measure time in seconds with very special clocks that corresponds with the time light travels in a meter.
oh, lets round it up to 300 000 km per sec.
Title: Re: Time dilation, length contraction, Relativity and the Bible!
Post by: Baruch on August 14, 2018, 07:08:33 AM
Quote from: Mousetrap on August 14, 2018, 06:55:22 AM
True.
However, if we take time as we measure it, eg Radio active decay, Sun Earth Greenwich median, we will still have a measurable medium to determine how far light travels in a second.

In this instance we measure time in seconds with very special clocks that corresponds with the time light travels in a meter.
oh, lets round it up to 300 000 km per sec.

As we define time and distance.  But if you move along the light beam, per Einstein's imagination ... then you see things differently than Slomos like us.  There is a paradox about "c" that I won't go into here.  But suffice it to day, that you can define the speed of light to any finite positive number ... and often in quantum mechanics it is defined to be = 1.  The current metrology is so convinced, that now length is defined in terms of time (atomic vibration as standard) it isn't a palladium meter bar in Paris.
Title: Re: Time dilation, length contraction, Relativity and the Bible!
Post by: Mousetrap on August 14, 2018, 07:36:53 AM
True Baruch,
Now, I see it as, when the special rocket departs from the Earth at 1C, at exactly 12H00,
When the rocket left the Earth.
1.the clock on the Moon showed 11H59s59 (because it took the light to travel one second to arrive on Earth, from the clock on the moon)
The Clock on the Earth showed 12H00
and the clock on the Rocket was also at 12H00.
When the rocket arrived at the moon
2. The clock on the Earth showed 12H00s01
The clock on the Moon showed 12H00s00 Because the light took 1 sec to travel to the Earth and 1 second passed since the rocket departed and arrived on the Moon)
and the rocket's clock also showed 12H00s00. (because the rocket traveled with its light that showed the clock was at 12H00 when it departed.)

When the rocket reached halfway to the Moon,
the Earth's clock said 12H00s0.5
the Moon's clock said 11H59s0.5
and the Rocket's clock said 12H00s00

The above is an example of how we will note what the different time increments are when we look at different time frames.
On earth it will seem as if the clock on the rocket stood still, until it stopped on the Moon.
The clock on the moon will be one second later than ours, and upon the arrival of the rocket on the moon, the clock of the Rocket and Moon will appear to be one second later than Earth's.

If we do the same with this experiment, but this time from the point of view of the observer on the Moon, he will note the times exactly as how we saw the time frame on the rocket and moon. Except that the clock on the rocket will appear to take off from earth at 1 second before noon, but the clock on the rocket will appear to run twice as fast, and will gain 2 seconds until it arrives at 12H00s1 on the moon traveling only one second.

The pilot in the rocket will obviously see something totally different. If he leaves the Earth, he will see the Earth clock showing 12H00s00 untill he gets to the Moon. the Moon clock will show 11H59s01 and will seem to run twice as fast, and upon arrival will show 12H00s01, and would have gained 2 Seconds in one second travelling time.

3 different time frames, 3 different observations, but still, time did not dilate.
One second is still one second.
Title: Re: Time dilation, length contraction, Relativity and the Bible!
Post by: Baruch on August 14, 2018, 01:28:39 PM
Personally, I accept actual experimental results.  The rocket-twin idea has never been tested, it simply falls as a logical consequence of what happens with sub-atomic particles.  Again, in that actual experiment, statistics intervenes.  So if you had a statistical sample of twin brothers on Earth, and their counterparts in the rocket ship, then on average, their internal clocks would be out of synch (Earth vs Rocket).  And this only applies to constant motion, not acceleration/deceleration (which is required in a practical comparison).  The actual people in the rocket/Earth would not notice anything out of the ordinary.  It is only the comparison that is out of the ordinary.
Title: Re: Time dilation, length contraction, Relativity and the Bible!
Post by: aitm on August 14, 2018, 06:38:47 PM
Right.....kid tries real hard to present ole god as a genius....the same god what claims a proper man can be determined by the way he drinks water from a river...or that a womans period is reason to put her in a tent outside of town for a week and she is filthy.....yep.....old god sure is a brilliant mind.
Title: Re: Time dilation, length contraction, Relativity and the Bible!
Post by: Baruch on August 14, 2018, 06:57:10 PM
Quote from: aitm on August 14, 2018, 06:38:47 PM
Right.....kid tries real hard to present ole god as a genius....the same god what claims a proper man can be determined by the way he drinks water from a river...or that a womans period is reason to put her in a tent outside of town for a week and she is filthy.....yep.....old god sure is a brilliant mind.

If you had to deal with an ancient woman on her period, you would temporarily exile her too.
Title: Re: Time dilation, length contraction, Relativity and the Bible!
Post by: Mousetrap on August 15, 2018, 02:27:20 AM
Quote from: aitm on August 14, 2018, 06:38:47 PM
Right.....kid tries real hard to present ole god as a genius....the same god what claims a proper man can be determined by the way he drinks water from a river...or that a womans period is reason to put her in a tent outside of town for a week and she is filthy.....yep.....old god sure is a brilliant mind.
Duhhhh!
Time dilation!!!
Are you OK pal?
Title: Re: Time dilation, length contraction, Relativity and the Bible!
Post by: Mousetrap on August 15, 2018, 07:31:58 AM
I wonder where is Hakurei Reimu.
Perhaps he can tell us what the Lorentz transformation explains.

You see, if I have heaps of apples of 20 per heap, and I have 5 heaps, I do not have to count every apple to get to 100 apples.
All I have to do is to multiply 20 by 5.
Therefore, the mathematics of multiplication helps me to work out an answer by multiplying how many groups of certain units I have.

If I have 5 apples, and eat 1, how many do I have?
In this sense subtraction allows me to see what results I have after removing a certain amount from a heap of units.

So, Hakurei Reimu can perhaps teach me, from his scientific physicist education, what does the Lorentz transformation do?

When will I use the Lorentz transformation.

Title: Re: Time dilation, length contraction, Relativity and the Bible!
Post by: Hydra009 on August 15, 2018, 12:26:42 PM
Quote from: Mousetrap on August 15, 2018, 07:31:58 AM
I wonder where is Hakurei Reimu.
Probably putting his time to better use.  Shopping for groceries, using the john, watching paint dry.  That sort of thing.
Title: Re: Time dilation, length contraction, Relativity and the Bible!
Post by: Baruch on August 15, 2018, 01:09:23 PM
Lorentz transformation ... from the point of view of Earth, the Rocket clock is going slow.  From the point of view of the Rocket, the Earth clock is going slow.  Doesn't make much sense does it?  If you accept some Platonic objectivity (absolute time and space).  The original problem for Einstein, was synchronization ... because in relative motion, this is impossible, the Lorentz transform follows, if you are using light beams to keep track of things.

Title: Re: Time dilation, length contraction, Relativity and the Bible!
Post by: Unbeliever on August 15, 2018, 01:49:30 PM
Quote from: Mousetrap on August 15, 2018, 07:31:58 AM
If I have 5 apples, and eat 1, how many do I have?

You still have 5 apples - 4 outside and one inside.
Title: Re: Time dilation, length contraction, Relativity and the Bible!
Post by: Jason78 on August 15, 2018, 02:11:34 PM
Quote from: Mousetrap on August 15, 2018, 07:31:58 AM
I wonder where is Hakurei Reimu.
Perhaps he can tell us what the Lorentz transformation explains.

You see, if I have heaps of apples of 20 per heap, and I have 5 heaps, I do not have to count every apple to get to 100 apples.
All I have to do is to multiply 20 by 5.
Therefore, the mathematics of multiplication helps me to work out an answer by multiplying how many groups of certain units I have.

If I have 5 apples, and eat 1, how many do I have?
In this sense subtraction allows me to see what results I have after removing a certain amount from a heap of units.

So, Hakurei Reimu can perhaps teach me, from his scientific physicist education, what does the Lorentz transformation do?

When will I use the Lorentz transformation.



What level of maths education do you have?
Title: Re: Time dilation, length contraction, Relativity and the Bible!
Post by: aitm on August 15, 2018, 08:39:58 PM
Quote from: Mousetrap on August 15, 2018, 02:27:20 AM
Duhhhh!
Time dilation!!!
Are you OK pal?

Yeah,,,stick to your thoughts and ignore the obvious. Typical xian, ignore the stupidity of the babble and invent its "genius". Right....yer a real brainiac
Title: Re: Time dilation, length contraction, Relativity and the Bible!
Post by: Mousetrap on August 16, 2018, 02:09:27 AM
Quote from: Jason78 on August 15, 2018, 02:11:34 PM
What level of maths education do you have?
Why, Dont you understand how many apples I have?
Title: Re: Time dilation, length contraction, Relativity and the Bible!
Post by: Mousetrap on August 16, 2018, 02:23:42 AM
I find it frustrating that an atheist challenges me to a mathematical think tank where he totally ignore anything observed science discovered on the origins of the Earth, and how hard he must fight to prove that the Earth was not wet when it formed.
He demands that I explain by means of Mathematics on the Nebular theory, and how I should steer clear from any reference that the original Earth had any water content, whilst the latest discoveries proves the contrary.

I thought for a while, and decided that going through the Special relativity, and General relativity with any person who are a master on Mathematics, might just be the answer on the accusations made by an Atheist against a Bible Believing Christian.

In particular, the Lorentz transformation.
Some basic mathematics that works out Time dilation.

I know there are many lectures on You tube, as well as Google showing exactly how to do the calculations, therefore, I can accept that we do not have to spent time in learning the maths.

However, I want to see if my opposing Atheist Mathematician could tell me what the Lorentz transformation does, and when we can use this mathematical formula.

Yet, it seems as if the learned Mathe-magician dont know.

Anyone up for grabs?



Title: Re: Time dilation, length contraction, Relativity and the Bible!
Post by: Mousetrap on August 16, 2018, 10:18:30 AM
Great stuff, so let me continue.

If I travel to this moon we moved to one light second from the Earth, at the speed of light.
It will take one second.

Now, I have yet to find a physicist who do not think that if this happen, we will travel and time will stand still.

BS! Time will not stand still at all!
The only thing that will happen, is that as we move away from the Earth, an observer on the Earth will notice that our clock on the rocket stands still because we are moving away with the light..
The same goes for the pilot, he will see the Earth clock stand still!

Now, lets consider.
If the rocket travels to the Moon at the speed of light, how long will the journey be?
One second.
How the heck does anyone think that time dilated?

Lets carry one with this experiment.
Physicists say, if we travel faster than light, we will go back in time!

Ok, so lets travel at 2X the speed of C!
How long does the journey take?
HALF A SECOND!
WHERE DID THEY GET THE IDEA THAT WE WILL TRAVEL BACK INTO TIME?

Ok, so lets travel at 4X the speed of Light.
How long does the journey take?

A QUARTER OF A SECOND!

at 8X the speed of light, it will take an eighth of a second, and so on...

Guys, do you see the constant?

Time!!!

Time can not be changed, Einstein lied, and the Bible is true!

Title: Re: Time dilation, length contraction, Relativity and the Bible!
Post by: Hakurei Reimu on August 16, 2018, 10:28:01 AM
Quote from: Mousetrap on August 14, 2018, 05:29:33 AM
Lets start here.

(Gedanken experiment)

<snip>

(to satisfy those that say you can not travel faster than light.)
The point of gedanken experiments is that we can ignore some unphysicalities to get at the meat of a physical principle. You're even allowed to fudge the exact numbers of real objects for the purposes of the experiment.

Quote
All these clocks were synchronized when they were built on Earth.

The questions to answer is:
1. When the Rocket left the Earth at 12H00, what did the clock on the Moon say time was seen from Earth?
The guys on the moon know that they are 1 light second away from Earth, because they can send a message, "Hey, what time is it?" at 11:59:59 and get a reply at 12:00:01 of "It's 12:00:00." If that is any other time, then the guys on the moon know that their clocks are out of synch. The guys on the moon know (because they know that they are 1 light second away from Earth) that their signal would arrive on Earth at 12:00:00 by their own clock (because they can do math), so they must get the reply at 12:00:01 by their own clock (again, because they can do math), so if they get a reply of any other time, they know their clock is out of synch. Given some math, this works to any arbitrary distance.

I would assume that the guys on the moon would spend some time synching their clock with Earth's before the experiment begins. Even if they just stupidly set their clock to the time they observe through their telescope, they just note that their time is 1 second behind Earth time. Hell, they didn't even need to do that, as if they moved the clock slowly enough to the moon, there would be no appreciable time dialation (launching the clock Apollo style, it would count as "slowly enough").

The same situation would be readily observable from the Earth. They would see the moon clock reading a time one second behind their own, but would know that the light they're seeing took one second to get to Earth, and therefore the clock on the moon is reading one second ahead of what they see. They thus deduce that the clock on the moon is synchronized with their own.

Quote
2. When the Rocket reached 150 000 Km from the earth, what did the clock on the Moon say time was seen from Earth?
When the rocket left, Earth's clock read 12:00:00, and the moon's clock is seen from Earth to read 11:59:59, so halfway there, the Earth sees 11:59:59.5 on the Moon clock, but know that they see the clock as one second behind theirs, so they know the moon clock actually reads 12:00:00.5, like Earth's.

Quote
3. and what did the clock on the rocket say time was seen from Earth?
Approximately 12:00:00, because the clocks started out in synch, and not much proper time has progressed on the near-c rocket.

Quote
4. and what did the clock on the Earth say it was as seen from Earth.?
12:00:00.5, same as they know the Moon clock is actually reading at that time.

The Moon and the Earth are comoving, albeit separated by a non-trivial distance in space. Any clocks that are synchronous and comoving will remain synchronous.

Quote from: Mousetrap on August 14, 2018, 07:36:53 AM
True Baruch,
Now, I see it as, when the special rocket departs from the Earth at 1C, at exactly 12H00,
When the rocket left the Earth.
1.the clock on the Moon showed 11H59s59 (because it took the light to travel one second to arrive on Earth, from the clock on the moon)
The Clock on the Earth showed 12H00
and the clock on the Rocket was also at 12H00.
You assume the clocks are out of synch. There is no reason to assume that, and even if they were, there are plenty of ways to find out and correct this synch issue even at distance.

Quote
When the rocket arrived at the moon
2. The clock on the Earth showed 12H00s01
The clock on the Moon showed 12H00s00 Because the light took 1 sec to travel to the Earth and 1 second passed since the rocket departed and arrived on the Moon)
and the rocket's clock also showed 12H00s00. (because the rocket traveled with its light that showed the clock was at 12H00 when it departed.)
Not necessarily. Again, you can keep those clocks in sync, even across disance. Even if they aren't, the guys on the moon know they are 1 second behind the Earth clock.

Just wanted to note that.

Quote
When the rocket reached halfway to the Moon,
the Earth's clock said 12H00s0.5
the Moon's clock said 11H59s0.5
and the Rocket's clock said 12H00s00
Nothing unusual here.

Quote
The above is an example of how we will note what the different time increments are when we look at different time frames.
On earth it will seem as if the clock on the rocket stood still, until it stopped on the Moon.
The clock on the moon will be one second later than ours, and upon the arrival of the rocket on the moon, the clock of the Rocket and Moon will appear to be one second later than Earth's.
Again, the Earth knows that the image of the clock on the moon is one second old, and thus they have to assume the clock on the Moon is one second ahead of what they see it. The experiment, as you have set it up (with its out-of-synch clocks), has it that the rocket arrives at the moon when both clocks read 12:00:00. When the Earth looks through its telescope at 12:00:02, it will correctly see that the rocket clock and the moon clock both read 12:00:00, and they know that this image is one second behind.

Nothing unusual here. They know that the Moon clock is one second behind their own, so the Moon clock image they see should read 12:00:01 if they were properly synchronized, and that the light from that clock took a further 1 second to get back to them, so they are properly seeing it at 12:00:02 Earth time.

Quote
If we do the same with this experiment, but this time from the point of view of the observer on the Moon, he will note the times exactly as how we saw the time frame on the rocket and moon. Except that the clock on the rocket will appear to take off from earth at 1 second before noon,
Wrong. The moon sees that the image of the Earth clock clearly reads 12:00:00 at the moment that the rocket passes that Earth clock (and the rocket's clock reads 12:00:00 at that moment, too). Of course, that light took one second to reach the moon, so they will get this image at 12:00:00 by their clock, but then, they know that their clock is out of synch with the Earth clock.

I say this again, because it's important: the moon knows (or should know, as it's easy to find out) it's clock is out of synch with the Earth's clock, and by how much (and thus can easily bring the two into synch with proper adjustments).

Quote
but the clock on the rocket will appear to run twice as fast, and will gain 2 seconds until it arrives at 12H00s1 on the moon traveling only one second.
Wrong. The clock on the rocket remains stubbornly at 12:00:00 through the entire trip, including the very moment it passes the moon at 12:00:00 Moon time (12:00:01 Earth time). The Moon correctly deduces that practically no time has passed on the rocket since it left Earth.

The gold standard for measurement of proper time (time as experienced by an object) is a nearby, comoving clock. The clock on the rocket is a nearby, comoving clock (it's onboard, so it's going to be nearby and comoving). Ergo, it correctly measures the time abord the ship.

Nothing unusual has happened.

Quote
The pilot in the rocket will obviously see something totally different. If he leaves the Earth, he will see the Earth clock showing 12H00s00 untill he gets to the Moon. the Moon clock will show 11H59s01 and will seem to run twice as fast, and upon arrival will show 12H00s01, and would have gained 2 Seconds in one second travelling time.
Wrong. At the moment the rocket arrives on the moon, the moon's clock is showing 12:00:00, and the moon can inform the rocket crew that their time is one second behind the Earth's clock. At the moment they arrive, the rocket crew sees the Earth clock as reading 12:00:00, but they know that the Earth is one light second away, so the image is one second old. The Earth clock should read 12:00:01 at the time of their arrival, which they confirm a second later.

Furthermore, they realize that they have not experienced any appreciable time between when they left Earth and when they arrived on the Moon, as evidenced by their clock not registering any time between Earth departure (where the ship clock read 12:00:00) and Moon arrival (where the ship clock still read 12:00:00).

Quote
3 different time frames, 3 different observations, but still, time did not dilate.
Wrong. The ship's clock clearly did not register the passage of a second that the other two clocks registered.

Quote from: Mousetrap on August 15, 2018, 07:31:58 AM
I wonder where is Hakurei Reimu.
Says the guy who disappears every weekend.

Don't criticize my schedule, and I won't criticize yours.

Quote
Perhaps he can tell us what the Lorentz transformation explains.
The explanation is simple: you bolloxed up the calculation of how much time actually passed between frames. The Earth and Moon clocks were comoving, and as such, they registered the same length of time. The only clock that was not registering the same length of time (and indeed, a shorter length of time) was the rocket clock, and it was the only clock that was moving.

Quote
You see, if I have heaps of apples of 20 per heap, and I have 5 heaps, I do not have to count every apple to get to 100 apples.
All I have to do is to multiply 20 by 5.
Therefore, the mathematics of multiplication helps me to work out an answer by multiplying how many groups of certain units I have.

If I have 5 apples, and eat 1, how many do I have?
In this sense subtraction allows me to see what results I have after removing a certain amount from a heap of units.

So, Hakurei Reimu can perhaps teach me, from his scientific physicist education, what does the Lorentz transformation do?
The Lorentz transformation doesn't apply here, unless you are interested in when and where each apple is eaten according to an observer moving relative to you.
Title: Re: Time dilation, length contraction, Relativity and the Bible!
Post by: Mousetrap on August 16, 2018, 10:31:56 AM
OK, so why do they say time will turn back if we travel faster than the speed of light?

It works this way.

Lets say we travel to A Centauri. 4.67 Light Years, and we travel at the speed of light.
There is this huge clock on Earth and we can see the time, date and years.
As we travel away from Earth, we will see the clock stands still, because we are travelling with the light as we departed from Earth.

However, If we travel at 2X C, the rocket will gain on the light that was travelling at C from the Earth, and it will seems as if this clock is turning in reverse,...
Using the formulas of Lorentz, it seems as if Time dilated!

No, it did not, all that happened is that the light we observed in our reference frame, was light we gained on as we traveled and gained on.
Title: Re: Time dilation, length contraction, Relativity and the Bible!
Post by: Mousetrap on August 16, 2018, 10:36:04 AM
Quote from: Hakurei Reimu on August 16, 2018, 10:28:01 AM

I would assume that the guys on the moon would spend some time synching their clock with Earth's before the experiment begins. Even if they just stupidly set their clock to the time they observe through their telescope, they just note that their time is 1 second behind Earth time. Hell, they didn't even need to do that, as if they moved the clock slowly enough to the moon, there would be no appreciable time dialation (launching the clock Apollo style, it would count as "slowly enough").

How about building the clocks on the Earth, load it on a rocket, blast all 3 into space, stop halfway, send the clocks with the same speed to the Moon and Earth with smaller rockets that travel at snailpace to both positions.
Why all the fuss to try and show that speed will have an effect on a clock?
Title: Re: Time dilation, length contraction, Relativity and the Bible!
Post by: Mousetrap on August 16, 2018, 10:53:10 AM

Quote from: Mouse trap
If we do the same with this experiment, but this time from the point of view of the observer on the Moon, he will note the times exactly as how we saw the time frame on the rocket and moon. Except that the clock on the rocket will appear to take off from earth at 1 second before noon,
Quote from: HRWrong. The moon sees that the image of the Earth clock clearly reads 12:00:00 at the moment that the rocket passes that Earth clock (and the rocket's clock reads 12:00:00 at that moment, too). Of course, that light took one second to reach the moon, so they will get this image at 12:00:00 by their clock, but then, they know that their clock is out of synch with the Earth clock.

I say this again, because it's important: the moon knows (or should know, as it's easy to find out) it's clock is out of synch with the Earth's clock, and by how much (and thus can easily bring the two into synch with proper adjustments).

I think you missed out when I wrote, lets do this experiment again, but from the time-frame of the Moon.
I want us to be clear about my claim.
If the observer on the Moon looks at the Earth at 12H00, when the rocket also blasts off from the Earth at 12H00, the observer on the Moon will see the clock on the Earth and the clock on the Rocket are both one second before Noon.

The clock on the Moon will move forward for one second, and the rocket will arrive at the same instance.
The clock on the Earth will appear to be noon, but both the Rocket and Moon clock will show one second past Noon.

Therefore, the Earth clock seems to be late with 1 second to the one on the moon, and the clock on the Rocket seemed to have moved forward 2 seconds on its journey of one second.

Are we both in agreement?
Title: Re: Time dilation, length contraction, Relativity and the Bible!
Post by: Hakurei Reimu on August 16, 2018, 10:53:27 AM
Quote from: Mousetrap on August 16, 2018, 10:18:30 AM
Great stuff, so let me continue.

If I travel to this moon we moved to one light second from the Earth, at the speed of light.
It will take one second.
It will take one second from any observer but you. For you, it will be instantaneous. One moment, your on Earth, and the next, you're on the Moon.

Quote
Now, I have yet to find a physicist who do not think that if this happen, we will travel and time will stand still.

BS! Time will not stand still at all!
Every physicist will say that YOUR time will stand still. You will not experience time as you travel at the speed of light. Everyone else, however, will not be affected.

Quote
The only thing that will happen, is that as we move away from the Earth, an observer on the Earth will notice that our clock on the rocket stands still because we are moving away with the light..
Even taking into account light delays, they will still see your clock stay still. Assuming you have time to transmit at all (which is not really an option since you experience no time at all at light speed), they will be able to correct for the time that it takes for light to get to Earth. And when they make that correction, they will see that time on your spaceship has not progressed at all.

Quote
The same goes for the pilot, he will see the Earth clock stand still!
Assuming that he even has time to contemplate anything before smashing into Alpha Centari, yes. But then, the guy on the rocket felt the acceleration on his own ship, and can see the accelerometers on Earth not registering anything, so he knows that the Earth is not accelerated. He is.

Quote
Now, lets consider.
If the rocket travels to the Moon at the speed of light, how long will the journey be?
One second.
By the Earth clock and the Moon clock, but not the rocket clock. The rocket clock is the only clock that is moving relative to anything else in the experiment, and as such is the only clock that "experiences" time dialation.

Quote
How the heck does anyone think that time dilated?
Again, we've seen it in action. Science has seen it in action. Too many muons that are generated in the earth's atmosphere from cosmic rays live to reach the surface. Short-lived particles generated in accelerators travel too far before they decay if their clocks were running at the same speed as ours.

Your inability to comprehend relativity doesn't mean it's wrong. It just means you don't comprehend relativity.

Quote
Lets carry one with this experiment.
Physicists say, if we travel faster than light, we will go back in time!
IF it were possible to go faster than light, then we can leverage it to go back in time. If you were actually to read up on what relativity says about how to do this, if it were possible, you need at least two legs to make it happen. Here's a discussion about the problem of FTL and time travel from Physicsmatt:

http://www.physicsmatt.com/blog/2016/8/25/why-ftl-implies-time-travel

Quote
Ok, so lets travel at 2X the speed of C!
How long does the journey take?
HALF A SECOND!
WHERE DID THEY GET THE IDEA THAT WE WILL TRAVEL BACK INTO TIME?

Ok, so lets travel at 4X the speed of Light.
How long does the journey take?

A QUARTER OF A SECOND!

at 8X the speed of light, it will take an eighth of a second, and so on...

Guys, do you see the constant?

Time!!!

Time can not be changed, Einstein lied, and the Bible is true!
No, you simply don't understand what you're talking about. From one frame of reference, there is no time travel, and you and Einstein agree on that. It takes ANOTHER reference frame to generate one of these paradoxes. The reason you haven't generated time travel in your thought experiment is because you haven't set up the conditions for it to occur yet.
Title: Re: Time dilation, length contraction, Relativity and the Bible!
Post by: Hakurei Reimu on August 16, 2018, 12:53:34 PM
Quote from: Mousetrap on August 16, 2018, 10:31:56 AM
OK, so why do they say time will turn back if we travel faster than the speed of light?

It works this way.

Lets say we travel to A Centauri. 4.67 Light Years, and we travel at the speed of light.
There is this huge clock on Earth and we can see the time, date and years.
As we travel away from Earth, we will see the clock stands still, because we are travelling with the light as we departed from Earth.

However, If we travel at 2X C, the rocket will gain on the light that was travelling at C from the Earth, and it will seems as if this clock is turning in reverse,...
Using the formulas of Lorentz, it seems as if Time dilated!

No, it did not, all that happened is that the light we observed in our reference frame, was light we gained on as we traveled and gained on.
You need to do something else in order for time travel to show up. Let's take a more extreme example. Suppose our FTL drive travels along a world line's line "Now" â€" that is, if we have a bunch of clocks spread all over the universe that are both synchronized with our home clock and comoving with our own clock, the FTL drive will arrive at a new clock that reads exactly the same time as the clock it left. But if this FTL engine obeys relativity, then it will be able to arrive at that clock at that time ONLY if that clock was initially comoving with the ship carrying the drive.

We now set our space ship to travel to Alpha Centauri using the FTL drive. Because our clocks are synchronized according to us (a very important qualifier), the FTL drive will arrive at Alpha Centauri when it left. Everyone will agree that when the space ship leaves at Jan 1, 2018 12:00:00.0 AM according to the Earth clock, it will arive on Jan 1, 2018 12:00:00.0 AM according to the clock we arranged to put on Alpha Centauri.

So far so good, but what if the Vogons are cruising along at some appreciable v ≤ c (assuming they can) directly away from Earth. They will see that the FTL ship departs from Earth on Jan 1, 2018 12:00:00.0 AM according to Earth's clock, and arrive at Alpha Centauri on Jan 1, 2018 12:00:00.0 AM according to our Alpha Centauri clock, BUT the Vogons think that our clocks are out of synch, and any duration deduced from them over any notable distance will need adjustment. They have their own set of clocks scattered throughout the universe, synchronized and comoving with them, and use them to decide how much time has passed between the departure of the FTL rocket from Earth and its arrival at Alpha Centauri.

Suppose that the Vogons are traveling at ~1c, just to make things neat and tidy. Then if they pass Alpha Centauri on Jan 1, 2018 12:00:00.0 AM according to the ship's onboard clocks, then because of time dialation, they must have passed the Earth on Jan 1, 2018 12:00:00.0 AM according to the ship's onboard clocks, too. But they know that Alpha Centauri is 4.67 light years away, so it took them actually ~4.67 years to travel that distance, so they had seen the Earth's clock read sometime in May 2013 when they passed it. So Jan 2018 on their clock translates to May 2013 in Earth time, and thus Alpha Centauri time, and the Alpha Centauri clock placed by earth is BADLY out of synch with Earth time. (They are valid in thinking this, because length contraction â€"which occurs hand-in-hand with time dialationâ€" places Alpha Centauri much much closer to the Earth, according to the Vogons.)

Nothing unusual has happened so far, because according to the Vogons, the FTL ship has left Earth after they did, and arrived at Alpha Centauri just as they do. But let's suppose that the Vogons immediately take on the FTL drive (thus making it comoving with their own clocks) and send it back. Because the FTL drive obeys relativity, it travels only to a place if a comoving and synchronized clock in the area reads the same time as the place it started. The clocks the FTL ship is now comoving with are the Vogon's clocks, so it travels to places where a Vogon clock reads Jan 2018, and the clock buzzing past Earth at the time reading Jan 2018 when Earth's own clock is reading May 2013.

And there is the time travel. The FTL ship left the Earth in Jan 2018 (Earth time), but returned to the earth May 2013 (Earth time).

Of course, this is only possible if FTL works as advertized. We don't see real FTL travel or communications in our universe, so causality is preserved.

-----

Quote from: Mousetrap on August 16, 2018, 10:36:04 AM
How about building the clocks on the Earth, load it on a rocket, blast all 3 into space, stop halfway, send the clocks with the same speed to the Moon and Earth with smaller rockets that travel at snailpace to both positions.
Why all the fuss to try and show that speed will have an effect on a clock?

We don't need any fuss to show that. We can see it on atomic clocks on commercial airliners, but only to the tune of 50 nanoseconds or so over many flight hours. It is a subtle effect, which is why we don't see much of it in our ordinary lives, but it's there and is detectable clear as day with sensitive experiments.

Also, very sensitive technologies have to account for it, too. The GPS satellites (and similar geopositioning systems) have to take into account both special and general relativity when keeping time. Otherwise, time on earth and on the satellites would get badly out of synch and introduce horrible errors into positional accuracy, many tens of meters. When you have to take it into account with a real technology, it's a thing.

----

Quote from: Mousetrap on August 16, 2018, 10:53:10 AM
I think you missed out when I wrote, lets do this experiment again, but from the time-frame of the Moon.
I want us to be clear about my claim.
If the observer on the Moon looks at the Earth at 12H00, when the rocket also blasts off from the Earth at 12H00, the observer on the Moon will see the clock on the Earth and the clock on the Rocket are both one second before Noon.
"Looking" requires light, which travels at the speed of light. Therefore, the very instant (according to the Earth and the Moon â€" that qualifier is very important) that the rocket leaves the Earth, the person looking from the moon sees the rocket on the pad at T-minus 1 second, and the Earth's clock reads 11:59:59, because the light of the Earth clock reading 12:00:00 and the rocket launching hasn't reached them yet. He doesn't see the launch at 12:00:00 Earth time until a second later Earth time. If light takes a substantial amount of time to travel from place to place, you kinda have to take that into account.

If Moon time is synched with Earth time, then he will see the launch at 12:00:01 Moon time, but observe the launch at 12:00:00 Moon time because he can do math and he knows the clocks are synchronized.

Quote
The clock on the Moon will move forward for one second, and the rocket will arrive at the same instance.
Yes, it takes one second for the rocket to travel from Earth to the Moon by Earth's observations, and one second for the rocket to travel from the Earth to the Moon by the Moon's observations. They are both comoving with each other, and will measure time the same way. It's only with objects moving relative to each other that time dialation shows up. The Earth and Moon are not moving relative to each other in our gedanken, so no dialation there.

Quote
The clock on the Earth will appear to be noon, but both the Rocket and Moon clock will show one second past Noon.
The rocket's clock is moving relative to both the earth and the moon. To say that the rocket's clock shows one second past noon is begging the question. It's what you have set out to prove; you can't take it as a given.

Quote
Therefore, the Earth clock seems to be late with 1 second to the one on the moon,
Well, yeah, but they also know that the Earth clock is exactly one second ahead of what it seems from the Moon, because everyone in your model knows physics. Again, if light takes a substantial amount of time to travel from place to place, you kinda have to take that into account.

Quote
and the clock on the Rocket seemed to have moved forward 2 seconds on its journey of one second.
No. That's the exact bone of contention. The guys on the moon can see perfectly well that the rocket launches when the clock on Earth (which is near the rocket) reads 12 noon. They see this image at 12:00:01 moon time, but they know that the light they see left Earth one second earlier. Once again, if light takes a substantial amount of time to travel from place to place, you kinda have to take that into account. Indeed, this image arrives at the moon and is seen at almost exactly the same time the rocket itself arrives (because they're traveling at almost the same speed, remember?). The moon sees that the rocket launched at exactly 12 noon, as measured by the local Earth clock (which is in synch with their own) and by the shipboard clock (which was at that moment in synch with their own), and arrives at one second past noon at the Moon.

The rocket will observe that they launch at 12 noon on Earth, and arrive at one second past noon on the Moon, according to Earth-Moon synchronized clocks. Their own shipboard clock remains at about 12 noon during the trip, however, because to the ship and everyone on it, the trip took less than a second and started at 12 noon. They deduce from this that the moon all of a sudden became much much closer than one light second away (because they were traveling near the speed of light for much much less than a second), yet the moon registered about one second for them to make the trip â€" ergo, the moon's clock slowed down, and since the moon clock stared and remains in synch with the Earth clock, the Earth clock must have slowed down, too.

Under no circumstances will anyone observe that the rocket's clock advances by two seconds. Earth doesn't observe it. The moon doesn't observe it. The rocket doesn't observe it.

Now, the moon suddenly becoming much closer is unphysical, but then, instantaneous acceleration to speed is equally unphysical. I've already given you that gimme, so you can't ding me for hand-in-hand gimme of the apparent teleportation of the moon.
Title: Re: Time dilation, length contraction, Relativity and the Bible!
Post by: Baruch on August 16, 2018, 12:56:14 PM
Quote from: Mousetrap on August 16, 2018, 10:31:56 AM
OK, so why do they say time will turn back if we travel faster than the speed of light?

It works this way.

Lets say we travel to A Centauri. 4.67 Light Years, and we travel at the speed of light.
There is this huge clock on Earth and we can see the time, date and years.
As we travel away from Earth, we will see the clock stands still, because we are travelling with the light as we departed from Earth.

However, If we travel at 2X C, the rocket will gain on the light that was travelling at C from the Earth, and it will seems as if this clock is turning in reverse,...
Using the formulas of Lorentz, it seems as if Time dilated!

No, it did not, all that happened is that the light we observed in our reference frame, was light we gained on as we traveled and gained on.

"There is this huge clock on Earth and we can see the time, date and years." ... far enough away, you are seeing the clock in the past, not in the present.
Title: Re: Time dilation, length contraction, Relativity and the Bible!
Post by: Unbeliever on August 16, 2018, 05:23:05 PM



https://www.youtube.com/watch?v=GguAN1_JouQ




https://www.youtube.com/watch?v=GgvajuvSpF4
Title: Re: Time dilation, length contraction, Relativity and the Bible!
Post by: Mike Cl on August 16, 2018, 06:51:04 PM
Quote from: Mousetrap on August 16, 2018, 02:09:27 AM
Why, Dont you understand how many apples I have?
I know how many brains you have--zip, nil, zero................
Title: Re: Time dilation, length contraction, Relativity and the Bible!
Post by: Baruch on August 16, 2018, 07:17:10 PM
Quote from: Unbeliever on August 16, 2018, 05:23:05 PM


https://www.youtube.com/watch?v=GguAN1_JouQ




https://www.youtube.com/watch?v=GgvajuvSpF4

Time breaks down?  That is happening a lot these days, with Americans.  Maybe after we all have a good cry, we can sober up.
Title: Re: Time dilation, length contraction, Relativity and the Bible!
Post by: aitm on August 16, 2018, 08:25:59 PM
Your "god" orders babies be smashed(dashed) against the rocks and little girls be raped...and you're trying to pretend that physics can prove your god exists? The real simple stunner is that people like you think that pos is worthy of defending.
Title: Re: Time dilation, length contraction, Relativity and the Bible!
Post by: Jason Harvestdancer on August 17, 2018, 10:13:34 AM
How odd.  I am usually pretty good at interpreting babble into English, but this one actually has me stumped.
Title: Re: Time dilation, length contraction, Relativity and the Bible!
Post by: Baruch on August 17, 2018, 01:17:32 PM
Quote from: Jason Harvestdancer on August 17, 2018, 10:13:34 AM
How odd.  I am usually pretty good at interpreting babble into English, but this one actually has me stumped.

Your Babble fish died.  Contact the Vogons for a new one ;-)
Title: Re: Time dilation, length contraction, Relativity and the Bible!
Post by: Mousetrap on August 22, 2018, 07:08:18 AM
Quote from: Hakurei Reimu on August 16, 2018, 10:53:27 AM
It will take one second from any observer but you. For you, it will be instantaneous. One moment, your on Earth, and the next, you're on the Moon.
Every physicist will say that YOUR time will stand still. You will not experience time as you travel at the speed of light. Everyone else, however, will not be affected.

Just work it out again from this perspective.
What was the time you left Earth?
Noon.
What time did you arrive at the Moon?
1 sec past Noon.
Why?
Because you were travelling at C.
Which will be, 300 000 Km per second!
You traveled for ONE SECOND

How did time stand still?

It never did.
This is exactly what is wrong with your interpretation on the Lorenz transformation.
You are measuring the time of S' as seen from S, and do not realize that what you see is the LIGHT travelling back to S.
The watch was still ticking on S', and on the S frame.

Sorry, you are wrong.
Time never stopped on the rocket.
If it did, you would leave S (Earth) at noon and arrive on S (Moon)' at noon. This when S(Moon)' is at one second past Noon. They will never see you because you are one light second in the past. Look again...
But you did not arrive at Noon, because you were travelling at C, therefore the journey will take one second!
How do you get instantaneous travelling with you moving at the speed of C ?

What you are trying to tell me is that light travels from the Sun to Earth just more than 8 Minutes 20 seconds.
But if you travel on a space ship at the speed of light, you ill arrive here 8 minutes and 20 seconds before the light?

Do you see your error?

The above is an experiment where measurements of science on the speed of light proves Time dilation incorrect.

Another experiment for you to think about.

NASA sends Lazer beams to reflectors on the moon.
It takes about 2.5 seconds round trip from the Earth to the Moon and back.

If you were traveling at C when the laser fires away, you want to tell me you will be back 2.5 seconds before the Laser returns?

Naaaa, you misinterpret Lorenz' transformation formula.
Title: Re: Time dilation, length contraction, Relativity and the Bible!
Post by: Mousetrap on August 22, 2018, 07:31:24 AM
Quote from: Hakurei Reimu on August 16, 2018, 10:53:27 AM


Your inability to comprehend relativity doesn't mean it's wrong. It just means you don't comprehend relativity.
IF it were possible to go faster than light, then we can leverage it to go back in time. If you were actually to read up on what relativity says about how to do this, if it were possible, you need at least two legs to make it happen. Here's a discussion about the problem of FTL and time travel from Physicsmatt:

Allow me to show you how wrong you interpret the Speed of light with the wrong interpretation of Lorenz' transformation.

1. If the Moon is 300 000 Km from the Earth.
2. I travel on a spaceship to the Moon at C,
...it will take 1 second to arrive there.
3. If I travel at 2X C, It will take 0.5 second to get there.
4. At 4X C, it will take 0.25 second...
5. At 8X C, 0.125 second.

So, when will we travel faster than Time?
Or as you claim, we will go back in time!

Never, you can travel at infinite X speed of light, it will take time!

What now?
I thought you knew all about SR and GR?
Title: Re: Time dilation, length contraction, Relativity and the Bible!
Post by: Mousetrap on August 22, 2018, 07:41:21 AM
Quote from: Hakurei Reimu on August 16, 2018, 12:53:34 PM

----
"Looking" requires light, which travels at the speed of light. Therefore, the very instant (according to the Earth and the Moon â€" that qualifier is very important) that the rocket leaves the Earth, the person looking from the moon sees the rocket on the pad at T-minus 1 second, and the Earth's clock reads 11:59:59, because the light of the Earth clock reading 12:00:00 and the rocket launching hasn't reached them yet. He doesn't see the launch at 12:00:00 Earth time until a second later Earth time. If light takes a substantial amount of time to travel from place to place, you kinda have to take that into account.

If Moon time is synched with Earth time, then he will see the launch at 12:00:01 Moon time, but observe the launch at 12:00:00 Moon time because he can do math and he knows the clocks are synchronized.
Yes, it takes one second for the rocket to travel from Earth to the Moon by Earth's observations, and one second for the rocket to travel from the Earth to the Moon by the Moon's observations. They are both comoving with each other, and will measure time the same way. It's only with objects moving relative to each other that time dialation shows up. The Earth and Moon are not moving relative to each other in our gedanken, so no dialation there.
The rocket's clock is moving relative to both the earth and the moon. To say that the rocket's clock shows one second past noon is begging the question. It's what you have set out to prove; you can't take it as a given.


You are running in circles.
In one instance you say that if you travel at C, then you arrive at the Moon instantaneous!
Then you say, it will take one second.
Then you say, the clock on the space ship will stand still, then you say, the clocks on the Earth and Moon will have no time dilation, becaust it needs to move relative to each other, which is correct, but you forget that the rocket is the moving time frame.

So, what is it, will it take you one second at C to travel to the Moon, or instantaneous.
Title: Re: Time dilation, length contraction, Relativity and the Bible!
Post by: Mousetrap on August 22, 2018, 08:34:31 AM
(https://i.imgur.com/dFIeqFE.jpg)

The faster We travel, the quicker we arrive.
Even when traveling at 9X the speed of light!

So, where do we go back in time when traveling faster than light?

We dont!

Time can not be changed, The Bible is correct again.
Title: Re: Time dilation, length contraction, Relativity and the Bible!
Post by: Hydra009 on August 22, 2018, 12:57:27 PM
Quote from: Mousetrap on August 22, 2018, 07:08:18 AMNASA sends Lazer beams to reflectors on the moon.
Getting in a physics argument with someone who seriously spells laser with a z.  Oy vey.

Light amplification by zimulated emission of radiation?  Little wonder that this isn't taken very seriously.
Title: Re: Time dilation, length contraction, Relativity and the Bible!
Post by: Hakurei Reimu on August 22, 2018, 01:17:12 PM
Quote from: Mousetrap on August 22, 2018, 07:08:18 AM
Just work it out again from this perspective.
What was the time you left Earth?
Noon.
By earth's clock.

Quote
What time did you arrive at the Moon?
1 sec past Noon.
By the moon's clock. I have to use the moon's clock because I can't see the time on the Earth's clock yet, being that it's one light second behind me and not here where I can see it. But that's not a problem if I think that the two clocks are synchronized.

Quote
Why?
Because you were travelling at C.
You were traveling at c? On the rocket, you see yourself traveling at 0 mph. The earth and the moon are traveling at c in your frame of reference. You see everything operating normally, including that light travels at c relative to you in your frame of reference, and that clocks traveling with you tick normally.

Quote
Which will be, 300 000 Km per second!
You traveled for ONE SECOND
Only from the Earth-Moon perspective. From the rocket's perspective, the earth and moon clocks are badly out of synch, and therefore I don't have to believe that subtracting the time on one clock from the time on the other clock has anything to do with the amount of time that I've traveled.

Put another way, if I see two clocks side by side and one is an hour behind the other, does this mean that one hour passed between the time I took to look from one clock to the other? Of course not. I simply say that the clocks are out of synch and their difference doesn't mean a thing.

Quote
How did time stand still?

It never did.
Of course YOU never see your time stand still. How could you? If your time is standing still, you never go from not realizing your time is standing still to having realized your time is standing still. Physically impossible.

What we observe, and what special relativity predicts, is that clocks moving relative to you tick slowly. And they do. Again, we have a wealth of evidence that it happens and technologies that have to take the effect into account to operate properly.

You have not shown that the guy on the rocket has any reason at all to believe that the Earth and moon clocks are synchronized with each other and as such should be believed when they arrive at the moon with the moon clock showing one second past noon when they arrive and the Earth clock having shown noon when they left. That difference only means something if they believe the two clocks are in synch, but the rocket can perform an experiment that shows that, according to them, the earth and moon clocks are not in synch and there's no reason to believe that a comparison between the two represents their travel time. We moon-dwellers know that the clocks were always synchronized, but how do we prove that to the rocket passengers?

Quote
This is exactly what is wrong with your interpretation on the Lorenz transformation.
I've been tested on special relativity and I've passed. I think I know it better than you do.

Quote
You are measuring the time of S' as seen from S, and do not realize that what you see is the LIGHT travelling back to S.
The watch was still ticking on S', and on the S frame.

Sorry, you are wrong.
Time never stopped on the rocket.
I never said that the time would stop for the people on the rocket. I said that time on the rocket would be seen to hardly progress for people on Earth and on the moon. You have yet to show that this statement is wrong.

Quote
If it did, you would leave S (Earth) at noon and arrive on S (Moon)' at noon.
<snip>
Do you see your error?
The only thing I see is you don't know your special relativity. Every tool is bad if you use it wrong, and you are, in fact, using special relativity wrong.

The Earth and the moon have the same reference frame, S. The only thing here with a different reference frame is the rocket, with frame S'. The Earth and moon can keep their clocks in synch, and their clocks will tick at the same rate. Even Einstein will agree that this can be done. What he argues, and what experiment shows, is that the rocket cannot keep its clocks in synch with the earth and moon because his clocks will tick at a different rate from them.

So, no, I'm not trying to tell you that the rocket will arrive at noon on the moon. The rocket leaves the earth with the Earth's clock showing 12:00:00, and arrives on the moon with the Moon's clock showing 12:00:01. We agree on this. Please stop harping on this. These two clocks do not show time dialation between them, and would not show time dialation because in special relativity, the only clocks that show time dialation with respect to each other are clocks that are moving relative to each other, and the earth and moon clocks are not moving relative to each other.

There is only one clock that will show time dialation in your example, and that is the shipboard clock â€" the one that is moving relative to the other two. The shipboard clock reads 12:00:00 when it arrives on the moon. It will show about one second behind the moon clock. When the people on the moon look at the Earth clock through their telescopes, they will see that clock read 12:00:00, too, but they won't worry about that because they know that the Earth clock is one light second away, and as such it will take the image of the Earth clock reading 12:00:01 one second to get here. But the shipboard clock reads 12:00:00, and it's right here on the moon, right next to the moon clock reading 12:00:01, and everyone can see it's out of synch with the moon clock (and the Earth clock because they know the earth clock is in synch with the moon clock).

The SR ignorant among them will be scratching their heads trying to figure out how the shipboard clock got so out of synch with the Earth-Moon. Even the ones on the rocket will be scratching their heads, because they saw nothing unusual happen (other than the horrendous acceleartions at both ends) â€" and they will also be puzzled why their SR ignorant moon dwellers are insisting that the trip took one second when it was near-instantaneous for them. The people who do know SR don't see that anything unusual has happened; the universe is working as designed.

This, by the way, is point-for-point identical with the experiment where atomic clocks were loaded onto airliners and flown around the world, and coming back home with those atomic clocks significantly out of synch with home-based atomic clocks. And furthermore, this was NOT a gedanken experiment. This actually happened. Atomic clocks traveling on airliners did in fact come back home significantly behind land-based ones.

Time dialation is observed. It's not a theoretical thing; we observe it regularly in our experiments. It doesn't matter how much you crow about your (bad) math and physics, that is still the fact that you need to explain. All I see you doing is sticking your fingers into your ears and denying a readily observable effect.

Quote
The above is an experiment where measurements of science on the speed of light proves Time dilation incorrect.
You compared a stationary clock with a stationary clock, which is exactly the situation where you would expect to see no time dialation in SR.

Quote
Naaaa, you misinterpret Lorenz' transformation formula.
For someone who has obviously never properly used a Lorentz transformation, this is completely hillarious. No, I don't say that the rocket will arrive before the light beam returns. The rocket will not even see anything unusual happens with its clocks. The wierdness only begins when we compare clocks from different reference frames. You are not doing this, so of course you don't see the time dialation.

---

Quote from: Mousetrap on August 22, 2018, 07:31:24 AM
Allow me to show you how wrong you interpret the Speed of light with the wrong interpretation of Lorenz' transformation.

<snip>

So, when will we travel faster than Time?
Or as you claim, we will go back in time!

Never, you can travel at infinite X speed of light, it will take time!
You never use the Lorentz transformation even once. Ever. How can you show that my interpretation of the Lorentz transformation is wrong when you never use it at all? No, calculating the duration of travel of the super-rocket in the Earth-Moon frame is not a Lorentz transformation. Time travel will not show up here. You have to consider the frame of an object that moves relative to the starting frame to see time travel.

Quote
What now?
I thought you knew all about SR and GR?
Like I said before, it won't show up here. The Earth and the Moon are not different reference frames, and as such, you can propose FTL and it will seem to work without violating causality, and I'm telling you it won't show up until you consider another reference frame (not another location in the same reference frame) and â€"surprise, surpriseâ€" I'm completely right. You didn't change frames of reference, so it didn't show up. Causality violations don't show up in your calculations because you OMIT A CRITICAL STEP IN SHOWING THAT IT HAS OCCURRED. It's like someone who is trying to show a factory is broken by not turning on the final assembly machine.

MT: "Look! See? It doesn't make sedans properly! This factory is broken!"
HR: "You didn't turn on the final machine. What did you expect? Turn on the final machine, then see what happens."
MT: "See? The factory still isn't turning out proper sedans! I didn't turn on the final machine, but I don't have to! The factory is broken!"
HR: *facepalm*

It's very hard to show someone who doesn't know a subject that they're wrong. You clearly don't even know where to begin, because you are talking about using a Lorentz transformation within the same reference frame. It shows that you don't even have a basic grasp of what you're talking about. Again, it's like a man who claims to be a car mechanic and doesn't know how to change a tire.

----

Quote from: Mousetrap on August 22, 2018, 07:41:21 AM
You are running in circles.
In one instance you say that if you travel at C, then you arrive at the Moon instantaneous!
Only by the shipboard clock. The Earth-Moon clocks disagree with the shipboard clock.

Quote
Then you say, it will take one second.
Yes, by the Earth clocks and the moon clocks. The shipboard clock disagrees with the Earth-Moon clocks.

Believe it or not, these are two different statements. I do not assume that the two clock systems are measuring the same time, because the two clock systems are in different frames of reference. A clock can only properly measure time of the frame of reference in which it is stationary. Outside the frame of reference, all bets are off. You need to find the relation of time and space between the two frames of reference to progress further. That's what the Lorentz transformation does: it allows you to relate measurements taking place in one frame of reference to measurements in another.

Quote
Then you say, the clock on the space ship will stand still,
In the Earth-Moon time system.

Quote
then you say, the clocks on the Earth and Moon will have no time dilation, becaust it needs to move relative to each other, which is correct,
Not to each other. They're not moving relative to each other. Time dialation only occurs between clocks moving relative to each other, and the Earth and moon clocks are not moving relative to each other.

Time dialation doesn't mean anything unless you're referencing another clock.

Quote
but you forget that the rocket is the moving time frame.
I didn't forget. I wasn't talking about the shipboard clock right then. According to the shipboard clock (in which the Earth and moon clocks are moving relative to the shipboard clock), the rocket will see them tick slowly.

Quote
So, what is it, will it take you one second at C to travel to the Moon, or instantaneous.
This statement betrays your ignorance. The answer depends on which frame of reference you're talking about, and you're asking if that answer doesn't.

From the rocket: Instantaneous, mostly due to the fact that in the rocket's frame of reference, the earth and moon are located practically on top of each other (length contraction).
From the Earth-Moon system: One second. Because the earth and moon are one light second apart in the Earth-Moon frame of reference, and the rocket is traveling at c in the Earth-Moon frame of reference.

These two statements are both true. The different frames of reference is what makes the difference between the two statements. Yes, that means that the two frames of references will disagree about how long the trip takes. That's why it's the theory of RELATIVITY and not the theory of ABSOLUTES.

----

Quote from: Mousetrap on August 22, 2018, 08:34:31 AM
<snip>
So, where do we go back in time when traveling faster than light?

We dont!

Time can not be changed, The Bible is correct again.
This is just a restatement of an earlier argument, one that I've already addressed. Again, nothing unusual happens if you only stay within one reference frame as you have done. (No, the moon is not a distinct reference frame from the Earth.)

And, again, time dialation is experimentally confirmed. Your bible can suck on it.

----

Quote from: Hydra009 on August 22, 2018, 12:57:27 PM
Getting in a physics argument with someone who seriously spells laser with a z.  Oy vey.

Light amplification by zimulated emission of radiation?  Little wonder that this isn't taken very seriously.

I admit, this is a little like trying to teach a camel how to bathe.
Title: Re: Time dilation, length contraction, Relativity and the Bible!
Post by: Mousetrap on August 23, 2018, 06:58:43 AM
Quote from: Hydra009 on August 22, 2018, 12:57:27 PM
Getting in a physics argument with someone who seriously spells laser with a z.  Oy vey.

Light amplification by zimulated emission of radiation?  Little wonder that this isn't taken very seriously.
Thank you for volunteering to correct my spelling.
Highly appreciated pal!

Oh, by the way, do you know English is my 3rd language, and I am the first to say, I am bad, Bad , Bad with the British voice?

How many languages do you speak?
Title: Re: Time dilation, length contraction, Relativity and the Bible!
Post by: Mousetrap on August 23, 2018, 07:24:15 AM
Quote from: Hakurei Reimu on August 22, 2018, 01:17:12 PM
By earth's clock.
Correct
Quote from: Hakurei ReimuBy the moon's clock. I have to use the moon's clock because I can't see the time on the Earth's clock yet, being that it's one light second behind me and not here where I can see it. But that's not a problem if I think that the two clocks are synchronized.
Correct
Quote from: Hakurei ReimuYou were traveling at c? On the rocket, you see yourself traveling at 0 mph. The earth and the moon are traveling at c in your frame of reference. You see everything operating normally, including that light travels at c relative to you in your frame of reference, and that clocks traveling with you tick normally.
Correct
Quote from: Hakurei ReimuOnly from the Earth-Moon perspective. From the rocket's perspective, the earth and moon clocks are badly out of synch, and therefore I don't have to believe that subtracting the time on one clock from the time on the other clock has anything to do with the amount of time that I've traveled.

Put another way, if I see two clocks side by side and one is an hour behind the other, does this mean that one hour passed between the time I took to look from one clock to the other? Of course not. I simply say that the clocks are out of synch and their difference doesn't mean a thing.
Of course YOU never see your time stand still. How could you? If your time is standing still, you never go from not realizing your time is standing still to having realized your time is standing still. Physically impossible.
Not so, no.
In our instance,the clocks are indeed synchronized.By whatever means possible. WE can have all 3 clocks loaded on a rocket, travel halfway, load one clock on another ship, travel to the moon with one, and return the other 2 to the earth with the first rocket.
This will render a clock on the Moon that is exactly the same as the 2 on Earth. However, if we look at the one on the Moon, from the Earth, it will seem to be 1 second before our clock on Earth. And vice versa, because it takes 1 second for the light to travel back to each other.

Quote from: Hakurei ReimuWhat we observe, and what special relativity predicts, is that clocks moving relative to you tick slowly. And they do. Again, we have a wealth of evidence that it happens and technologies that have to take the effect into account to operate properly.
And here we go again with this great "keep your mouth shut because we have evidence by experiment-argument"
Come on man, if you move a cesium clock to a high altitude, it will have an effect on the frequency it runs at due to gravitational differences.
If you measure Muons at high altitude, and at sea level, and manipulate the Iron covering accordingly, obviously you will get the same amount of Muons in both experiments.
Oh, and dont even come with the GPS proves relativity argument. as if it will have any effect on 1 000 Km from Earth.
All I could see was scientists who wanted to become the famous one to prove the SR and GR with experimentation. Just to get more grants! Dont fall for that man, go and see for yourself on how they manipulated the calculations.
Quote from: Hakurei ReimuYou have not shown that the guy on the rocket has any reason at all to believe that the Earth and moon clocks are synchronized with each other and as such should be believed when they arrive at the moon with the moon clock showing one second past noon when they arrive and the Earth clock having shown noon when they left. That difference only means something if they believe the two clocks are in synch, but the rocket can perform an experiment that shows that, according to them, the earth and moon clocks are not in synch and there's no reason to believe that a comparison between the two represents their travel time. We moon-dwellers know that the clocks were always synchronized, but how do we prove that to the rocket passengers?
I did explain my experiment to deliver 3 clocks at 2 different positions with a money back guarantee.
Quote from: Hakurei ReimuI've been tested on special relativity and I've passed. I think I know it better than you do.
I never said that the time would stop for the people on the rocket. I said that time on the rocket would be seen to hardly progress for people on Earth and on the moon. You have yet to show that this statement is wrong.
This was what You said!
Why did you argue about it?
You were the one who claimed that if you travel at the speed of C to the Moon, you will be there instantaneous, with no time that passed for you!!
Now you are telling me that "It will only look like it to the people on Earth!"
And this is what I have been telling you all the time!
You are measuring how LIGHT WILL APPEAR TO DIFFERENT TIME FRAMES WHEN TRAVELLING AT HIGH SPEED!
Time will not stop or go faster!
I also passed my examination on SR and GR.
But the difference between us is that I understood what I learned!
Quote from: Hakurei ReimuThe only thing I see is you don't know your special relativity. Every tool is bad if you use it wrong, and you are, in fact, using special relativity wrong.
realy?

Quote from: Hakurei ReimuThe Earth and the moon have the same reference frame, S. The only thing here with a different reference frame is the rocket, with frame S'. The Earth and moon can keep their clocks in synch, and their clocks will tick at the same rate. Even Einstein will agree that this can be done. What he argues, and what experiment shows, is that the rocket cannot keep its clocks in synch with the earth and moon because his clocks will tick at a different rate from them.
So I am correct!
It is only the appearance of Time dilation we observe!
It has sweet nothing to do with time that actually goes faster, or slower, or even stops!
You just proved me correct and Neil de Grase Tyson et al totally incorrect.
And the Bible true!

Quote from: Hakurei ReimuSo, no, I'm not trying to tell you that the rocket will arrive at noon on the moon. The rocket leaves the earth with the Earth's clock showing 12:00:00, and arrives on the moon with the Moon's clock showing 12:00:01. We agree on this. Please stop harping on this. These two clocks do not show time dialation between them, and would not show time dialation because in special relativity, the only clocks that show time dialation with respect to each other are clocks that are moving relative to each other, and the earth and moon clocks are not moving relative to each other.
Great! I agree!

Quote from: Hakurei ReimuThere is only one clock that will show time dialation in your example, and that is the shipboard clock â€" the one that is moving relative to the other two. The shipboard clock reads 12:00:00 when it arrives on the moon. It will show about one second behind the moon clock.
Why?
It left at Noon on Earth and traveled for one second!
the Moon clock was at noon when the rocket left Earth, and it took the rocket 1 second to get there.
Therefore upon arrival of the rocket on the Moon, both will be one second past noon.
Quote from: Hakurei ReimuWhen the people on the moon look at the Earth clock through their telescopes, they will see that clock read 12:00:00, too, but they won't worry about that because they know that the Earth clock is one light second away, and as such it will take the image of the Earth clock reading 12:00:01 one second to get here. But the shipboard clock reads 12:00:00, and it's right here on the moon, right next to the moon clock reading 12:00:01, and everyone can see it's out of synch with the moon clock (and the Earth clock because they know the earth clock is in synch with the moon clock).
So, now you say you traveled faster than the speed of light!
And
Now you are again contradicting yourself.
what you say is that the clock for some magical reason just stopped to run during its travel. The occupants saw the clock run, but when they came to the Moon it was ctually standing still.
What university did you write that exam?

Quote from: Hakurei ReimuThe SR ignorant among them will be scratching their heads trying to figure out how the shipboard clock got so out of synch with the Earth-Moon. Even the ones on the rocket will be scratching their heads, because they saw nothing unusual happen (other than the horrendous acceleartions at both ends) â€" and they will also be puzzled why their SR ignorant moon dwellers are insisting that the trip took one second when it was near-instantaneous for them. The people who do know SR don't see that anything unusual has happened; the universe is working as designed.
In their deranged minds yeh!
What mysterious magic made the clock stop on its way to the moon.
You just claimed that the rocket can travel instantaneous, but the light that traveled from the Earth to the Moon took one second!

Quote from: Hakurei ReimuThis, by the way, is point-for-point identical with the experiment where atomic clocks were loaded onto airliners and flown around the world, and coming back home with those atomic clocks significantly out of synch with home-based atomic clocks. And furthermore, this was NOT a gedanken experiment. This actually happened. Atomic clocks traveling on airliners did in fact come back home significantly behind land-based ones.
So what is insignificant in your point of view?
Quote from: Hakurei ReimuYou compared a stationary clock with a stationary clock, which is exactly the situation where you would expect to see no time dialation in SR.
For someone who has obviously never properly used a Lorentz transformation, this is completely hillarious. No, I don't say that the rocket will arrive before the light beam returns.
You did!
You said that the light that traveled next to the rocket will show the Earth clock time at Noon as it left the earth.
The Moon clock will show one second after noon.
but the occupants on the rocket arrived at one second before noon.
Quote from: Hakurei ReimuThe rocket will not even see anything unusual happens with its clocks. The wierdness only begins when we compare clocks from different reference frames. You are not doing this, so of course you don't see the time dialation.
Now I am astounded.
You flew a rocket at the speed of light over 300 000 Km, and somehow won the race against a light beam travelling at the same distance and you beat him to a full track length!!!!


Tell you what.
I only want to know from you this.
If you travel at C, will time stand still for you?
You are going to say Yes, but I want to know this:
Do you believe this because when you calculate the Lorenz transformation, you are observing where Light will be from reference frame S looking upon reference frame S'?

If so, do you agree that time does not stand still in S', but it seems to do so looking from S?
do you see the error of interpretation on Lorenz?
What you calculate is where the point of light is when looking from S to S'
It does not mean that the clock in S' stopped working.
It only means that you will see the time as light returns from S' to you.
just as we will see from earth, the clock on our rocket shows noon when arriving on the Moon, but the real time is one sec past noon.

This is the error on SR and GR.
You are interpreting observation with reality, which is nothing but observing old light!.


Title: Re: Time dilation, length contraction, Relativity and the Bible!
Post by: Baruch on August 23, 2018, 09:56:14 AM
"You are interpreting observation with reality, which is nothing but observing old light!."

Exactly the point of Einstein's redefinition in philosophy of physics.  He was influenced by Ernst Mach.  Who was anti-Platonist.  For them, only observations are relevant, reality is not.  Newton's absolute reality (aka Platonist) is not observable in practice, though ordinary physics comes close.
Title: Re: Time dilation, length contraction, Relativity and the Bible!
Post by: Sal1981 on August 23, 2018, 10:37:54 AM
The thing about relativity is that light ALWAYS travels at C in a vacuum, which I reckon where the confusion comes from. The Newtonian of adding speeds together for two reference systems simply doesn't happen for light in a vacuum.

Here's a (non-exhaustive) brief explanation of what Mousetrap simply isn't getting:
https://www.youtube.com/watch?v=9BFKqIoqSIY
Title: Re: Time dilation, length contraction, Relativity and the Bible!
Post by: Hakurei Reimu on August 23, 2018, 11:17:47 AM
Quote from: Mousetrap on August 23, 2018, 07:24:15 AM
Not so, no.
In our instance,the clocks are indeed synchronized.By whatever means possible. WE can have all 3 clocks loaded on a rocket, travel halfway, load one clock on another ship, travel to the moon with one, and return the other 2 to the earth with the first rocket.
Yes, the clocks start out synchronized. They don't remain synchronized. That's the problem that you are ignoring instead of addressing. The shipboard clock reads 12:00:00 instead of 12:00:01 on the moon at 12:00:01 (moon time) because it ticks slower in the Earth-Moon frame as it travels, not because it somehow launched at 11:59:59.

Quote
This will render a clock on the Moon that is exactly the same as the 2 on Earth. However, if we look at the one on the Moon, from the Earth, it will seem to be 1 second before our clock on Earth. And vice versa, because it takes 1 second for the light to travel back to each other.
Yes, we agree on this. Again, stop harping about this. The clocks all start synchronized, reading 12:00:00 in the same "now". The problem is that they don't remain synchronized in your example.

Quote
And here we go again with this great "keep your mouth shut because we have evidence by experiment-argument"

https://www.youtube.com/watch?v=LIxvQMhttq4

Evidence and experiment is king in science. It doesn't matter what the bible says, and it doesn't matter what you say, if a hypothesis doesn't agree with experiment, it's wrong.

The fact remains that we have observed the time dialation effect in experiments. The fact remains that we use technologies that must compensate for the time dialation effect or it will fail. There's no wiggle room for you to fit your constancy of time through. Time dialation is real.

Quote
Come on man, if you move a cesium clock to a high altitude, it will have an effect on the frequency it runs at due to gravitational differences.
And we know the magnitude of the gravitational redshift and can compensate for it. There's still a residual, which cannot be accounted for by the changes in altitude.

And of course, the reason why gravitational differences cause time dialation is because of GR, of which SR is a special case, and the time dialation of GR can be justified and deduced by triangulating off of SR.

And furthermore it's still time dialation. Whether it's caused by gravity or speed, time still is not uniform throughout the universe.

Quote
If you measure Muons at high altitude, and at sea level, and manipulate the Iron covering accordingly, obviously you will get the same amount of Muons in both experiments.
You made that up. You are now asserting that these scientists are either idiots or frauds. Sorry, replication would quickly expose either.

Knowing how many muons reach the earth's surface gives you data on how much radiation there is in space, and as such, it is important for aerospace applications. Thus, anything that affects how you measure radiation from outer space is going to be important to know about. Knowing the muon lifetime is also important to nuclear and particle physics, because the forces that affect the muon lifetime also affect the other particles in the Standard Model, and as such also affects how nuclear reactors and nuclear bombs work. It's important to get this stuff right, or your nuclear power and nuclear weaponry aren't going to work.

Quote
Oh, and dont even come with the GPS proves relativity argument. as if it will have any effect on 1 000 Km from Earth.
So on one hand, altitude (and speed) makes a difference in how atomic clocks tick (airliners), and on the other hand, altitude (and speed) makes no difference in how atomic clocks tick (GPS). You accuse me of arguing in circles, yet you are the only one actually doing it.

Quote
All I could see was scientists who wantd to become the famous one to prove the SR and GR with experimentation. Just to get more grants! Dont fall for that man, go and see for yourself on how they manipulated the calculations.
You obviously don't know how science works. If you were to disprove Einstein, with data and evidence, then you are up for a Nobel Prize. You would have the admiration of your scientific peers, and you would have grants coming into you anyway, because you are the fellow who disproved Einstein, and maybe your research can open avenues to the FTL drive, or even more mundane technologies, that tend to make inventors and companies very rich.

Seriously, this "Conspiracy of Dogmatic Science" canard has gotten stale.

Quote
I did explain my experiment to deliver 3 clocks at 2 different positions with a money back guarantee.
And, again, the problem is not synching them up initially, it's keeping them in synch. Notice that in every description of the problem from my end, all the clocks start out at 12:00:00. As soon as you light the rocket, the clocks go out of synch.

Quote
I did explain my experiment to deliver 3 clocks at 2 different positions with a money back guarantee.
And, again, the problem is not synching them up initially, it's keeping them in synch. Notice that in every description of the problem from my end, all the clocks start out at 12:00:00. As soon as you light the rocket, the shipboard clocks go out of synch with the Earth-moon clocks.

Quote
You were the one who claimed that if you travel at the speed of C to the Moon, you will be there instantaneous, with no time that passed for you!!
And I explained why. In the rocket's frame of reference, traveling at speed, the moon and earth are practically on top of each other due to length contraction, so the trip is short to the rocket because they don't travel very far in the first place. The rocket passengers don't see the Earth and moon as one light second apart, but very very much less than that. How much less depends on how fast the ship is going, but it's still not very far.

Quote
Now you are telling me that "It will only look like it to the people on Earth!"
And this is what I have been telling you all the time!
No. That is a mischaracterization of what I've been telling you. You keep insisting that the time elapsed on the rocket is the same amount of time that has elapsed for the Earth-Moon system. This is wrong.

When I say that the rocket experiences practically no time, I really mean that â€" when the guys on the rocket arrive on the moon, they are one second younger than everyone else in the Earth-Moon system by any means available to assess that. The rocket's radiological, mechanical, atomic clocks, biological clocks all agree that the crew is one second younger than the guys on the Moon â€" just like when commercial airliners came back to their home base, their atomic clocks were 50 someodd nanosecond younger than the clocks back home.

You are conflating the real differences in how time is measured in two different frames with an illusion â€" "It only seems like that, but it isn't real." No, it's very real. The shipboard clock measures a real, proper time, just as the Earth and Moon clocks do. It's just that the proper time along those paths are different. And, yes, it's bizarre, but that does turn out to be the kind of universe we live in.

Quote
You are measuring how LIGHT WILL APPEAR TO DIFFERENT TIME FRAMES WHEN TRAVELLING AT HIGH SPEED!
Light will always be observed traveling at c from any reference frame. Michelson and Morley were the first to show this, even though the didn't know what they were looking at at the time. Every experiment we have done confirms this fact. But in order for c to be constant from all reference frames, time and space have to vary, and vary together in a particular way.

Quote
Time will not stop or go faster!
Again, only within the same reference frame. Earth and Moon are in the same reference frame. The rocket isn't once it launches.

Quote
But the difference between us is that I understood what I learned!
No, you don't. You seem to think that the time effects of relativity is an illusion. It is not, but very real. You don't realize that the gravitational effects on clocks in GR is a time dialation as real and as observable as any other effect in physics, and you also don't seem to realize that GR incorporates SR into itself, with all of its timey-wimey wierdness. So, no, you DON'T understand what you "learned."

Quote
It is only the appearance of Time dilation we observe!
...
You just proved me correct and Neil de Grase Tyson et al totally incorrect.
And the Bible true!
No. The time dialation is real. By any means available to them, the guys on the rocket really do experience less time than the guys on the Earth and Moon on their trip. The rocket's clock is behind the Earth-Moon clock precisely because it ticks slower. That's time dialation.

And "Neil de Grase Tyson et al [are] totally incorrect"? Really? You think that you're smarter than all the other physicists in the world? What hubris you have, Mr. Dunning-Kruger.

Quote
It left at Noon on Earth and traveled for one second!
the Moon clock was at noon when the rocket left Earth, and it took the rocket 1 second to get there.
Therefore upon arrival of the rocket on the Moon, both will be one second past noon.
Only from the Earth-Moon system. According to any means available on the ship, the trip took very very much less than one second due to the fact that length contraction (which is also not an illusion) puts the moon very very much less than one light second away from Earth. The moon is also traveling near the speed of light from the rocket's frame of reference. The exact distance and travel time will depend on the exact numbers involved, but traveling one foot at almost the speed of light doesn't take very long, and is by any practical measure instantaneous.

Again, you speak as if everyone will agree on the time and distance that the rocket ship travels. They won't, and they'll all be correct.

Quote
So, now you say you traveled faster than the speed of light!
No. The distances are shorter. To the rocket, the entire universe is compressed along the Earth-Moon axis. All the distances are shorter in their frame. Shorter distances take less time to traverse than longer distances. The bleeding obvious.

Quote
Now you are again contradicting yourself.
No, I'm not. The clocks measure time along their path, and the paths each clock takes is different and cannot be directly compared with each other.

The situation is completely analogous to measuring the length of a speeding train by planting a stake in the ground where its nose is, and walking down its length and planting a stake in the ground where its caboose ends. You're not going to come up with the same measurement as the guy on the train because you have performed your experiment in such a way that time is very much tangled up with the way you measure space. What Einstein found was that space is just as much tangled up with your measurement of time.

<split for length>
Title: Re: Time dilation, length contraction, Relativity and the Bible!
Post by: Hakurei Reimu on August 23, 2018, 11:17:58 AM
<continued>

Quote
what you say is that the clock for some magical reason just stopped to run during its travel. The occupants saw the clock run, but when they came to the Moon it was ctually standing still.
No. The guys on the rocket think that the Earth-Moon clocks were out of synch during their travel and as such aren't good indicators of the amount of time they spent traveling. Every means they have to assess that will reveal that the two are out of synch from the rocket's speeding frame, even though every means the Earth and Moon have to assess whether their clocks are in synch will reveal that they are and always were.

They're both correct, because measurements from one frame aren't commeasurate with measurements from a different frame. You have to relate the two measurement from different frames by the Lorentz transformation.

Quote
What university did you write that exam?
A good one, which teaches that the proper term is "take an exam." You are either lying about you being tested on either SR or GR, or your own "university" was a diploma mill.

Quote
You just claimed that the rocket can travel instantaneous, but the light that traveled from the Earth to the Moon took one second!
Because the two measurements are in different frames, and as such are not comeasurate with each other. They need not and in fact do not equal each other. In fact, along its own world line, the photon took no time either to travel its length, even though it took one second to travel the length in the Earth-Moon system.

Yes, I know what I wrote above. The photon experinces no time from beginning of its trip to its end, whereas the Earth-Moon clocks are experiencing one second for the exact same trip. Both statements are true, because you are measuring two physically different paths. So, yeah, it's not strange that measuring different paths would produce different answers.

Quote
So what is insignificant in your point of view?
Don't pretend that the both of us weren't glossing over the differences between almost-but-not-quite-c travel with actual c travel. You've been calling the rocket's travel c even though you initially acknowledged that it would merely be close to c. Personally, I think a difference of 3 billionths of a second is close enough for our purposes, but we can get as close as would satisfy you.

Quote
You did!
You said that the light that traveled next to the rocket will show the Earth clock time at Noon as it left the earth.
The Moon clock will show one second after noon.
but the occupants on the rocket arrived at one second before noon.
Liar. I kept telling you that nobody sees the rocket take off an instant before noon from any observation. I even specifically pointed out that the rocket arrives at 12:00:01 (Moon time), and that is absolutely NOT "one second before noon" unless you failed kindergarden.

Quote
Now I am astounded.
You flew a rocket at the speed of light over 300 000 Km, and somehow won the race against a light beam travelling at the same distance and you beat him to a full track length!!!!

At what point did you EVER compare the shipboard clock with the clocks on Earth and the Moon, because I cannot find a single reference to you comparing the Moon and Earth clock to the shipboard clock, even though that is exactly where the interesting stuff happens; that this is exactly the point we are aruging over. You instead simply assume that the shipboard clock would remain synchronized instead of working through what SR would actually tell you about the shipboard clock.

Quote
If you travel at C, will time stand still for you?
Strictly speaking, I cannot, as a mass-bearing object, travel at c at all. I can travel arbitrarily close to c, though, to any gamma-factor you desire.

I will not see or observe that my clocks are slowing down.

Everyone else will see and observe my clocks slow down. All of them, includiung my biological processes which make a crude clock. I will count off seconds slower, and even my voice will be slower and deeper due to Doppler shift, but all my clocks will tick in synch with each other.

I will, however, see and observe that the universe is squished along my axis of travel, and a corresponding shortening of all lengths associated with travel. All trips will take shorter time, because I have shorter distance to travel.

Nobody else will see or observe this squishing. I travel across the full distance and take about as much time (only an arbitrarily small amount longer) than an accompanying light beam. I won't notice, of course, because they see and observe my clocks ticking slower.

We will all be right, because they are all in a very real sense different measurements and as such they need not be equal.

Quote
Do you believe this because when you calculate the Lorenz transformation, you are observing where Light will be from reference frame S looking upon reference frame S'?
I observe no slowing because all of my clocks are ticking at the same rate. Of course, if all my clocks are slowed by the same factor, then I won't notice anything. I can only judge the rate of a clock by comparing it against another clock, and my biological processes and my subjective perception of time are slowed just as much as the other clocks. Time dialation affects all clocks equally, by the same factor. I don't need a Lorentz transformation to tell me that.

Anyway, as my actual answer to your question was "No," your screed doesn't matter.

Quote
If so, do you agree that time does not stand still in S', but it seems to do so looking from S?
Not only seems so, by any real measurement, S' clocks were ticking arbitrarily slow ("staying still") in S during the trip, because when the rocket arrives on the moon at 12:00:01, the shipboard clock is a second behind the moon clock even though it would have started out in synch with the Moon clock (because it was in synch with the in-synch Earth clock). Ergo, the shipboard clocks ticked slow. This is not hard.

On the rocket, the trip length to the moon is very short indeed, a meter from Earth to the Moon, depending on how fast the rocket actually travels, and light and the rocket take a few billionths of a second to travel that distance. So the rocket experiences only a few billionths of a second of travel time between Earth to the Moon, practically instantaneously, so of course its shipboard clocks register hardly anything at all, and are still at practically 12:00:00 when they arrive on the moon. Again, this is not hard.

The two reference frames both think that the rocket experiences very little travel time, but for very different reasons: the rocket says that it only had very little distance to go, so of course it didn't take much time; the moon says that it's because the rocket's clocks were ticking slow, so of course it didn't take much time for them, but much longer for the moon and earth. They're both right.

Quote
do you see the error of interpretation on Lorenz?
No. All I see is that you are interpreting the situation wrong. The measurement of proper time elapsed on the moon is a different measurement from the proper time elapsed on the rocket. The paths the clocks take through space matters, and all three clocks are taking very different paths through space. That's why the shipboard clocks falls out of synch with the Earth and moon clocks.

Quote
What you calculate is where the point of light is when looking from S to S'
The Lorentz transformation takes all points in one coordinate frame (S) into another coordinate frame moving with respect to the first (S'). Physics evaluated in one frame only is completely valid. Furthermore, proper time between points is an invariant of the transformation, and it is what clocks measure. Both the rocket and the Earth-Moon system agree that the trip only took an arbitrarily very short amount of time on the rocket, much much less than a second, but they ascribe VERY DIFFERENT reasons for the cause.

Quote
It does not mean that the clock in S' stopped working.
That's right. It's properly measuring the proper time along that path... it's just that not much proper time elapsed along that path, arbitrarily small.

Quote
It only means that you will see the time as light returns from S' to you.
Even correcting for light delay, the time on S' is ticking slow in S. Even when they arrive on the moon, the shipboard clock will register that it is behind the moon clock by about one second, and that will line up with the experience of the passengers.

Quote
just as we will see from earth, the clock on our rocket shows noon when arriving on the Moon, but the real time is one sec past noon.
Yes, because they can look through their telescopes and see that the rocket's clock reads about 12:00:00 and right next to it, the moon's clock reads 12:00:01. But the rocket's clock was 12:00:00 when it left, which was in synch with their own clock at the time. Ergo, the rocket's clock registered hardly any time elapsed on its trip.

Again, because the measurements are of quite different paths.

Quote
You are interpreting observation with reality, which is nothing but observing old light!.
No. Time dialation really happens, even when correcting for old light. "Observing" here means locating each event taking place at its proper coordinates in space and time, as observed in that frame of reference. In SR, observation is not mere sight. It is the integration of all the measurements you have performed and constructing a consistent picture of what happened. When you observe that the rocket arrives on the moon at 12:00:01 moon time, it means that you have already taken into account the fact that the moon is one light second away. What you have seen is that at 12:00:02 Earth time, the rocket has arrived at the moon in your telescope. But that same sighting means that you have observed the rocket arrive at the moon at 12:00:01 Earth-Moon time (because Earth and moon clocks are synched).

I already know that there is light delay, and I am properly taking it into account. Time dialation has nothing to do with light delays. This really happens. So, no, there's no error in SR or GR.
Title: Re: Time dilation, length contraction, Relativity and the Bible!
Post by: Mousetrap on August 24, 2018, 02:59:02 AM
Quote from: Sal1981 on August 23, 2018, 10:37:54 AM
The thing about relativity is that light ALWAYS travels at C in a vacuum, which I reckon where the confusion comes from. The Newtonian of adding speeds together for two reference systems simply doesn't happen for light in a vacuum.

Here's a (non-exhaustive) brief explanation of what Mousetrap simply isn't getting:
https://www.youtube.com/watch?v=9BFKqIoqSIY
I would have liked to look at your video link, but unfortunately I dont see it.
I think it is because I am using South African servers or something.

Anyhow, I do not claim that if a rocket travels at say, 0.5 C, and if one sends a light beam out from its' front, that that light will now travel at 1.5C.
In relation to the movement of the Rocket, the light will now travel at 0.5C
But in relation to the point where the light beam was fired, light travels at 1C.

This is the difference between scientists that claim that light will still travel at 1C in the time frame of the Rocket; and what I see.
This is how RC proponents are making a mistake.

It does not matter at what speed you travel, light will only travel at 1C.
If you travel faster than 1C, time does not change, but you do is to overtake older light!
It will appear as if you are moving back in time, but once you arrive at the point of arrival, light will catch up with you because you are now traveling at 0C.
You still did not travel into history, because your time on the rocket was still syncronised with the point of departure and arrival's clocks.

Title: Re: Time dilation, length contraction, Relativity and the Bible!
Post by: Mousetrap on August 24, 2018, 03:22:36 AM
Quote from: Hakurei Reimu
You were the one who claimed that if you travel at the speed of C to the Moon, you will be there instantaneous, with no time that passed for you!!
And I explained why. In the rocket's frame of reference, traveling at speed, the moon and earth are practically on top of each other due to length contraction, so the trip is short to the rocket because they don't travel very far in the first place. The rocket passengers don't see the Earth and moon as one light second apart, but very very much less than that. How much less depends on how fast the ship is going, but it's still not very far.
HR, I think your posts are too long, and think we should attempt to stay on specific topics per post.
The point you are making in the above is why I say you are interpreting the Lorenz transformation incorrect.

You need to make length to contract, and Time to stand still in order to manipulate the speed of light to remain at 'C' even if you travel at 2 C.

Let me explain.

If we travel to the Moon( at distance C), at velocity 1 C, It will take one second.
If we are on the rocket we will see 2 things.
1. Looking at the Light coming from the Earth(our special clock) it will seem to run zero the speed of C. (Because we are traveling with the light as we left Earth.
2. But, looking towards the moon, that light will seem to run 2XC. Because when we left Earth, the light arriving from the Moon was already one second old, and the clock's light was minus 1 second. As you travel towards the moon, it will take one second to get there, and seeing that you left at noon, and will arrive at noon +1 sec, but the clock on the moon showed -1 second originally and upon arrival was at +1 Noon, You will observe the clock on the Moon running twice as fast as you approach.

Therefore, the EARTH clock SEEMED TO HAVE STOPPED, THE MOON CLOCK SEEMD TO HAVE RUN TWICE AS FAST, AND THE ROCKET CLOCK WAS RUNNING NORMAL SPEED.

This is from the time frame of the Rocket.
From the time frame of the Earth, the rocket clock stood still, the Moon clock ran normal, but was -1 sec, and the Earth clock was running normal.

Do you see that the rocket's clock did not stand still, the Moon and Earth did not move towards each other?
It is only the observation OF LIGHT from the time frame of Earth about the LIGHT FROM THE TIME FRAME OF THE ROCKET AND MOON you are observing.

Sorry pal, time and distance does not change when you travel at C or higher.

Title: Re: Time dilation, length contraction, Relativity and the Bible!
Post by: Sal1981 on August 24, 2018, 08:07:03 AM
Well, at least I tried.
Title: Re: Time dilation, length contraction, Relativity and the Bible!
Post by: Hakurei Reimu on August 24, 2018, 08:21:15 AM
Quote from: Mousetrap on August 24, 2018, 03:22:36 AM
HR, I think your posts are too long, and think we should attempt to stay on specific topics per post.
The point you are making in the above is why I say you are interpreting the Lorenz transformation incorrect.

You need to make length to contract, and Time to stand still in order to manipulate the speed of light to remain at 'C' even if you travel at 2 C.
It is contracting, in its proper frame of reference, S'.

Quote
Let me explain.

If we travel to the Moon( at distance C), at velocity 1 C, It will take one second.
Correct, from S, where the length of the Earth-Moon distance is indeed 1 light second.

Quote
If we are on the rocket we will see 2 things.
1. Looking at the Light coming from the Earth(our special clock) it will seem to run zero the speed of C. (Because we are traveling with the light as we left Earth.
A light beam going at 0c? Incorrect. The light still travels at c. If you take the Lorentz transformation of the coordinates of a light beam in one frame of reference, which travels at c, then it will still be traveling at c in any other frame of reference. That's simply a fact of the transformation.

Quote
2. But, looking towards the moon, that light will seem to run 2XC. Because when we left Earth, the light arriving from the Moon was already one second old, and the clock's light was minus 1 second. As you travel towards the moon, it will take one second to get there, and seeing that you left at noon, and will arrive at noon +1 sec, but the clock on the moon showed -1 second originally and upon arrival was at +1 Noon, You will observe the clock on the Moon running twice as fast as you approach.
Again, incorrect. The light will travel at c in the rocket's reference frame. It is traveling at 1c in S, and it is traveling at 1c in S'. Again, this is a basic fact of the Lorentz transformation: A particle traveling at c in one frame of reference is traveling at c in any other frame of reference.

Quote
Therefore, the EARTH clock SEEMED TO HAVE STOPPED, THE MOON CLOCK SEEMD TO HAVE RUN TWICE AS FAST, AND THE ROCKET CLOCK WAS RUNNING NORMAL SPEED.
While the rocket sees something like this, yours cannot be the explanation, because the rocket will measure every beam of light traveling at exactly c. Not 0c for the receeding Earth and not 2c for the approaching moon. Both are going at c. They can use their own rocket as a ruler and their own clocks to judge how much time it takes a light beam to travel the length of their own ship, or any other aparatus they have to make this measurement, and it will come up with the same answer, c.

Because the light from the earth catches up to them just fine, they know that yours cannot be the explanation the fact that they see the clock on earth running slow. Furthermore, at the speed the Moon seems to be approaching (near c), to get out as far ahead as it did, that light beam had to be traveling a long while, much longer than 1 second, and so the apparant speed of the clock on the moon must be an artifact of old light.

Now, let's specify how fast our ship travels.

Let's set c = 300 000 000 m/s, and the rocket's velocity 1 m/s less than this, v = 299 999 999 m/s. It takes the rocket 1.000 000 003 s to transverse the 1 light-second distance from the Earth to the Moon, in refernce frame S. The proper time along this path is 81.6 microseconds in S. At v, this gives us a distance traveled of 24 494.9 m for the rocket in reference frame S' (the rocket is stationary in its own frame of reference, but the universe around it is moving at -v in S'). Length contraction of 300 000 000 m at v = 299 999 999 m/s yields the same value, 24 494.9 m.

So, we see that we arrive at a consistent answer. The shipboard clock registers a mere 81.6 µs travel time in whichever frame we consider. In S, it's because the ship's clock slowed down at v = c - 1 m/s; in S', it's because D/γ = 24 494.9 m (D = (1 second)·c = 1 light second), which takes only 81.6 µs to transverse at v. This proper time will only get smaller as the rocket approaches c.

This is what SR gives you. It gives you a proper time for the rocket's journey that is legitimately 81.6 µs regardless of which of S or S' you measure from. This is what the shipboard clock will register as it travels to the moon, and is practically instantaneous. It is also the time everyone agrees the shipboard clock has registered on its journey.

Once more, the rocket experiences very little time because in S', the journey was legitimately short (due to length contraction), and in S the ship's clock was ticking slowly. The rocket never outraces it's own light beams, because they always travel at c in any reference frame. In S, the light from it's launch is 1 meter and a bit ahead of it the moment it arrives on the moon. In S', the light from the rocket's initial launch is 24 494.9 m ahead of it (c·(81.6 µs)) when the rocket arrives at the moon. In only a fevered dream would either scenario constitute the ship arriving head of its own light.

Quote
Do you see that the rocket's clock did not stand still, the Moon and Earth did not move towards each other?
The rocket's clock will not stand still in its own frame, S', but I have proven above that the rocket's clock will not register as much time as the required 1+ second it would take from S.

I find it interesting that you have to violate one of the core principles of relativity to justify your assertion that everything is hunky dory and that time and distance do not change when you travel near c "or higher." You appeal to the Lorentz transformation, but it seems strangely absent from your calculation. So, don't spooge it, prove it. Use the Lorentz transformation to prove that these light beams really do travel at 0c and 2c. I double-dog dare you.
Title: Re: Time dilation, length contraction, Relativity and the Bible!
Post by: Mousetrap on August 24, 2018, 08:25:27 AM
Quote from: Sal1981 on August 24, 2018, 08:07:03 AM
Well, at least I tried.
Which is of coarse to be admired.

I do know exactly what the transformation formula is about, and the SR and GR theory.
However, the more I ask Scientists to explain why they think Time and Length can change at the speed of light, the less they explain and become sentimental about what they learned as students.
What they just do not want to think about is that the Michelson Morley experiment proved that light does not need to travel through aether. They actually proved that Light travels at C at all times.
Even if it is projected from a moving body travelling at huge speeds, it will not just add up and go faster.

What the Lorenz transformation does is to measure the time delay as something moves away from another observation point.
This is the actual scientific observation and conclusion.

Time does not dilate, length does not contract.

It is only an observation of old light, nothing more.
Title: Re: Time dilation, length contraction, Relativity and the Bible!
Post by: Mousetrap on August 24, 2018, 08:29:39 AM
Quote from: HR
If we are on the rocket we will see 2 things.
1. Looking at the Light coming from the Earth(our special clock) it will seem to run zero the speed of C. (Because we are traveling with the light as we left Earth.
A light beam going at 0c? Incorrect. The light still travels at c. If you take the Lorentz transformation of the coordinates of a light beam in one frame of reference, which travels at c, then it will still be traveling at c in any other frame of reference. That's simply a fact of the transformation.

Heck pal,
I never said light travels at 0C!
I said, it will seems as if light travels at 0C.

Let me make it simpler.
If you stand on the Earth, and look at the watch on the rocket as it leaves, the clock will seem to be standing still, because the rocket is departing with the light it had on Earth.
it does not mean that light stopped travelling, but it still needs to get back to us.
Title: Re: Time dilation, length contraction, Relativity and the Bible!
Post by: Mousetrap on August 24, 2018, 08:50:02 AM
Quote from: MT
2. But, looking towards the moon, that light will seem to run 2XC. Because when we left Earth, the light arriving from the Moon was already one second old, and the clock's light was minus 1 second. As you travel towards the moon, it will take one second to get there, and seeing that you left at noon, and will arrive at noon +1 sec, but the clock on the moon showed -1 second originally and upon arrival was at +1 Noon, You will observe the clock on the Moon running twice as fast as you approach.
Quote from: HRAgain, incorrect. The light will travel at c in the rocket's reference frame. It is traveling at 1c in S, and it is traveling at 1c in S'. Again, this is a basic fact of the Lorentz transformation: A particle traveling at c in one frame of reference is traveling at c in any other frame of reference.
Again----Seem to...

But lets see what you are now claiming.
Because the rocket travels at C, the light inside the rocket will travel also at C if measured in the rocket.

Well, this is wrong.
Light will travel at C from its point of origin.
If the rocket travels at C, the light in the rocket will actually standing still.

This is the great distortion of the interpretation of Lorenz.
you will say that light will be measured at C if we place 2 detectors in the rocket.
One at the tail and one at the nose, and if we fire a light beam from the tail, it will reach the nose.

Well, this is not so.
The light fired at the tail of a rocket traveling at C, will stay at the tail!

This is the characteristic of light!

It travels at C which is 300 000 km per hour(about)

Look at this example.

We shoot a beam of light to the moon from our Earth observer.
our rocket had a running start, and exactly as he passed the earth  observer (at C)when he fired the light beam to the moon,
our rocket man fired a light beam to the moon also.
Now, reality has it that the light beam of the rocket, as well as the one from the earth observer will arrive at the same time on the moon.
therefore, the rocket man will see the point of the light beam and the rocket light beam travelling with him, and all 3 arrive at the same instance at the moon.
Therefore, the Rocketman will not be able to measure light at C, because light will slow down in his point of view.
He knows that the light is travelling at C, but from ITS SOURCE!
NOT FROM THE ROCKET'S SPEED ADDED TO C!

Title: Re: Time dilation, length contraction, Relativity and the Bible!
Post by: Mousetrap on August 24, 2018, 08:59:03 AM
And that is what will really be observed.

What Relativists say is,
The man on the Earth, and the rocket man will shoot a light beam at the same time when the rocket passes the starting point.
The man in the rocket will see the light in the rocket travelling at C '''''IN HIS ROCKET RIGHT UP TO THE ROCKETS' NOSE''''', BUT OUTSIDE THIS ROCKET the OBSERVER ON EARTH WILL SEE, THE LIGHT BEAM STAYS AT THE TAIL OF THE ROCKET.

THE ROCKET MAN WILL ALSO SEE THE LIGHT OUTSIDE HIS ROCKET, THAT WAS FIRED FROM THE EARTH, TRAVELLING AT C RUSHING PAST HIM TO THE MOON, BUT ON EARTH THE LIGHT SEEMS TO TRAVEL WITH THE ROCKET.

Do you see the incorrect interpretation of Lorenz?
Title: Re: Time dilation, length contraction, Relativity and the Bible!
Post by: Hakurei Reimu on August 24, 2018, 12:30:46 PM
Quote from: Mousetrap on August 24, 2018, 08:29:39 AM
Heck pal,
I never said light travels at 0C!
I said, it will seems as if light travels at 0C.
That is an impossible observation according to special relativity; if their measuring stick is one light nanosecond (c·(1 ns)) long, it will take exactly one nanosecond for any light they put it against to travel along it. The light will not even seem to go 0c. They will see it go the full 1c.

Furthermore, yours is exactly the kind of scenario the Michelson-Morley experiment was designed to detect in real life, but did not. And they were quite thorough. Their aparatus was quite sensitive enough to detect it were it present. They tested it in all sorts of conditions, at various times during the day, and year. If there was any "aether wind" (which is what your notion is compeletely congruent to), they would have detected it, but they never did.

Quote
Let me make it simpler.
No. Let's not. You've already dumbed down your example past the point where it's too simple even for you. I understand what you're saying, it's just plain WRONG.

Anyway, gotta run, so I'll respond to your other posts later.
Title: Re: Time dilation, length contraction, Relativity and the Bible!
Post by: Unbeliever on August 24, 2018, 01:16:55 PM
According to Einstein, "Everything should be made as simple as possible, but not simpler."

Making things too simple means leaving out important bits.
Title: Re: Time dilation, length contraction, Relativity and the Bible!
Post by: Hakurei Reimu on August 24, 2018, 11:05:50 PM
As promised, the continuation.

Quote from: Mousetrap on August 24, 2018, 08:50:02 AM
Well, this is wrong.
Light will travel at C from its point of origin.
If the rocket travels at C, the light in the rocket will actually standing still.
No, it will not. Light is made of electromagnetic waves. Light "standing still" will result in a static electromagnetic wave in a vacuum, which in Maxwell's equations is an absurd result, and cannot be derived using Maxwell's equations. This is because magnetic fields in a vacuum are generated by changing electric fields, and vice versa. The way light propagages means that, if it were possible to move with the light wave, then this is exactly what we'd see â€" a static, unchanging magnetic and electric wave with no source in a vacuum. And again, there's no allowance in Maxwell's equations for the speed of the observer; carrying through with the equations yields a result that demands an electromagnetic wave propagating at c, even when moving at c yourself.

Physicists in the nineteenth century knew this. Yes, it was mind-blowing, but it is also what Maxwell's equations were telling them, and those equations were proving their worth otherwise. They knew that a static electromagnetic wave is not permitted by Maxwell's equations; if it were, Einstein would never needed to come up with special (and general) relativity in the first place! Physicists were coming up with ways around this precisely because they knew what you described above is not possible.

Nobody has ever observed a light wave "standing still." Nobody has ever made any observation that supports that proposition, either. Again, if it did, the Michelson-Morley experiment would have yielded a positive result in detecting the aether wind. The aether wind would be effectively light being slowed down a bit by motion. But no, there was no detection even with thorough testing over a wide range of velocities.

You have not given me any reason to believe that you are correct, while on my side I have plenty of reason to believe that you are incorrect. Yours are just assertions. You simply assert that a light wave can be stationary from some observer, even though currently physics has firmly established that it isn't true, and as such only a physical experiment could knock that principle over.

TL;DR â€" the existence of special relativity is a result of physics knowing, even in the nineteenth century, that a stationary electromagnetic wave is an absurdity. Your proposition wouldn't even survive physics pre-Einstein.

Quote
This is the great distortion of the interpretation of Lorenz.
You keep asserting that, but I have yet to see you use it even once. How could you know what I say is "distorted" if you don't even bother finding out what it says?

I don't think you even know what the Lorentz transformation is, or what it implies. Here, let me demonstrate that in all frames of reference, where v < c, the speed of a light beam will always be observed at going c, using the Lorentz transformation that I supposedly don't understand.

The Lorentz transformation is for some velocity v in the x+ direction:

t' = γ(t - xv/c²)
x' = γ(x - vt)
y' = y
z' = z

where γ = 1/sqrt(1-v²/c²)

Let's ignore the y and z axis. Similar algebra will reveal that other direction will yield a similar result.

In frame S, a photon begins at P0 = (x0, t0) and after some time, T, arrives at P1 = (x0 + cT, t0 + T). The difference between the two points is Î"P = (Î"x, Î"t) = (x0 + cT - x0, t0 + T - t0) = (cT,T). Pretty straightforward.

The velocity of the photon in S is u = cT/T = c.

In frame S' (moving at v in S), the two points are P'0 = (x'0, t'0), and P'1 = (x'1, t'1), and Î"P' = (x'1 - x'1, t'1 - t'0). Using the Lorentz transformation on these two points and a little algebra reveals that Î"P' = (γ(Î"x-vÎ"t), γ(Î"t-vÎ"x/c²).

The velocity of the photon in S' is u' = γ(Î"x-vÎ"t)/γ(Î"t-vÎ"x/c²) = (cT-vT)/(T-vcT/c²) = (c-v)/(1-v/c) = c·(1-v/c)/(1-v/c) = c.

Oh, look at that! As I asserted, a particle that has velocity c in one frame of reference will have velocity c in another frame of reference. Of course, if v = c, the Lorentz transformation blows up on you (it divides by zero), so there can be no Lorentz transformation into a frame going at c in any other frame.

So, you are catagorically NOT using a Lorentz transformation. Vectors traveling at c are eigenvalues of this transformation â€" they remain unchanged up to being stretched or squished; ergo, if any particle traveling at c in one frame do not travel at c in another frame, you have not used a Lorentz transformation.

Which is basically what's been happening.

Quote
The light fired at the tail of a rocket traveling at C, will stay at the tail!
Mere repetition of the same faulty reasoning.

The rocket cannot travel at c. I've been generous up to this point in allowing this sloppy reasoning, but I'm putting my foot down here. The rocket has mass. Only massless particles can travel at c. Ergo, the rocket is traveling strictly less than c, even though it may be traveling arbitrarily close to c. Therefore, in any physically possible scenario the light will eventually reach the nose, even by your ignorant argument.

We have never observed a mass-bearing particle traveling at c. Everything we have observed about particles with mass reveals that they cannot travel at c. Furthemore, only mass-bearing particles can travel slower than c. Massless particles are never observed traveling at any speed other than c. Because rockets occasionally travel slower than c (like standing still on a rocket pad), they must be mass-bearing and therefore speeds of c are not possible for them.

The above is a summary of what is known about c and particles that can and cannot travel at c. The characteristic that makes this difference is mass. The rocket, which has mass, cannot travel at c.

Again, the Lorentz transformation preserves vectors traveling at c. Furthermore, there are no Lorentz transformations taking a frame into one traveling at c.

Quote
This is the characteristic of light!
It travels at C which is 300 000 km per hour(about)
Yes, it does, in all reference frames. This includes the rocket, so it must also see light go at c. QED.

Indeed, not only is it the characteristic of light, but it also is the maximum speed that any interaction can occur. It's specialness goes beyond merely being the speed of propagation of the waves of a particular field â€" it's one of the big damn important constants.

Quote
Now, reality has it that the light beam of the rocket, as well as the one from the earth observer will arrive at the same time on the moon.
therefore, the rocket man will see the point of the light beam and the rocket light beam travelling with him, and all 3 arrive at the same instance at the moon.
You talk about reality, yet immediately speak of an unreal scenario.

Again, I refer you to Maxwell's equations which asserts that static electromagnetic waves cannot exist, and therefore a electromagnetic wave seen to be standing stationary is not possible. Even by 19th century standards of physics, your assertion would be false, because if it were true, there would be no special relativity for you to argue against.

Quote
Therefore, the Rocketman will not be able to measure light at C, because light will slow down in his point of view.
He knows that the light is travelling at C, but from ITS SOURCE!
NOT FROM THE ROCKET'S SPEED ADDED TO C!
The rocketman will observe that the light going at c, even in his frame. It will take the light about one nanosecond by his clock to traverse his wooden 12" ruler. He may think that he is measuring v + c, but any experimenter outside the rocket and seeing the rocket move at v will also observe the light moving at c, by his own rulers and his own clock. I know this is wierd, but it is what special relativity asserts, and what happens in real life by any experiment we care to perform.

Quote from: Mousetrap on August 24, 2018, 08:59:03 AM
What Relativists say is,
<snip>
THE ROCKET MAN WILL ALSO SEE THE LIGHT OUTSIDE HIS ROCKET, THAT WAS FIRED FROM THE EARTH, TRAVELLING AT C RUSHING PAST HIM TO THE MOON, BUT ON EARTH THE LIGHT SEEMS TO TRAVEL WITH THE ROCKET.
More repetition of the same old crap. No, it won't. No "Relativist" will say this. They're called physicists, by the way, and they have forgotten more physics than you will ever know. They are very clear on this, and I have used the Lorentz transformation to demonstrate this: there is no frame of reference that will ever observe light going at any speed other than c.

The rocket cannot travel at c. Ergo, your scenario is impossible. Period. It must travel at less than c, which means that even by your logic, the light beam does not stay at the tail of the rocket because the light beam is still faster, even if only marginally so.

Quote
Do you see the incorrect interpretation of Lorenz?
I have seen lots of assertions from you that I have the incorrect interpretation of the Lorentz transformation, but I have yet to see you demonstrate it, just like in your other threads. Above, you will find a demonstration that light traveling at c from one frame will also be traveling at c in another. So tell me again how I have the "incorrect interpretation of Lorentz."

Better yet, demonstrate this, instead of simply spewing about it. Please have this demonstration in your next rounds of posts, or I win.

Your assertions are supported by no experiment or observation, and there is plenty of experiment and observation to the contrary. If a hypothesis doesn't agree with experiment, it's WRONG. Special relativity, and the constancy of the speed of light in all frames, by all means of measurement, is the foundation of all modern physics and is not going to be dismissed by some crank on the internet with a messiah complex.
Title: Re: Time dilation, length contraction, Relativity and the Bible!
Post by: trdsf on August 25, 2018, 10:08:23 AM
About thirty years ago, CalTech did two remarkable series, The Mechanical Universe and The Mechanical Universe and Beyond.  The second series has three episodes that explain Einstein, time dilation, length contraction, and the apparent increase in mass of moving objects in as clear and cogent a manner as I've ever seen, and I have read and calculated a lot on the matter.  Spoiler alert for our little church mouse, there is no reference to any of it coming from any bible.

So, here are the relevant episodes, on: The Lorentz Transformation
https://www.youtube.com/watch?v=feBT0Anpg4A

Velocity and Time
https://www.youtube.com/watch?v=BFLUa0ciMjw

and Energy, Momentum and Mass
https://www.youtube.com/watch?v=lZUrLq0LLIU

For the interested viewer, the entire 52-episode series (and others) is here (https://www.youtube.com/playlist?list=PL8_xPU5epJddl1dmAZWlERA0zplgD0W4E).
Title: Re: Time dilation, length contraction, Relativity and the Bible!
Post by: Jason78 on August 25, 2018, 07:54:32 PM
Quote from: Hakurei Reimu on August 24, 2018, 11:05:50 PM
As promised, the continuation.

Damn I love a good smackdown! 
Title: Re: Time dilation, length contraction, Relativity and the Bible!
Post by: Baruch on August 26, 2018, 03:27:20 AM
Shooting mice in a barrel ... how hard is that?
Title: Re: Time dilation, length contraction, Relativity and the Bible!
Post by: Hakurei Reimu on August 26, 2018, 10:26:17 PM
Teaching mice in a barrel seems harder.

I think it's pretty clear that our little MT doesn't know what a frame of reference is and why they're so important, and seems very much to think that a Lorentz transformation produces an illusionary world where things only seem distorted and wierd, but actually things are all hunky dory.

So let's tell a tale of physics.

In the four hundred or so years after Newton, physics has derived many many laws. Laws of mechanics, laws of chemistry, laws of hydrodynamics, etc. All of them shared one particular thing in common: as far as could be determined, the laws of nature did not care about the absolute position of any object, how they were oriented absolutely, what absolute time an experiment was performed in, or (most importantly for our story) how fast the whole experiment is traveling through space. At best, only relative distances, orientations, times, and motions mattered. You could always expand the scope of an experiment to the point where you end up with a system that could be as a unit freely relocated, reoriented, advanced and retarded in time, and given a boost in any given direction and magnitude, and the experiment would operate in the same way.

Translations in time and space do not cause the physical laws to work differently. Rotations in space do not cause the physical laws to work differently. Using a different system of measurement does not cause the physical laws to work differently. When a transformation does not change how the physical laws work, it is said that the laws of physics are invariant under the transformation.

The Galilean transformation (https://en.wikipedia.org/wiki/Galilean_transformation) applies a boost to a physical system or experiment, making it move in a uniform motion in a particular direction. The physics known in the 19th century did not work differently between two states of affairs related by a Galilean transformation, with one singular exception (more on that later).

Each state of affairs, that could be related to each other by any of the above means of transformation, is a frame of reference. If S and S' are frames of references related by some transformation under which the laws are invariant, you can transform all of S into S' and all the physics we deduced for S will still work in S' without referring to S at all, and the results of applying those physical laws to S' will translate back to S as if you had worked in S all along. That's a very powerful concept and it is what makes frames of reference so important in physics instead of the mere mathematical curiosities that MT seems to think they are. If an object or entity in a particular frame of reference is measured to be stationary in that reference frame, then it can be treated in that frame as exactly that, regardless of what its state of motion in any other reference frame. In this sense, a reference frame is entire in and of itself, and all reference frames are equivalent to each other in the eyes of physics. Thus, since Newton, evidence had been mounting for the case of the principle of relativity, as these invariances of the laws of physics came to be known.

But with the Galilean transformation, you can have a situation where you can observe a light wave traveling at zero velocity relative to you by choosing an appropriate boost. By the principle of relativity, you should be able to treat this situation as if there really is a light wave not moving in space and all of the laws of physics will work just as well on it as it does according to an observer that observes that light propagates at the usual c, but it doesn't. Maxwell's laws don't allow for a light wave to propagate at any speed other than c, and without an aether wind to take up the burden, the Galilean transformation leads to an unphysical notion: a stationary, frozen electromagnetic wave in a vacuum â€" something that Maxwell's equations does not allow to exist. If you had to remember that the light is "actually" traveling at c, even though the frame doesn't measure it going at c, then it destroys the principle of relativity.

That's why when Albert Michelson and Edward Morley failed to find the aether wind, it was a big deal. It appeared that Maxwell's equations and light were the sole exception to the principle of relativity with respect to boosts in velocity. When James Clerk Maxwell first found that his equations would produce a constant speed of light, he and others were confident that it was merely because that constant was with respect to the aether in which light propagated, like the way a sound wave propagates through air. Had aether been found, the experiment would have vindicated the Galilean transformation as the proper transform for relating between reference frames of different boosts, but it didn't, leaving physics with a quandary.

Now we know that it is the Lorentz transformation that is the proper transform between reference frames related by a boost, and the Galilean transformation is merely the low-velocity approximation. The principle of relativity allows us to with confidence state that we fairly well know the physical laws we have derived on Earth, because our true spatial and temporal coordinates, our true orientation within the universe, and our true velocity through that universe are all irrelevant for deriving the physical laws, if those attributes even have an ontological meaning in the first place.

If there are any lengths and times that are God's length and time, it is proper length and proper time; duration and distance as deduced through coordinate time and space are all things of man. MT is trying to assert that the time experienced by a near-c rocket is one second based upon an imaginary line drawn between the rocket and his stationary clock over the shipboard clock that is actually measuring time along the rocket's path. While that imaginary line, constructed by man, may have some usefulness, it's laughable to think that it reveals a truth superior than the reality of proper time (God's time). Quite bizarre.
Title: Re: Time dilation, length contraction, Relativity and the Bible!
Post by: Mousetrap on August 27, 2018, 02:51:25 AM
Quote from: Hakurei Reimu on August 24, 2018, 12:30:46 PM
That is an impossible observation according to special relativity; if their measuring stick is one light nanosecond (c·(1 ns)) long, it will take exactly one nanosecond for any light they put it against to travel along it. The light will not even seem to go 0c. They will see it go the full 1c.

Furthermore, yours is exactly the kind of scenario the Michelson-Morley experiment was designed to detect in real life, but did not. And they were quite thorough. Their aparatus was quite sensitive enough to detect it were it present. They tested it in all sorts of conditions, at various times during the day, and year. If there was any "aether wind" (which is what your notion is compeletely congruent to), they would have detected it, but they never did.
No. Let's not. You've already dumbed down your example past the point where it's too simple even for you. I understand what you're saying, it's just plain WRONG.

Anyway, gotta run, so I'll respond to your other posts later.
Well, I am of the opinion that you are so brainwashed about the incorrect characteristics of light which you were forced to believe, that you obviously ascribe things to the Mickelson Morley experiment that are incorrect.

Lets see, they took a beam of light entering from a specific direction, split the beam on a reflective mirror to have a 90 degree detour on one half, and a 180 degree detour on the other.
They then checked to see if the light beams showed distortion when combined again.
Now, this will prove that light travelling in 2 different directions are influenced by anything such as "AETHER WIND" that might be travelling from one direction, and would have slowed down one of the beams resulting in a wave distortion, and evidence of Aether Winds!

Cool, they proved there was no Aether winds, and light travels at C, no matter which direction.

Now, please give me the experiments where it showed that light will travel at C in whatever reference frame it is observed!


Mickelson Morley's experiment does nothing of this sort.

Look at this.
Goddard have sent lasers to satellites, one orbiting the moon, over a distance of 240 000 miles, as the satellite travels at over 3 200 miles an hour.
In this instance they must allow and calculate where the satellite will exactly be.
The satellite must be fired in advance of the position to the satellite, and as the laser travels to the satellite, the satellite will actually move in front of the laser.

Whats more, the reflected photons will now travel back to the earth, but the light beam must be positioned from the satellite to travel to a point on the earth again.

Great, now what does this mean?
Light does not travel in straight lines, but in a 3 D spherical shape.
One piece of the sphere's surface can not travel faster than any other point of the spherical surface, but is dependent upon C from ITS' POINT OF ORIGIN!

THEREFORE, THE MICKELSON MORLEY EXPERIMENT ACTUALLY ONLY PROVED THAT THERE IS NO AETHER, AN THAT THE SOURCE OF THE LIGHT BEAM THAT WAS SPLIT IN 2, WAS AT THE MIROR.

AND THEY PROVED THAT LIGHT WILL TRAVEL AT THE SAME SPEED FROM IT'S SOURCE, NO MATTER IN WHICH DIRECTION YOU SEND IT.


Title: Re: Time dilation, length contraction, Relativity and the Bible!
Post by: Mousetrap on August 27, 2018, 03:12:38 AM
let me continue with the characteristics of light.

Light travels at C in a vacuum, FROM ITS SOURCE.
it travels on a 360 degree SPHERE.
if a light is flashed when a satellite travels at 7 000 Km per hour, the light will travel at C, FROM THE POINT OF ORIGIN.
actually, the satellite will travel away from the point of origin of the flash of light, and will experience the Doppler effect and will observe the light as red-shift.

If the satellite would ever be able to achieve C+ speeds, the satellite will travel outside of the Light sphere, and will not see the flash, untill the satellite slows down, and allows the light to gain.

Now, the exact same will happen when a flash of light was set off on the satellite.

The light will travel at C from it's point of origin.
If there was 2 flashes of light, one on the satellite, and one on another satellite that passed the first from an opposite direction, at the same point (perhaps a few meters from each other for argument) both satellites will see the light at red-shift.

Now, this is the nice thing about Physics and Light.

With the use of Light, one will be able to determine which satellite is travelling, and which is stationary.
Flash both lights on the satellites at the same time they pass, and read the red shift, or Blue shift.
If one satellite was stationary, It will measure accordingly, and if the other was travelling at 0.5 C, it will measure both lights at Red shift.

This is exactly as we observe far off galaxies, where they are travelling away in relation to us.
Their light is measured in our frame of reference as red shifted, and our light at normal C.

Therefore, Newton was correct, Einstein not.


Title: Re: Time dilation, length contraction, Relativity and the Bible!
Post by: Mousetrap on August 27, 2018, 05:03:45 AM
Quote from: HRBut with the Galilean transformation, you can have a situation where you can observe a light wave traveling at zero velocity relative to you by choosing an appropriate boost. By the principle of relativity, you should be able to treat this situation as if there really is a light wave not moving in space and all of the laws of physics will work just as well on it as it does according to an observer that observes that light propagates at the usual c, but it doesn't. Maxwell's laws don't allow for a light wave to propagate at any speed other than c, and without an aether wind to take up the burden, the Galilean transformation leads to an unphysical notion: a stationary, frozen electromagnetic wave in a vacuum â€" something that Maxwell's equations does not allow to exist. If you had to remember that the light is "actually" traveling at c, even though the frame doesn't measure it going at c, then it destroys the principle of relativity.

I had to find one discussion point in an attempt to show you what I mean.
Here we have the problem with SR and GR in a nut shell.
1. the Galilean transformation adds velocities up.
2. the Lorentz transformation shows how something behaves once it reaches huge speeds that can be measured in fractions of C.
And to which I agree.

But it actually shows you how light will appear to look from another time frame, in the observed time frame.
And it does not mean that Time and Length changed at all, but your interpretation of what you would observe about light is what you incorrectly now think is the error.

Never did I say that Light can be stopped.
Light can never be stationary.
But travelling at C, It will appear to be just that!

You see, this is what Lorenz and Einstein missed out on.
They presumed that it is impossible for us to determine if light, just as stationary objects, is moving or not.

If this is the case, then SR and GR is 100% correct.

However, the total contradiction about this characteristic of light is evident once we look at all the different claims made by physicists.

For instance, even Stephen Hawking in his publications claims that light travels at C, and if you are travelling at C, and is not in the 3 dimensional sphere of light. (from its source obviously)[added by me], you will ever see that light!

Therefore, I conclude that Light has it's source, and Must travel from A to B, A being the origin, and B being our position.

Sorry pal, you can not claim that light will have 2 measurable speeds, one where you travel at say 0.5C, and one stationary. and both will measure C.

 

I know the Lorentz transformation by heart, and do not need you to give me the formulas.
I need just one thing from you.

What, in your mind, does the Lorentz transformation measure.

Do it simple like,
take a vehicle that travels to the left at XV.
Shoot a laser forward.
shoot a laser backward.
now add both together.

Then explain why.
and and
I need to see where and what you see, that I dont.
Title: Re: Time dilation, length contraction, Relativity and the Bible!
Post by: Cavebear on August 27, 2018, 05:06:32 AM
Quote from: Mousetrap on August 27, 2018, 05:03:45 AM
I had to find one discussion point in an attempt to show you what I mean.
Here we have the problem with SR and GR in a nut shell.
1. the Galilean transformation adds velocities up.
2. the Lorentz transformation shows how something behaves once it reaches huge speeds that can be measured in fractions of C.
And to which I agree.

But it actually shows you how light will appear to look from another time frame, in the observed time frame.
And it does not mean that Time and Length changed at all, but your interpretation of what you would observe about light is what you incorrectly now think is the error.

Never did I say that Light can be stopped.
Light can never be stationary.
But travelling at C, It will appear to be just that!

You see, this is what Lorenz and Einstein missed out on.
They presumed that it is impossible for us to determine if light, just as stationary objects, is moving or not.

If this is the case, then SR and GR is 100% correct.

However, the total contradiction about this characteristic of light is evident once we look at all the different claims made by physicists.

For instance, even Stephen Hawking in his publications claims that light travels at C, and if you are travelling at C, and is not in the 3 dimensional sphere of light. (from its source obviously)[added by me], you will ever see that light!

Therefore, I conclude that Light has it's source, and Must travel from A to B, A being the origin, and B being our position.

Sorry pal, you can not claim that light will have 2 measurable speeds, one where you travel at say 0.5C, and one stationary. and both will measure C.

 

I know the Lorentz transformation by heart, and do not need you to give me the formulas.
I need just one thing from you.

Blind faith?
Title: Re: Time dilation, length contraction, Relativity and the Bible!
Post by: Jason78 on August 27, 2018, 05:12:03 AM
Quote from: Mousetrap on August 27, 2018, 03:12:38 AM
Therefore, Newton was correct, Einstein not.

You do realise that some of our modern technology relies on Einstein's equations to work don't you?   

Your little fantasy doesn't change that fact one bit.
Title: Re: Time dilation, length contraction, Relativity and the Bible!
Post by: Cavebear on August 27, 2018, 05:26:19 AM
Quote from: Mousetrap on August 27, 2018, 05:03:45 AM

Do it simple like,
take a vehicle that travels to the left at XV.
Shoot a laser forward.
shoot a laser backward.
now add both together.

Then explain why.
and and
I need to see where and what you see, that I dont.

1.  Lightspeed can be slowed (https://en.wikipedia.org/wiki/Slow_light)
2.  Light in a vacuum sent opposite directions affects time, not light speed. 
Title: Re: Time dilation, length contraction, Relativity and the Bible!
Post by: Mousetrap on August 27, 2018, 05:47:34 AM
QuoteA brief history of time
(https://i.imgur.com/OUXo5gy.png)
I think this image will clarify everything that light has an origin, and travels at a certain speed.
Therefore, to say that if one travels in the same direction of light, light will still be measured at C from your position, is incorrect.
Frankly, just as gravity is a fact, so it is that light has an original start off point, and any time frame must be in reference to that point of origin.

Where is the way where light dwells (travels)? and as for darkness, where is its place (stationary)?  Job 38:19
Title: Re: Time dilation, length contraction, Relativity and the Bible!
Post by: Cavebear on August 27, 2018, 06:38:26 AM
Quote from: Mousetrap on August 27, 2018, 05:47:34 AM
(https://i.imgur.com/OUXo5gy.png)
I think this image will clarify everything that light has an origin, and travels at a certain speed.
Therefore, to say that if one travels in the same direction of light, light will still be measured at C from your position, is incorrect.
Frankly, just as gravity is a fact, so it is that light has an original start off point, and any time frame must be in reference to that point of origin.

Where is the way where light dwells (travels)? and as for darkness, where is its place (stationary)?  Job 38:19

Lightspeed does not equal lightspeed plus forward or backwards motion. 

I guess you are trying to explain how light can come from 13 billion years away when your assumption is that the universe is only a few thousand years old.
Title: Re: Time dilation, length contraction, Relativity and the Bible!
Post by: Mousetrap on August 27, 2018, 07:03:39 AM
Quote from: Cavebear on August 27, 2018, 06:38:26 AM
Lightspeed does not equal lightspeed plus forward or backwards motion. 

I guess you are trying to explain how light can come from 13 billion years away when your assumption is that the universe is only a few thousand years old.
On the contrary!
This is what guys such as Ken Ham and Kent Hovind tries to sell.

I understand from the Bible that light travels at C.
I understand that distance is measured by triangulation, and that some galaxies are measurable in billions of light years.
I understand that it should mean that this light traveled for billions of years to reach us.

This is then also evidence that I am correct in claiming that, that light had an origin a few billion light years away.

This means that time and distance is a continuum.
Speed is distance traveled in a set time.

Now, this is what Lorenz tried to do.
He wanted speed to be a continuum, and distance and time to be variable.
Title: Re: Time dilation, length contraction, Relativity and the Bible!
Post by: Cavebear on August 27, 2018, 07:23:41 AM
Quote from: Mousetrap on August 27, 2018, 07:03:39 AM
On the contrary!
This is what guys such as Ken Ham and Kent Hovind tries to sell.

I understand from the Bible that light travels at C.
I understand that distance is measured by triangulation, and that some galaxies are measurable in billions of light years.
I understand that it should mean that this light traveled for billions of years to reach us.

This is then also evidence that I am correct in claiming that, that light had an origin a few billion light years away.

This means that time and distance is a continuum.
Speed is distance traveled in a set time.

Now, this is what Lorenz tried to do.
He wanted speed to be a continuum, and distance and time to be variable.

My apologies.  I am too used to theists "explaining" that the universe appears old because a deity created light in mid-travel, or some such nonsense.

But you never explained about the water source of the flood.  Want to explain that?
Title: Re: Time dilation, length contraction, Relativity and the Bible!
Post by: Unbeliever on August 27, 2018, 02:50:07 PM
Quote from: Mousetrap on August 27, 2018, 03:12:38 AM
Therefore, Newton was correct, Einstein not.

Newton and Einstein were both correct - in the domain in which their theories were intended. At low speeds and lighter gravities, Einstein's solutions match Newton's.
Title: Re: Time dilation, length contraction, Relativity and the Bible!
Post by: Cavebear on August 27, 2018, 02:55:33 PM
Quote from: Unbeliever on August 27, 2018, 02:50:07 PM
Newton and Einstein were both correct - in the domain in which their theories were intended. At low speeds and lighter gravities, Einstein's solutions match Newton's.

Indeed, I think I read read somewhere that Newton had the data, but not the maths, for spacetime.  Still, it is all one shoulder to climb on at a time.  Without Newton, Einstein might have been a very good patent clerk.  Of course, in the opposite sense, if it weren't for those damn Greek geometry mystics, we might be on Alpha Proxima by now.
Title: Re: Time dilation, length contraction, Relativity and the Bible!
Post by: Baruch on August 27, 2018, 08:06:10 PM
Quote from: Cavebear on August 27, 2018, 02:55:33 PM
Indeed, I think I read read somewhere that Newton had the data, but not the maths, for spacetime.  Still, it is all one shoulder to climb on at a time.  Without Newton, Einstein might have been a very good patent clerk.  Of course, in the opposite sense, if it weren't for those damn Greek geometry mystics, we might be on Alpha Proxima by now.

You will never master the triangular or square number theory of Pythagoras!

Actually Einstein didn't do space-time, his teacher, Minkowski did that.  Then Einstein was smart enough to realize it was necessary, so he could extend Relativity to gravity (acceleration).  But it was a re-invention ... distance vs time diagrams go back to Galileo (who was the first to experiment quantitatively with motion).
Title: Re: Time dilation, length contraction, Relativity and the Bible!
Post by: trdsf on August 27, 2018, 10:29:04 PM
Quote from: Cavebear on August 27, 2018, 02:55:33 PM
Indeed, I think I read read somewhere that Newton had the data, but not the maths, for spacetime.  Still, it is all one shoulder to climb on at a time.  Without Newton, Einstein might have been a very good patent clerk.  Of course, in the opposite sense, if it weren't for those damn Greek geometry mystics, we might be on Alpha Proxima by now.
Certainly Newton was aware of the problem of universal collapse as predicted by his theory of gravity; he said it wouldn't happen if you had infinite stars scattered evenly over infinite space since there wouldn't be a central point to collapse to.

Newton was, of course, completely wrong.  There is no way to have a stable collection of massive objects in which gravity is always attractive that does not ultimately collapse.  What defeated Newton wasn't his lack of math, it was his dual assumptions that the universe must be static and that gravitational force is transmitted instantaneously.
Title: Re: Time dilation, length contraction, Relativity and the Bible!
Post by: Hakurei Reimu on August 27, 2018, 11:44:55 PM
Quote from: Mousetrap on August 27, 2018, 02:51:25 AM
Now, this will prove that light travelling in 2 different directions are influenced by anything such as "AETHER WIND" that might be travelling from one direction, and would have slowed down one of the beams resulting in a wave distortion, and evidence of Aether Winds!

Cool, they proved there was no Aether winds, and light travels at C, no matter which direction.

Now, please give me the experiments where it showed that light will travel at C in whatever reference frame it is observed!
The experiments that showed that Maxwell and Lorentz's electrodynamics is the correct theory of electrodynamics also establish the independence of c from the speed of the source. Thus, no matter what the source is doing, a shell of light propagating at c from a source would be centered upon the point in that frame of reference the shell was emitted.

Also, synchrotron radiation, produced from accelerating relativistic particles, do not travel at any speed measureably different from c in a vacuum. If the speed of the emitted light depended upon the speed of the source, here is where it would show up most profoundly and obviously. It does not. Synchrotron radiation is used every day to probe objects, and you kind of need to know when you should expect a scatter from your radiation to arrive at your detector so you can minimize noise. Discrepencies, especially as large as 2c, would be noted.

Here's the thing. Michelson and Morley knew the Earth was moving, and furthermore has a wide veriety of movement through space. This has been a long established fact in cosmology for hundreds of years by the 19th century. Yet the Michelson-Morley experiment established that, dispite this known motion of the Earth, the aparatus was behaving as if it were completely stationary.

Quote
Mickelson Morley's experiment does nothing of this sort.
That's because you don't know your physics. That's been obvous to us for about a month now.

Quote
Whats more, the reflected photons will now travel back to the earth, but the light beam must be positioned from the satellite to travel to a point on the earth again.

Great, now what does this mean?
It's basically a replication of the stellar abaration experiments that conclude that there was no aether dragging. The laser is behaving exactly as advertised.

Quote
Light does not travel in straight lines, but in a 3 D spherical shape.
Exactly wrong. If the light was traveling as a 3D spherical shape, why would they need to aim the laser at all?

Something's wrong with your logics, boy.

Quote
One piece of the sphere's surface can not travel faster than any other point of the spherical surface, but is dependent upon C from ITS' POINT OF ORIGIN!
How are you proving Einstien wrong here? Of course it does. If you emit a shell of light, that light will move out at c centered about where the light was emitted in your frame of reference. The thing is, that happens whatever frame you're considering. If the source is moving in your frame, then the source object will move off of that center point, but the center point of that shell will not move. But this also happens when you go to a moving reference frame â€" you see the exact same phenomenon.

Quote
THEREFORE, THE MICKELSON MORLEY EXPERIMENT ACTUALLY ONLY PROVED THAT THERE IS NO AETHER, AN THAT THE SOURCE OF THE LIGHT BEAM THAT WAS SPLIT IN 2, WAS AT THE MIROR.
Oh, come on, people knew that the light beam split in two at the beam splitter. That's why it's called a "beam splitter." And, yeah, there's no aether to carry the weight of the Galilean transformation that you are so fond of using in defiance of the Lorentz transformation that is the actual way to translate between boosted frames.

Quote
AND THEY PROVED THAT LIGHT WILL TRAVEL AT THE SAME SPEED FROM IT'S SOURCE, NO MATTER IN WHICH DIRECTION YOU SEND IT.
That was already obvious from Maxwell's equations. What Michelson and Morley proved, among other experiments, was that there was no way to reconcile the principle of relativity with Maxwell's equations using the Galilean transform, which had up to that point proven to be quite effective in preserving the invarance of physical law to boosts in velocity.

Quote from: Mousetrap on August 27, 2018, 03:12:38 AM
let me continue with the characteristics of light.

Light travels at C in a vacuum, FROM ITS SOURCE.
Travels at c according to whom? Please be specific about what conditions this will hold. I contend that it will travel at c regardless of the source or the receiver. Who will see the light travel at c? My answer is everyone, no matter how they happen to be moving, or even knowing whether they are moving and how fast.

Quote
it travels on a 360 degree SPHERE.
That would imply that light couldn't be columized, which it can definitely be.

Quote
if a light is flashed when a satellite travels at 7 000 Km per hour, the light will travel at C, FROM THE POINT OF ORIGIN.
actually, the satellite will travel away from the point of origin of the flash of light, and will experience the Doppler effect and will observe the light as red-shift.
Please compare the relativistic Doppler effect to the Doppler effect for sound. You will find a bit of a difference between the two.

Quote
If the satellite would ever be able to achieve C+ speeds, the satellite will travel outside of the Light sphere, and will not see the flash, untill the satellite slows down, and allows the light to gain.
Assertion of an impossible circumstance, even in a gedanken experiment. We can't get particles to move faster than the speed of light even with the lightest objects imaginable and with as much energy as we can pump into them. It's as if mass-bearing particles need infinite energy to even make it to c, let alone surpass it.

Quote
With the use of Light, one will be able to determine which satellite is travelling, and which is stationary.
And right there is a violation of the principle of relativity. Every other law of the universe obeys it; none of them care about who is stationary and who is traveling, and produce identical results regardless of who you consider to be each. Now you are proposing that this very well-established principle of relativity is wrong, using a phenomanon (light) that has already been brought under the umbrella of the principle of relativity, and in defiance of a theory that is the cornerstone of all modern physics.

Quote
This is exactly as we observe far off galaxies, where they are travelling away in relation to us.
Their light is measured in our frame of reference as red shifted, and our light at normal C.

Therefore, Newton was correct, Einstein not.
So you think that the Doppler effect is indicative of Newton? Bollocks. Look up "Relativistic Doppler effect", and you will see that Einstein also predicts that receeding objects will have redshifted light. After all, if all of the distant clocks are slowed down in the receeding object, wouldn't their oscillators slow down, and thus send out lower-frequency light?

No, you have not "disproved" Einstein here.

Quote from: Mousetrap on August 27, 2018, 05:03:45 AM
I had to find one discussion point in an attempt to show you what I mean.
Here we have the problem with SR and GR in a nut shell.
1. the Galilean transformation adds velocities up.
2. the Lorentz transformation shows how something behaves once it reaches huge speeds that can be measured in fractions of C.
And to which I agree.

But it actually shows you how light will appear to look from another time frame, in the observed time frame.
And it does not mean that Time and Length changed at all, but your interpretation of what you would observe about light is what you incorrectly now think is the error.
Wrong. All you have done is exposed your ignorance of what a reference frame is. If I have to keep in mind the "real" time and length of objects in a construct to do physics, even to assure myself that I'm not actually going mad, then you don't have a real frame of reference, but a mathematical curiosity. In order for it to be a frame of reference, you need to be able to forget about any measurement taken outside of that frame; you need to be able to derive what will happen using physical laws using how the world "appears" and treating it as if that were the reality. Again, that's what makes frames of reference special.

But if you don't have to refer to "real" times and lengths, ever, when doing physics then how are they "real"?

The Galilean transformation does not "add velocities." It takes you from one frame of reference into another where the frames are related by a boost velocity. In the tranformed frame, everything in the frame is moving with some additional velocity opposing the velocity that characterizes the transformation. You add velocities with a velocity addition formula. For Newtonian physics, that's the rather straightforward u = v + u' (where u' is the speed of whatever in the reference frame where something traveling at v is stationary). In relativity, u = (v + u')/(1 + vu'/c²). Why the difference?

Quote
Never did I say that Light can be stopped.
Light can never be stationary.
But travelling at C, It will appear to be just that!
If you have a true frame of reference instead of the mere mathematical curiosities you are actually using, you can treat the light as if it were actually stationary. That's the point of a reference frame. It is entire in and of itself.

Thank you for admitting that you are not actually using a Lorentz transforamtion, because no Lorentz transformation will allow you to see a light beam traveling at any velocity other than c, let alone "stopped." And you have already admitted that the Lorentz transformation legitimately shows how things would "appear."

Quote
2. the Lorentz transformation shows how something behaves once it reaches huge speeds that can be measured in fractions of C.
And to which I agree.

Quote
You see, this is what Lorenz and Einstein missed out on.
They presumed that it is impossible for us to determine if light, just as stationary objects, is moving or not.

If this is the case, then SR and GR is 100% correct.

However, the total contradiction about this characteristic of light is evident once we look at all the different claims made by physicists.
Yes, millions of physicists have it wrong when they claim that from all frames of reference c will appear at the same speed. You're right, c is a characteristic of light, but it won't allow you to determine who is moving and who is stationary. The laws of physics seem set up to preclude that.

Quote
For instance, even Stephen Hawking in his publications claims that light travels at C, and if you are travelling at C, and is not in the 3 dimensional sphere of light. (from its source obviously)[added by me], you will ever see that light!
Because obviously YOU know what Stephen Hawking meant better than the whole discipline of physics. Because, you know, if it was that obvious, then it wouldn't take Stephen Hawking to tell us this. I want to see which publication he makes the claim. I'm betting my life that you have missed an imporant piece of context.

Quote
Therefore, I conclude that Light has it's source, and Must travel from A to B, A being the origin, and B being our position.
This is not different from Einstein.

Quote
Sorry pal, you can not claim that light will have 2 measurable speeds, one where you travel at say 0.5C, and one stationary. and both will measure C.
Not only can I claim this, I do, and I have showed that the claim holds up. I already showed you the CALCULATION that shows that the Lorentz transformation (which you admit legitimately shows what each frame of reference will measure) will not change a velocity measured at c in one frame into a velocity measuring any other value in the other frame.

Quote
I know the Lorentz transformation by heart, and do not need you to give me the formulas.
For someone who knows them "by heart," you don't use them or understand what they are actually for. I call bullshit.

Quote
I need just one thing from you.

What, in your mind, does the Lorentz transformation measure.
The Lorentz transformation doesn't "measure" anything. It transforms how one frame of reference would place every event in it to how another frame of reference would place those same exact events, if those frames are related by a boost in velocity. Measurements are all carried out in their respective frames. In fact, they must be carried out in their respective frames; measurements made between events using coordinates from mixed frames of references are meaningless.

The term "transformation" comes from linear algrebra, and is in fact the main subject of that particular field. But I digress. A transformation transforms all object objects in the source vector space into their corresponding objects in the target vector space. The Lorentz transformation does analogously with frames of reference.

The fact that you ask such a question shows your lack of understanding.

Quote
Do it simple like,
take a vehicle that travels to the left at XV.
Shoot a laser forward.
shoot a laser backward.
now add both together.
Add the lasers together? Is that some sort of zen riddle?

Quote
Then explain why.
and and
I need to see where and what you see, that I dont.
I'll assume you're on the vehicle. I've already shown the calculations that tell you that a light beam will travel at c no matter what frame you're considering. So, I will see both light beams traveling at c according to my measurements. You also will see both light beams traveling at c according to your measurements. You have already accepted that the Lorentz transformation legitimately tells you what each frame of reference will measure.

Instead of asking me why you should observe all light beams going at c (you already knew what I was going to say), why don't you use the Lorentz transformation that you supposedly know "by heart" to show me how my description differs from your "reality".

Quote from: Mousetrap on August 27, 2018, 05:47:34 AM
(https://i.imgur.com/OUXo5gy.png)
I think this image will clarify everything that light has an origin, and travels at a certain speed.
Unlike you, I know what the bloody hell I'm looking at when I see the above. First off, that's for a given event, the one at the apex of the two cones. Second, that's for a particular reference frame. If you transform that same diagram onto another reference frame going at a different velocity, the image is going to be quite different. The cones will remain the same, though. However, the image of the time axis will be tilted and stretched in the direction of the boost velocity, and the hypersurface of the other frame's now will be tilted to the future in the direction of the boost velocity.

Quote
Therefore, to say that if one travels in the same direction of light, light will still be measured at C from your position, is incorrect.
Prove it using the Lorentz transformation. After all, you agree that the Lorentz transformation legitimately tells you how each frame of reference will measure.

If you're able to project another frame's diagram onto this one, you'll see the difference immediately.

Quote
Frankly, just as gravity is a fact, so it is that light has an original start off point, and any time frame must be in reference to that point of origin.
Special relativity and all of its implications are just as much fact as gravity. I'm sorry that it contradicts your Bible (but not sorry), but that is the fact.
Title: Re: Time dilation, length contraction, Relativity and the Bible!
Post by: trdsf on August 28, 2018, 01:04:51 AM
@Hakurei Reimu I can only say I admire your patience.
Title: Re: Time dilation, length contraction, Relativity and the Bible!
Post by: Mousetrap on August 28, 2018, 04:49:07 AM
Quote from: Hakurei Reimu on August 27, 2018, 11:44:55 PM
T...

Also, synchrotron radiation, produced from accelerating relativistic particles, do not travel at any speed measureably different from c in a vacuum. If the speed of the emitted light depended upon the speed of the source, here is where it would show up most profoundly and obviously. It does not. Synchrotron radiation is used every day to probe objects, and you kind of need to know when you should expect a scatter from your radiation to arrive at your detector so you can minimize noise. Discrepencies, especially as large as 2c, would be noted....
I have never claimed that the speed of light depends on THE SPEED OF IT'S SOURCE.
I CLAIMED THAT THE SPEED OF LIGHT IS C, BUT THAT LIGHT HAS AN ORIGIN AND TRAVELS AT THE SPEED OF c FROM THIS POINT.
iF THE SOURCE OF THE LIGHT WAS TRAVELLING AT 0.5 C FOR INSTANCE, THE LIGHT WILL TRAVEL AT C FROM THAT POINT ON, and not at 1+0.5 C.
lets clarify it more, if the source of the light beam travels at speed greater than C, then the source will totally overtake the light beam.
This is where you claim that the light beam will still be present on the source and will travel at C.
Therefore, you are the one that tells me that the speed of light is dependent upon the speed of it's source

Quote from: Hakurei ReimuHere's the thing. Michelson and Morley knew the Earth was moving, and furthermore has a wide veriety of movement through space. This has been a long established fact in cosmology for hundreds of years by the 19th century. Yet the Michelson-Morley experiment established that, dispite this known motion of the Earth, the aparatus was behaving as if it were completely stationary.
That's because you don't know your physics. That's been obvous to us for about a month now.
And they set one light beam from one point on their apparatus, to differnt directions in equal lengths of what, 2 meters, and tried to see if there was any distortion in the light waves when the light beams was meeting again.
It is as if I go clockwise, and you anti clock wise, and we hope one of us arrives earlier.
Quote from: Hakurei ReimuI
Exactly wrong. If the light was traveling as a 3D spherical shape, why would they need to aim the laser at all?
Perhaps you do not know that even a laser flares out over distance.
This is exactly why it is a laser, because the light is concentrated in one direction.
The beam sent out to the moon is as thick as a telephone pole, but only a few photons manage to be detected upon it's return.




Title: Re: Time dilation, length contraction, Relativity and the Bible!
Post by: Mousetrap on August 28, 2018, 05:28:18 AM
Quote from: Hakurei Reimu on August 27, 2018, 11:44:55 PM
Quote from: Mousetrap on August 27, 2018, 03:12:38 AM
let me continue with the characteristics of light.

Light travels at C in a vacuum, FROM ITS SOURCE.[/quote

Travels at c according to whom? Please be specific about what conditions this will hold. I contend that it will travel at c regardless of the source or the receiver. Who will see the light travel at c? My answer is everyone, no matter how they happen to be moving, or even knowing whether they are moving and how fast.
I really think you are posting a lot more than what you should.
Look at this one for instance.
What dont you understand about my statement.
Light travels at C from it's source.
Thats not so difficult to understand.
If a flash sets off in space and light travels away from it's source, we can say that light travels at C from it's source to where we are now.

Here is the scientific evidence.
You are looking at light that came from outer space, and know it should be 5 billion years old.
the light you observe originated from a galaxy that is not there anymore.
why, because in 5 billion years it moved so far away, that its position as it is NOW, will only be visible in another 5 billion years.
However, the light that we now observe, came from its position of origin.
and it traveled in a straight line towards us.
But, this light was sent out into space in a 3D sphere, and we are lucky to be in the area where the surface of this sphere reached us.

Therefore, your claim that everyone will see light travel at C, is incorrect, and on the speed that we are moving away from the origin of the light beam, does prove that we see light moving slower. But not as slow to show it less than C, only to move it into a red shift.
This is your answer.

We observe light from distant galaxies at red shift.
Why?
Due to the Doppler effect.
Why?
Because we are moving away from the source of where the light came from.
So what will you say?
well, just as sound waves is observed lower in frequency as the source moves away from one, just so will the frequency of light lower, rendering a red shift.
Now, this is where I make a claim that you do not see light at C wherever you are, but I say wherever the light came from.

If I get on a rocket and travel away from the observed red shift galaxy I saw from Earth, It will increase in it's red shift.
As we travel faster we will eventually outrun the light received from the red shift galaxy, and will eventual catch up on light that was sent out prior to our departure.

In theory, travelling away from this galaxy at 5 billion times C, and we will catch up with the light that came from it 5 billion years ago.
It does not mean we became 5 Billion years younger.


Title: Re: Time dilation, length contraction, Relativity and the Bible!
Post by: Hakurei Reimu on August 28, 2018, 11:06:04 PM
Quote from: Mousetrap on August 28, 2018, 04:49:07 AM
I have never claimed that the speed of light depends on THE SPEED OF IT'S SOURCE.
Your descriptions are so confused that I don't think even you know what you're talking about. Some of your descriptions only make sense if light depends on the source's speed. Others, not.

Quote
I CLAIMED THAT THE SPEED OF LIGHT IS C, BUT THAT LIGHT HAS AN ORIGIN AND TRAVELS AT THE SPEED OF c FROM THIS POINT.
iF THE SOURCE OF THE LIGHT WAS TRAVELLING AT 0.5 C FOR INSTANCE, THE LIGHT WILL TRAVEL AT C FROM THAT POINT ON, and not at 1+0.5 C.
Light will travel at c according to which reference frame, MT? Without a reference frame, without telling me who or what is doing the measurement, your statement that 'Light goes at c' is meaningless... unless you mean that every observer will measure light going at c â€" which blasts your claim that Einstein's relativity is false clear out of the water.

The question is simple: Who gets to see light going at its true speed?

To clarify, we normally specify speed in terms of some understood frame of reference. When it is said that I'm driving my car at 30 mph, that 30 mph is the speed relative to the road. When it is said that the surface of the earth spins at ~1000 mph, we mean that the earth's surface is going at ~1000 mph relative to its center point. When we say that the earth orbits the sun at 29.78 km/s, we mean that the Earth is moving zt 29.78 km/s relative to the sun. Get it? Specify who or what gets to see light moving at c, and you'll be talking.

Quote
lets clarify it more, if the source of the light beam travels at speed greater than C, then the source will totally overtake the light beam.
That is your assertion. I have yet to see any experiment or line of reasoning from you that demonstrates your assertion. I understand that if the Galilean transformation is the transformation between frames related by a boost velocity, then someone can go faster than c according to some (yet unspecified) privilaged frame of reference, but it's not. It's the Lorentz transformation that properly transforms between boosted reference frames; the Galilean transform is only an approximation of the Lorentz transformation.

Do you know what an "approximation" is, MT?

Quote
This is where you claim that the light beam will still be present on the source and will travel at C.
Amazing, two particles traveling at exactly the same velocity emitted from exactly the same point arrive at a distant point at exactly the same time. [/sarcasm] Of course, the ship can't travel at or over c, being a massive object.

Quote
Therefore, you are the one that tells me that the speed of light is dependent upon the speed of it's source
No. I cited the synchrotron radiation as specific proof otherwise. What I claim is that all observers will observe the same speed of light c from their frame of reference. If a ship traveling at nearly c in your reference frame emitting a pulse of light forward, then the light pulse will only be barely faster than the ship, exactly c; on the ship, that exact same light pulse will be measured as going as exactly c as well, but I outside the ship, explain the discrepency as the lengths and time on the ship being distorted. The ship will make exactly the same claim as I, my lengths and times are distorted to the ship.

It's a lot more subtler than your inane "you are the one that tells me that the speed of light is dependent upon the speed of it's source" strawman. It also tells me that you do not, in fact, understand special relativity.

Quote
And they set one light beam from one point on their apparatus, to differnt directions in equal lengths of what, 2 meters, and tried to see if there was any distortion in the light waves when the light beams was meeting again.
No. The light would produce an interference pattern regardless of what they did. They rotated the aparatus 90° and watched for changes in the fringe pattern, because the two arms would change roles, and light that arrived in phase x and y from each arm would instead arrive in phase y and x respectively, causing a change in the fringing pattern.

The time that the light takes to travel down the tube perpendicular to the speed of the Earth is straightforwardly t1 = 2l1/c·sqrt(1-v²/c²) while the parallel tube is t2 = 2l2/c·(1-v²/c²). As you can see, the expressions are different â€" they will produce different answers, and it will show up as a change in the interference patterns as the aparatus is rotated. With some approximation, the change in time between one orientation and another is Î"t - Î"t' ≈ (l1+l2) v²/c³. In the particular case of their experiment, the earth's orbit around the sun would have produced fringe shifts of 0.4, and their aparatus could detect them down to 0.01.

And before you say, "But that's aether!" the only thing necessary for this is to assume that the Earth has some speed relative to the special reference frame where the speed of light is exactly c. The aether only provided a physical grounding for this frame. This is why I say you have to specify the frame where c is the speed of light, if it's not c for everyone; it's what allows calculations and experiments such as this.

Of course, being cagey is perhaps what you want.

Quote
Perhaps you do not know that even a laser flares out over distance.
It does, but it doesn't disperse as much as a full 3D shell does. With reasonably good optics and a well-columnated beam over a sufficiently limited distance, a laser can be treated as if it were a line.


Quote from: Mousetrap on August 28, 2018, 05:28:18 AM
I really think you are posting a lot more than what you should.
Look at this one for instance.
What dont you understand about my statement.
Light travels at C from it's source.
Which is what I've been saying, too, except that I assert that every frame will measure it at c, regardless of which one does it. You apparently think that the speed of light is constant according to some unspecified reference frame. Specify it. Because if you're using a Galilean transformation to transform between reference frames, only one particular reference frame will get to see light going at c. Everyone else will only get to see speeds of c-v (bolding indicates vectors) where v is the boost velocity that takes you from this special frame to yours.


Quote
Thats not so difficult to understand.
If a flash sets off in space and light travels away from it's source, we can say that light travels at C from it's source to where we are now.
That's because you're cutting too much out of your "understanding." Every other speed we talk about is done in a background of an assumed frame of reference. You need to say it explicitly here. Which frame of reference gets to see light traveling at c?

Suppose in your example, I observe light going some velocity c1, and my friend, who is traveling toward me in the opposite direction of the light beam at some v. Of course, he will see the light beam travel at c2 = c1+v (because if he saw it traveling at c1 exactly, that implies Einstein's relativity and your argument collapses). Now, who has observed the real speed of light? Have either of us observed the real speed of light? Do either of us know what the real speed of light is? How do we know that light has a constant speed, given that nothing else in the universe seems to?

Quote
Here is the scientific evidence.
You are looking at light that came from outer space, and know it should be 5 billion years old.
the light you observe originated from a galaxy that is not there anymore.
why, because in 5 billion years it moved so far away, that its position as it is NOW, will only be visible in another 5 billion years.
However, the light that we now observe, came from its position of origin.
and it traveled in a straight line towards us.
So far, you have not described anything different from Einstein's relativity.

Quote
But, this light was sent out into space in a 3D sphere, and we are lucky to be in the area where the surface of this sphere reached us.
If it's expanding in a 3D sphere, it's going to reach us with probability 1 â€" certainty. We are not particularly lucky. 3D shells of light also occur in Einstein's relativity.

Quote
Therefore, your claim that everyone will see light travel at C, is incorrect,
That's a non-sequitor. There is no logical connection between anything you have said above and the conclusion that I'm wrong. Everything you have said is consistent with Einstein's relativity, and Einstein's relativity does indeed have c as a constant for all observers. Your above "argument" doesn't get you there.

Quote
and on the speed that we are moving away from the origin of the light beam, does prove that we see light moving slower. But not as slow to show it less than C, only to move it into a red shift.
This is your answer.
Your answer is stupid. The cosmic microwave background is extremely redshifted, from the visible spectrum (~500 nanometers) to the microwave spectrum (~3 mm) which corresponds to a change in wavelength of 6000 times. This requires us to receed from the emitter of the cosmic microwave background at 0.9998c. This is decidedly in the realm of detectability, and we do not see any such light from the CMB going this slow.

Your notion that the observer sees c a different value because of his motion will also make it vulnerable to a Michelson-Morely type experiment.

Quote
We observe light from distant galaxies at red shift.
Why?
Due to the Doppler effect.
Why?
Because we are moving away from the source of where the light came from.
So what will you say?
Again, we have a relativistic Doppler effect. The mere presence of a Doppler effect does not indicate that your theory is true. But it does predict a different number for the frequency shift.

The nonrelativistic Doppler effect has f = (c ± v)/(c ± u) f0, where c is the speed of the wave (in this case, light), v is the speed of the receiver (positive for approaching, negative for receeding) and u is the speed of the source (positive for receeding, negative for approaching).

The relativistic Doppler effect has f = sqrt(c-v/c+v) f0, where v is the relative speed of the observer and source (negative if observer and source are moving towards each other, and positive if they are moving away).

And, nope, the relativistic Doppler effect is what is observed in experiments like the Ivesâ€"Stilwell experiments, Mössbauer rotor experiments, and sensitive spectroscopy experiments (because atoms emit in very precise thin-line spectra, and their shift can be readily quantified, and may be driven to very high velocities and their speeds can be very well controlled).

Quote
well, just as sound waves is observed lower in frequency as the source moves away from one, just so will the frequency of light lower, rendering a red shift.
Now, this is where I make a claim that you do not see light at C wherever you are, but I say wherever the light came from.
You do realize that sound does not change speed when it is Doppler shifted, right? That's why supersonic aircraft create sonic booms when they fly past. The nonrelativistic Doppler effect does not affect the propagation speed of the wave; the propagation speed of the wave is a property of the medium.

Quote
If I get on a rocket and travel away from the observed red shift galaxy I saw from Earth, It will increase in it's red shift.
Yes, it will. As it will do in the relativistic case.

Quote
As we travel faster we will eventually outrun the light received from the red shift galaxy, and will eventual catch up on light that was sent out prior to our departure.
This is the only part where the relativistic case differs from yours, and yours is based on speculation only. You have not done nearly enough to prove your case that any material rocket could catch up to previously emitted light waves.

Quote
In theory, travelling away from this galaxy at 5 billion times C, and we will catch up with the light that came from it 5 billion years ago.
It does not mean we became 5 Billion years younger.
It does not in the relativistic case, either. At no speed available to you in relativity (and FTL speeds are not available to you in relativity), there will be no case where you will observe a galaxy reverse-aging.
Title: Re: Time dilation, length contraction, Relativity and the Bible!
Post by: Hakurei Reimu on August 28, 2018, 11:38:02 PM
Let me take a stab at trying to explain the error in your thinking, MT.

Suppose you and your friend are trying to locate two points A and B on the Euclidean plane. This plane doesn't come pre-equipped with a coordinate system, so you have to roll your own. You come up with your own coordinates for A and B, and give the points PA = (xA, yA), and PB = (xB, yB). Your friend has chosen the the same orgin for his coordinate system, but his is rotated some angle Ï• from yours. He assigns the same points the coordinates P'A = (x'A, y'A), and P'B = (x'B, y'B).

Now, I hope that nobody thinks that the above is really any cause for argument if the coordinates assigned to PA, PB ≠ P'A, P'B. They're different coordinate systems, so the exact numbers assigned really don't matter. Heck, even the difference between the coordinates don't matter, because the coordinate systems for Î"P and Î"P' are different, so the values we come up with don't really matter that much. All we have to do is use the appropriate transformation to switch coordinate systems:

x' = x cos Ï• - y sin Ï•
y' = x sin Ï• + y cos Ï•

x = x' cos Ï• + y' sin Ï•
y = -x' sin Ï• + y' cos Ï•

No problem, right? All calculations of the lengths and angles are going to coincide, because they take into account the fact that x and y are mixing differently in the two coordinate systems.

Well, a Lorentz transformation is the exact same thing. It's a coordinate transform from one set of coordinates in one frame to another set of coordinates in another frame where the frames are related by some boost velocity, v. It gives different coordinates to the same events. As such, it shouldn't make a difference whether you use the t, x, y, z coordinates or the t', x', y', z' coordinates, so long as you don't try to mix coordinate of different systems. Both sets of coordinates and their coordinate systems are valid, but none are right in an absolute sense.

What you are insisting, MT, is that one particular set of coordinates are just that â€" right in an absolute sense. You are insisting that some difference in time coordinates denotes the "true" time, without considering the space dimensions that would mix with it over any significant distance. You are insisting that some difference in space coordinates denotes the "true" length between events, without considering the time dimension that would mix with it. Sorry, even if you may have come up with those numbers in good faith, those numbers you're coming up with are just coordinates, and they depend on how you choose to measure space and time, and are not "true" in any absolute sense.

There are measurements of time and space that do this, that are analogous to the distances and angles in the Euclidean plane and are invariant to the Lorentz transformation as the Euclidean distance and angles are invariant to the rotation transformation. The interval is the analogy to Euclidean distance in relativity, defined (ds)² = c²(dt)² - (dx)² + (dy)² + (dz)².* The interval is not positive-definite, and in this form is in dimensions of length². If the interval is positive, then the proper time between points is dÏ,, = sqrt((ds)²)/c and does not exist if the interval is zero or negative. If the interval is negative, then the proper length between points is dσ = sqrt(-(ds)²) and does not exist if the interval is zero or positive. Since the interval is Lorentz invariant, so too are proper time and proper length. (Note, in the above, Î" can be substituted for d if the spacetime is flat.) If any length or time is God's length and time, it's proper length and proper time.

* Note that the exact definition of the interval, proper time and proper length vary depending on the author, but they all come to the same result. Interval definitions differ only by sign and units. Proper time and length differ only by the choice of units for time and distance.
Title: Re: Time dilation, length contraction, Relativity and the Bible!
Post by: Mousetrap on August 29, 2018, 03:05:32 AM
And again, I do not have any problems with the mathematics.
I have a problem with what is claimed with the results, on what is observed.

And with these long posts where you try to throw all the info as evidence that I am wrong, you are allowing a huge amount of info to smother a few important facts.

Please note.
We have ample time to go through the SR and GR theory.
one post at a time.

Now, before we continue, all I want to clarify is the following.
If we flash a light on THE OUTSIDE OF a moving satellite traveling at say, 0.5 C, will the light now travel at C when observed from a stationary satellite, next to the position of the stationary one when it fired?
If so, do you agree that the exact point of where the light flashed, is the starting point for the light to travel parallel with the moving satellite?

Lets keep it simple and short.

Title: Re: Time dilation, length contraction, Relativity and the Bible!
Post by: Baruch on August 29, 2018, 06:57:12 AM
One wall of text is no more read than another wall of text.

But then I have studied this, and acknowledge that human intuition is wrong.

However people do draw false conclusions "that all things are relative, including morality" that don't follow from the math.
Title: Re: Time dilation, length contraction, Relativity and the Bible!
Post by: Hakurei Reimu on August 29, 2018, 12:06:31 PM
Quote from: Mousetrap on August 29, 2018, 03:05:32 AM
And again, I do not have any problems with the mathematics.
I have a problem with what is claimed with the results, on what is observed.
The mathematics have a direct effect on what you will observe. That's why they're used: to make predictions on what should be observed so they can be tested. The predictions of special relativity is what is observed at high speeds; Newtonian mechanics is only the low speed approximation of relativistic mechanics.

Quote
And with these long posts where you try to throw all the info as evidence that I am wrong, you are allowing a huge amount of info to smother a few important facts.
You have not presented "facts" at all. You have presented speculations and assertions as if they are facts, but they are not facts. You have asserted that the speed of light would appear slow to a receeding observer, when our instruments were good enough to detect tiny differences in the speed of light to the tune of one part in ten thousand even in the nineteenth century (Michelson and Morley), and are only better today. Evanson et al. measured the speed of light to 299,792.4574 ± 0.0001 km/s in 1973 â€" that's as good as 10 cm/s, and that was forty years ago. If there were any truth to your notion that you could observe red-shifted Doppler shifted light as slower, it would have been noticed long ago. Furthermore, other experiments show that light follows the relativistic Doppler effect of relativity and not the non-relativistic Doppler effect. They produce different degrees of redshift and there's also an asymmetry in the non-relativistic Doppler effect that we don't find in light.

I refer you to my Feynman video once more. Evidence is king. What we see about light is consistent with Einstein and not with Newton.

Quote
Please note.
We have ample time to go through the SR and GR theory.
one post at a time.
If you don't spam posts at me, I won't throw huge compilation posts at you. You do realize that my posts are really large because I'm responding to multiple posts, right?

Quote
Now, before we continue, all I want to clarify is the following.
If we flash a light on THE OUTSIDE OF a moving satellite traveling at say, 0.5 C, will the light now travel at C when observed from a stationary satellite, next to the position of the stationary one when it fired?
If so, do you agree that the exact point of where the light flashed, is the starting point for the light to travel parallel with the moving satellite?
How do you figure out that the satellite you're shooting that light at is really moving at 0.5c, or the satellite you're on is really stationary? Every other speed we use is referenced to something else, the thing it is moving at that speed relative to. It's vital in Newtonian mechanics. Newtonian mechanics doesn't have a notion of absolute speed, and "stationary" is a term of convenience â€" if you say some frame is stationary, it's stationary, regardless of any motion we know it to have. For most of our history, we've been calling a platform that has been puttering about at around 29.78 km/s in various directions "stationary" â€" the Earth.

Even in Newton's world, you can't build a speedometer that could tell you your absolute speed. It was principally impossible to do. To be an absolute speedometer, an aparatus is required to work differently depending on how fast the whole setup is "really moving." However, none of our laws of physics is sensitive to that â€" every aparatus works the same regardless of the speed of the physical setup. All speeds are equivalent in Newton's world as well as in Einstein's.

So you've barely left the gate and already you're entangled in a snarl. I'm sitting on this satellite and I'm calling it "stationary," but I don't know that it's stationary in any absolute sense. You say that I'm stationary, but the me on the satellite doesn't know that and has no means of finding that out. All Isatellite know is that the other satellite is moving at 0.5c in a frame of reference I have labeled as "stationary" as a matter of convenience. Isatellite see the flash traveling at c in all directions from the space coordinates they began from, but Isatellite don't know if that's really the same point in any absolute sense, or even if that term means anything at all. All Isatellite can tell you is that it's the same space coordinates in my frame's coordinate system, and in that coordinate system all the light is moving at c away from that coordinate. That is all. Anything further would be speculation.

Now, let's suppose Isatellite were on the other satellite, the one that the first satellite sees as going 0.5c. I'satellite would also call this satellite "stationary," as a label of convenience. Again, I'satellite don't know if that's true in any absolute sense. You say that it's moving, but I'satellite have no means of assessing that. I'satellite see the first satellite, which you claim is stationary, is moving at 0.5c, and for good measure let's turn our frame's orientation 180° so that the first satellite is moving along the same nominal axis and in the same sense in this new frame as this one is moving in the old frame.

Let's hold off the emission of our light flash for the moment to assess our scenario. We are right now in a scenario that, even in Newtonian mechanics, would be perfectly symmetrical from both satellites. There would be no means of assessing whether Isatellite or I'satellite are "really moving" or "stationary," if either of us are. At least one of us is definitely moving, to be sure, but we can't tell who is; it may be that we are both moving in opposite directions at the same speed, or we're moving in the same direction but at speeds that would make our partner appear to move at 0.5c relative to us. We don't know and have no means of assessing our absolute state of motion, or even if we had an absolute state of motion.

Now let's emit that flash. I've already discussed what Isatellite would observe and what Isatellite would be able to say and not say. Now, what does I'satellite observe? The exact same thing! I'satellite would observe the flash spreading out at c from the same space coordinates they began from in my' frame, but I'satellite don't know if that's really the same point in any absolute sense either, or even if that term means anything at all. All I'satellite can tell you is that it's the same space coordinates in my' frame's coordinate system, and in that coordinate system all the light is moving at c away from that coordinate. That is all. Anything further would be speculation.

So the scenario is completely symmetrical from both ends! I've derived the above answer using only basic principles of Einstein's relativity, but a Lorentz transformation would reveal the same answer. Before emitting the flash, neither of us know who's "really moving" and after, we still don't know.

Quote
Lets keep it simple and short.

It's not simple and it's not short because a simple and short answer requires us to be on the same page as far as physics understanding goes, and we're not.
Title: Re: Time dilation, length contraction, Relativity and the Bible!
Post by: aitm on August 29, 2018, 07:37:13 PM
That really is fascinating stuff that is nowhere in the babble.....but god thinks how a guys drinks water from a river is very important. Odd how xians seem to "pretzelize" themselves into all kinds of science and yet can't figure out why god thinks it is incredibley important how a guy drinks water or keeping women away when they are dirty.....

Xians love to ignore what the babble really says and invent all the sciencey shit the babble doesn't say and proclaim their god is all that.

Still ignoring that 2/3rds of the stars fell to the earth already....but seem to ignore that one as a typo....
Title: Re: Time dilation, length contraction, Relativity and the Bible!
Post by: Baruch on August 29, 2018, 07:38:28 PM
The river water lappers ... were transgenic ... part dog.
Title: Re: Time dilation, length contraction, Relativity and the Bible!
Post by: Mousetrap on August 30, 2018, 02:14:36 AM
Quote from: Hakurei Reimu on August 29, 2018, 12:06:31 PM
The mathematics have a direct effect on what you will observe. ...

It's not simple and it's not short because a simple and short answer requires us to be on the same page as far as physics understanding goes, and we're not.
Thank you for staying on the one point of the discussion.
And what you said is 100% true.
I would like to take this examination a bit further.

Please do not think I am being mischievous, all I am doing is to cut through some thinking I have been working on for a very long time, and the only way to find answers, is to speak to someone with the knowledge you have.

Now, I would like you to critique the following.

Light travels at C in any reference point, no matter where it comes from, or where you are in space.
If, lets say it is possible that you can see the actual light waves in your position (because you made a special contraption to observe it), and you observe the waves coming from head on towards you.
You turn to your side and let it pass through your contraption.
You will see that the waves are actually running the frequencies of the color you observe, and it will be one of 2 colors indicating its origin.
If it is blue, you will know that the origin point of this light source was moving towards you, if red, you will know it was moving away from you.
You will not be able to measure the speed of the light, because you dont know when the light started to shine, and it's nose passed at any uncertain time which you did not observe. (As Roemer did when he measured light speed in 1676 with Io, he measured the nose of the light beam)

Now you turn to your rear, and let some head-on light coming in the opposite direction from the first, pass your contraption.
Again, you will see the waves of the light as exactly observed in the first. Red shift, tells you the source was moving away, blue tells you the source advanced.

Now my logical conclusion says,
1. Light is directional.
2. its position in relation to velocity can be determined with the adjustment of your velocity, to have equilibrium in the light spectrum.
3. If you adjust your velocity on that very light beam, you will discover the light source's direction.
4. Maintaining the light's true spectrum, will now give you the means you are observing the correct wave frequency.
5. Looking at the light coming from the opposite direction, will now give you the true shift in relation to the first light beam, and it will also supply you with the information about its source, and velocity it is moving in relation to the first source of light. So, we will have the X co ordinates of light source 1 and of light source 2.

Now we continue with our experiment.

We hyper speed at almost C towards the first light beam.
1. We look at the light waves and notice it is turning blue shift (actually we will move into X rays and beyoned), because more waves passes us than before we began our movement.
2. The light shift we measured coming from behind turns more red-shift (we will see slower than Ultra low frequencies.)

Now, do you also perceive that what is going on is that we are measuring light frequencies, (if I can compare the light waves with the keys of a piano that continue for millions of miles), and we are looking at these keys, and not the nose or tail of the light beam.

This is my whole story.
Light is all around us, and we observe it's waves.
We do not observe the speed of light at all, but the ribs of this huge snake that passed us a long time ago.
The light spectrum we do not see, is the speed at which the frequencies are passing us, either because we are moving towards, or away from the source at a speed that changes the frequency out of our perception.

Now, it will be clear to you on what I say about the speed of light when, if I flash a beam, for a millionth of a second towards the Moon (our old experiment) it will take one second to get there.
the same if it is flashed from the Moon,
AND IF IT IS FLASHED ON THE SPACE SHIP.
Because now we will measure the head, and the tail of the light beam, and not the frequencies of the piano key.
If this light beam was switched on say a few minutes ago, then all the calculations on SR will hold true, but not when we change the situation where we pulse a beam, and measure it's speed.

I hope you understand what I say, and I am not doing this to mock, or to say I am somehow above science.
It is just some long thought out problems I worked on for a few years.
Greetings


Title: Re: Time dilation, length contraction, Relativity and the Bible!
Post by: Hakurei Reimu on August 30, 2018, 12:16:53 PM
Quote from: Mousetrap on August 30, 2018, 02:14:36 AM
Light travels at C in any reference point, no matter where it comes from, or where you are in space.
If, lets say it is possible that you can see the actual light waves in your position (because you made a special contraption to observe it), and you observe the waves coming from head on towards you.
You turn to your side and let it pass through your contraption.
You will see that the waves are actually running the frequencies of the color you observe, and it will be one of 2 colors indicating its origin.
If it is blue, you will know that the origin point of this light source was moving towards you, if red, you will know it was moving away from you.
Let's say that you know that the light emitted is monochrome light of a middling frequency, like green. Then the above will be true, except to note that you don't know that it's because the light source is moving towards you or you are moving towards it or both of you are moving towards each other. All you can say is that it's moving towards you in your frame of reference.

Quote
You will not be able to measure the speed of the light, because you dont know when the light started to shine, and it's nose passed at any uncertain time which you did not observe. (As Roemer did when he measured light speed in 1676 with Io, he measured the nose of the light beam)
No. Any real light pulse would be made up of untold bajillions of photons traveling in formation. You can use a semi-silvered mirror to pick off some of the light as it passes by the nose into a detector, and catch the rest some known distance away in another detector, and measure the time between when the first detector goes off and the second detector goes off. We have come up with many ways of measuring the speed of light, some of them are immune to the particular frequency the light comes in as.

Of course, we're assuming that our special contraption is able to track light along its path, so all we have to do is note when a pulse of light passes a known point, and then passes another known point a known distance away, do your math and Bob's your uncle.

Quote
Now you turn to your rear, and let some head-on light coming in the opposite direction from the first, pass your contraption.
Again, you will see the waves of the light as exactly observed in the first. Red shift, tells you the source was moving away, blue tells you the source advanced.
Again, I'm assuming that you know ahead of time that the light is monochromatic and some middling frequency like green.

Quote
Now my logical conclusion says,
1. Light is directional.
That statement is vague. I do not really know what you mean by that statement. If you mean that these light beams come to us in a specific direction, well, yeah. If you mean light has an anisotropy â€"that it's behavior is fundamentally different depending which direction it's traveling in the universeâ€" then no.

Quote
2. its position in relation to velocity can be determined with the adjustment of your velocity, to have equilibrium in the light spectrum.
What? I think you mean that you know by how much the frequency of your light is shifted due to the Doppler effect. In many cases we do know the unshifted spectra of various sources because of absorption lines, which take place at very specific frequencies. Figuring out what the unshifted spetrum is takes some doing, but it can be done. In our example, we're assuming a monochromatic source at a known frequency.

Quote
3. If you adjust your velocity on that very light beam, you will discover the light source's direction.
Don't you already know the light source's direction? You did point your aparatus in the right direction, correct?

Or do you mean the velocity your light source is going? First off, you already know how fast it's approaching or receeding in your frame of reference from the Doppler shift. (We're assuming that the source is far off so transverse effects are minimal.) Adjusting your speed accordingly will bring you nearly comoving with the light source. But I must stress that you don't know that's the source's "true" velocity. Once again, "stationary" is a term of convenience in both Newtonian and relativistic mechanics.

Quote
4. Maintaining the light's true spectrum, will now give you the means you are observing the correct wave frequency.
I've been assuming that you already knew the light's true spectrum because you had prearranged it with your compatriots. If you don't already know what spectrum the incoming light should be, you will not be able to recover it without knowing the relative velocity of your source. All the frequencies get shifted, after all, so if your Doppler shift is D, then the frequency f gets shifted to Df, but the frequency f/D now gets shifted to Df/D = f. If you had a uniform spectrum, it will look exactly the same under the Doppler effect, no matter what the shift.

Quote
5. Looking at the light coming from the opposite direction, will now give you the true shift in relation to the first light beam, and it will also supply you with the information about its source, and velocity it is moving in relation to the first source of light. So, we will have the X co ordinates of light source 1 and of light source 2.
"True shift"? They're all true shifts, dearheart. You'll get the Doppler effect for the second source as it would appear to the first, because you are now comoving with the first source.

Quote
Now we continue with our experiment.

We hyper speed at almost C towards the first light beam.
1. We look at the light waves and notice it is turning blue shift (actually we will move into X rays and beyoned), because more waves passes us than before we began our movement.
It's still called a "blue shift" even if the frequency goes into the X-rays. If the original light is microwaves that get shifted to the red part of the visible spectrum, it's still called a blue shift.

Quote
2. The light shift we measured coming from behind turns more red-shift (we will see slower than Ultra low frequencies.)
Yes.

Quote
Now, do you also perceive that what is going on is that we are measuring light frequencies, (if I can compare the light waves with the keys of a piano that continue for millions of miles), and we are looking at these keys, and not the nose or tail of the light beam.

This is my whole story.
Light is all around us, and we observe it's waves.
We do not observe the speed of light at all, but the ribs of this huge snake that passed us a long time ago.
The speed of light is an inference. We have to involve our measuring sticks in some way, because that is how we get the vital distance measurement to divide the time traveled by. That's just simple dimensional analysis. However, once we have a distance to work with, we can measure the speed of light as directly as we measure anything.

Quote
The light spectrum we do not see, is the speed at which the frequencies are passing us, either because we are moving towards, or away from the source at a speed that changes the frequency out of our perception.
Why are you bringing perception into it? We're doing measurements. Relativity affects measurements.

Quote
Now, it will be clear to you on what I say about the speed of light when, if I flash a beam, for a millionth of a second towards the Moon (our old experiment) it will take one second to get there.
the same if it is flashed from the Moon,
With respect to the frame of reference that the two both share because they are co-moving. This is because they're using the same time coordinate, t, to judge that this would be one second.

I'm puzzled what the hell this has to do with your previous discussion, though. I don't see any sort of connection between your discussion of the Doppler shift and our Earth-Moon supership example. It's actually a little jarring, this segue.

Quote
AND IF IT IS FLASHED ON THE SPACE SHIP.
Yes, if the space ship emitted a light pulse just as it was staring off from the Earth, the Earth-Moon frame of reference will say that it will take one second to reach the moon. But the rocket is using it's own time coordinate, t', to judge that time, and it does not agree with the Earth-Moon time.

Quote
Because now we will measure the head, and the tail of the light beam, and not the frequencies of the piano key.
You're using really confused terminology. I think I understand, but I'm still confused what this has to do with the previous discussion.

Quote
If this light beam was switched on say a few minutes ago, then all the calculations on SR will hold true, but not when we change the situation where we pulse a beam, and measure it's speed.
SR applies to pulsed beams just as well as applies to beams turned on for minutes on end.

The statement, "The light pulse takes one second to go from the Earth to the Moon" is an inference based upon the sighting of the Earth clock reading 12:00:00 from the moon at 12:00:01 (moon time) and the knowledge that the Earth clock and Moon clock are synchronized within their frame of reference. The Earth clock measures Earth time, as the Earth clock travels with the Earth's world line (its path through time and space). The Moon clock measures Moon time, as the Moon clock travels with the Moon's world line. Neither the Moon clock nor the Earth clock measures the time along the ship's world line; that job falls to the shipboard clock.

Just as in calculating the length of a diagonal of a rectangle, the separation of the two sides must be taken into account as well as the length of those sides, in calculating the duration between two events, the spatial separation between the two events also enters into it. Measuring one side of the rectangle is not enough to tell you the length; you must measure across the adjacent side and use the Euclidean distance to get the diagonal's length (or use a ruler across the diagonal, which gives you a direct measurement of the length). Consulting the Earth and moon clock is not enough to tell you the duration along the path of the rocket; you must consider the distance traveled and use the proper time formula (which is the Minkowskian distance formula) to get the duration along the rocket's flight path (or just consult the shipboard clock, which gives a direct measurement of the rocket's flight time).

And don't forget that your background assumption that the Earth and Moon aren't moving is just that, an assumpiton. Once more, "stationary" is a term of convenience in mechanics, Newton or Einstein. For all you know, your Earth-Moon system may be itself moving at a pretty good clip in either direction, so that in this new frame of reference, the Earth-Moon travel time for light of one second is just as false to them as the rocket travel time of 81.6 µs is to the Earth people.

Quote
I hope you understand what I say, and I am not doing this to mock, or to say I am somehow above science.
It is just some long thought out problems I worked on for a few years.
Greetings
The problem with thought experiments is that they require a thorough understanding of physics in order to come up with the correct answers. You can only perform gedanken experiments with physics you understand. Einstein understood the physics at the time, including the Newtonian physics he eventually overthrew. That's why he was able to come up with relativity and revolutionized physics. You have revealed that your grasp of physics is not nearly as solid.

The other thing is that you've been thinking about it for years, but you haven't been performing experiments to test what you've been thinking, or even referencing experiments that have already been done to test your ideas, have you? I refer you once again to Professor Feynman:

https://www.youtube.com/watch?v=LIxvQMhttq4

So far, you have taken your guess, which is the first step, and have attempted the calculation of consequences step in some cases to varying success, which is the second step. I have never seen you proceed to the third, most vital step, which is to compare your calculations with reality â€" to experiment or observation.

You seem to believe that physics swallowed Einstein's relativity whole and unexamined because Einstein was so briliant. This is not true. At the time, Einstein was working as a patent clerk, and was a nobody. Relativity is what made him. His theory of relativity was tested (along with quantum mechanics) throughout the first part of the 20th century, with very carefully designed experiments to test all aspects of it, and relativity passed all of them. It might be difficult to see how a particular experiment tests ideas like time dialation, but they do because all of the concepts of relativity are very intimately woven together.
Title: Re: Time dilation, length contraction, Relativity and the Bible!
Post by: Baruch on August 30, 2018, 12:46:41 PM
With sophists of any stripe, it is mental masturbation all the way down.  Unless of course you have a repeatable quantitative experiment.
Title: Re: Time dilation, length contraction, Relativity and the Bible!
Post by: trdsf on August 30, 2018, 01:18:31 PM
Quote from: Hakurei Reimu on August 28, 2018, 11:38:02 PM
Let me take a stab at trying to explain the error in your thinking, MT.
Will you have enough time?  I mean, the sun is going to burn out in just a few billion years... ;)
Title: Re: Time dilation, length contraction, Relativity and the Bible!
Post by: trdsf on August 30, 2018, 01:39:33 PM
Quote from: Hakurei Reimu on August 30, 2018, 12:16:53 PM
You seem to believe that physics swallowed Einstein's relativity whole and unexamined because Einstein was so briliant. This is not true. At the time, Einstein was working as a patent clerk, and was a nobody. Relativity is what made him. His theory of relativity was tested (along with quantum mechanics) throughout the first part of the 20th century, with very carefully designed experiments to test all aspects of it, and relativity passed all of them. It might be difficult to see how a particular experiment tests ideas like time dialation, but they do because all of the concepts of relativity are very intimately woven together.
it's worth noting that Einstein's Nobel prize wasn't for what he's usually remembered for: Special Relativity, mass/energy equivalence, or General Relativity.  It was for his explanation of the photoelectric effect, the first of the four papers of his Wunderjahr (https://en.wikipedia.org/wiki/Annus_Mirabilis_papers).  Although Eddington had provided some observational evidence for General Relativity by measuring the bending of starlight during the 1919 eclipse, it really was considered too theoretical in 1921.  They settled on the photoelectric effect paper as the basis for his Nobel (one of the founding papers of quantum mechanics, as it turned out) because everyone agreed he deserved the honor, but Relativity was still too "far out".

I have always considered it one of the Nobel Committee's great oversights that they never awarded him a second Physics prize for General Relativity.
Title: Re: Time dilation, length contraction, Relativity and the Bible!
Post by: Mousetrap on August 31, 2018, 02:33:07 AM
Quote from: HRConsulting the Earth and moon clock is not enough to tell you the duration along the path of the rocket; you must consider the distance traveled and use the proper time formula (which is the Minkowskian distance formula) to get the duration along the rocket's flight path (or just consult the shipboard clock, which gives a direct measurement of the rocket's flight time).
Which (the clock on the rocket that traveled to the moon) will be the one that ran for a second inside the rocket, but when it came to the moon, it seemed to have stood still for one second and everyone shook their heads because they dont understand SR, but those who does, understood that this was an natural observation.
HR, I can see where the interpretation of Lorentz and Mickelson Morley is incorrect.
To me it has nothing to do with time dilation, or length contraction, but everything to do with Observation of light.

I am very thankful for your contribution you made so far, due to me having to test a few ideas with someone who understands SR.
I am quite happy with what I collected, and I am finally ready to complete my findings on Time that can not change.

Greetings

Title: Re: Time dilation, length contraction, Relativity and the Bible!
Post by: Hakurei Reimu on August 31, 2018, 08:11:19 AM
Quote from: Mousetrap on August 31, 2018, 02:33:07 AM
Which (the clock on the rocket that traveled to the moon) will be the one that ran for a second inside the rocket,
No.
It doesn't.
That's the point.

The clock on the rocket does not run for a second inside the rocket. The rocket going at near c to the moon travels along a path that has a physically shorter proper time than the ones on the moon and the Earth. The same way that the diagonal line from one corner of a rectangle to the opposing corner is physically longer than either of the two sides, a path traveling at speed doesn't have as much physical time elapse between two points that the stationary clocks consider to be "now."

It's not a seeming, either. It's a physical measurement. Every means of determining time on that rocket will register that not as much time passed on it. The reason why those clocks don't measure more time is that there was no more time to measure. Short-lived radioactive particles do not show as much decay on the rocket during that time because there was no more time to expect more decay. Quartz clocks don't register as much time because their crystals didn't go through as many oscillations for the time keeping circuit to count. In our example, our passengers haven't even had a single heartbeat between departure and arrival.

The guys on the moon and the Earth don't even need to see that the clock on the rocket has slowed down during the trip to know that it has done so. They can see all the passengers come off with their wristwatches saying 12:00:00, while the moon clock reads 12:00:01. None of the clocks jump ahead a second when the rocket stops. They keep ticking at 1 second per second of elapsed time, but they are one second behind the Moon clocks and will remain one second behind the moon clocks until they reset their own clocks.

The reason why you don't see it in your ordinary life is because you don't travel very far compared to the time it takes you to travel that distance. Approximating the length of a diagonal of a rectangle by taking the length of its long side and saying that this is the length of the diagonal will work well enough when the rectangle is long and skinny, but not so well when the rectangle is more squat. The same thing occurs with the shipboard clocks compared to the Earth and Moon clocks; you're using the Earth and Moon clocks to approximate the rocket's elapsed time in exactly the way that you would use a side of a rectangle to estimate the length of its diagonal. It only works well with slow trips the same way that using the side only works well with long, skinny rectangles.

In order to calculate the time elapsed on the rocket, you have to use the proper time formula or use the onboard clocks to judge this. Both say that time dilation does happen.

Quote
I am quite happy with what I collected, and I am finally ready to complete my findings on Time that can not change.
Then you are in conflict with SR, and every experiment that has done so far that confirms it, and there are many.
Title: Re: Time dilation, length contraction, Relativity and the Bible!
Post by: Baruch on September 01, 2018, 09:44:43 AM
QM and RT don't make sense in classical physics terms, let alone in Biblical terms ...

Mouse, watch this .. you can't really do RT without QM ... but we can leave out gravity/acceleration for now ...

https://www.youtube.com/watch?v=OCuaBmAzqek part 1 of 3 ... not too hard to follow if you have college maths

Basically it is an abstract exercise ... which hardly touches on ordinary common sense experience, but it does match up with experiment.

Macro-studies of RT ... generally aren't done ... with the exception of RT + gravity.  After all, only subatomic particles can be accelerated to near light speed.

QFT ... QM for the EM field is quite a bit more complicated, here is a qualitative description ...

https://www.youtube.com/watch?v=oQ1WZ-eJW8Y part 4 of 7

Notice you end up with an infinite series of Feynman diagrams (or path integrals) ... this is the origin of the multiple universe interpretation of QFT.  The math is very hard, and the usual infinite series converge very slowly (bitch to calculate, you need a supercomputer).

In the later part of the second film, perturbation theory is mentioned.  Why leave out gravity .. because it messes up perturbation theory, and nobody has actually solved that problem, not Feynman, not Hawking, nobody.

Hawking was one of several people in the 1970s, who worked on the thermodynamics of black holes, and a hack was developed, not an actual theory.  And since we don't have any nearby black holes to study (thank G-d) ... it is rather hard to experimentally backup this hack.  It may be right, at least as a first approximation, but we don't know.  That is why Feynman got a Nobel, but Hawking didn't.  But at least Hawking was a real person, unlike Dr Who.

Here is the macroscopic GRT ... done by the same folks as the last film

https://www.youtube.com/watch?v=AwhKZ3fd9JA part 4 of 4

The practical effect of this is GPS that you use in your car or phone.  GRT is necessary to make this accurate, because time is modified by the gravitational field, and exact timing is necessary for accurate location finding.
Title: Re: Time dilation, length contraction, Relativity and the Bible!
Post by: Mousetrap on September 02, 2018, 03:50:16 AM
Quote from: Hakurei Reimu on August 31, 2018, 08:11:19 AM
...
Then you are in conflict with SR, and every experiment that has done so far that confirms it, and there are many.
.HR, I always find references to experiments done that proves SR.
But I never find any criticism or scrutiny on what was tested.
However, once I started questioning just one claim, (The claim that an observer inside a space ship will see his flashing light touch the Back and front of the space ship at the same time, travelling from the center), I realized that this is an assumption, and not fact.

I knew that even though we are travelling through space, the one thing that will be observed as a point in space, will be a flash of light.
You, or the whole universe, can travel at whatever speed, the exact point where light was flashed, will be its source of measurement for it's velocity.

Therefore, you can travel in a space ship, at half the speed of light, you will not measure light at c, but at 0.5c.
what will be observed is that the space ship traveled with the light from the lights origin, and light will measure c only from that point.

Now, to come up with non falsification tests on experiments such as "we tested it by atomic clocks on flights around the world, is simply a parrot speech with zero credibility.

I will give you a small bit of facts I already studied and collected.

Look at Dr. Borchardt on SR proven by around the world flights (http://thescientificworldview.blogspot.com/2011/02/time-dilation-and-hafele-and-keating.html)

Please note on the sloppy scientific work, and the assumptions made by SR scientists.

Luckily there are scientists who do scrutinize claims made by ones that want to become famous in being the first to prove SR true.
Title: Re: Time dilation, length contraction, Relativity and the Bible!
Post by: Mousetrap on September 02, 2018, 04:03:14 AM
Quote from: Baruch on September 01, 2018, 09:44:43 AM
QM and RT don't make sense in classical physics terms, let alone in Biblical terms ...

Mouse, watch this .. you can't really do RT without QM ... but we can leave out gravity/acceleration for now ...

...

The practical effect of this is GPS that you use in your car or phone.  GRT is necessary to make this accurate, because time is modified by the gravitational field, and exact timing is necessary for accurate location finding.

Well, I also tried to find any falsification testing on that claim (GPS).
I just could not found any.
However, there are very highly educated scientists who prove that making such claims that without SR and GR, our GPS would not work, are pure nonsense.
[qrl=https://medium.com/@GatotSoedarto/top-4-reasons-why-gps-doesnt-need-einstein-s-relativity-895cabc6e619]Gatot Soedarto, why GPS does not prove GR and SR[/url]
Title: Re: Time dilation, length contraction, Relativity and the Bible!
Post by: Mousetrap on September 02, 2018, 05:00:34 AM
 Ron Hatch proves GPS calculations will be 100% correct, only if the relativity calculations is removed! (https://www.gps.gov/governance/advisory/members/hatch/)

Oh, Hahurei Reimu.
It seems as if the 2 evidences for SR and GR, actually proves SR and GR is one big error!

Planes flying around the world with Cesium clocks, as well as Global positioning systems does not prove Relativity at all.
On the contrary, when there are scientists who would like to get the calculations correct, they realized....
Einstein was wrong, and he fed us BS!!!
Title: Re: Time dilation, length contraction, Relativity and the Bible!
Post by: Baruch on September 02, 2018, 08:45:08 AM
Quote from: Mousetrap on September 02, 2018, 04:03:14 AM
Well, I also tried to find any falsification testing on that claim (GPS).
I just could not found any.
However, there are very highly educated scientists who prove that making such claims that without SR and GR, our GPS would not work, are pure nonsense.
[qrl=https://medium.com/@GatotSoedarto/top-4-reasons-why-gps-doesnt-need-einstein-s-relativity-895cabc6e619]Gatot Soedarto, why GPS does not prove GR and SR[/url]

The usual educational descriptions of STR (standard relativity theory) are hoary pedantry ... of cable cars of Berne Switzerland, moving at near light speed past the coffee shop where Einstein took his breaks while a patent clerk ... Einstein's original thought experiment (or even earlier, moving at light speed, on a bicycle in N Italy before he started college).

But really, one should today start with actual experimental results ... which is basically just one item ... the decay rate of unstable particles such as muons varies as the Lorentz transformation equation for time indicates.  The other consequences are not experimentally observed, because moving rulers at near light speed has never been attempted ;-)

The other results are the logical consequence ... but rationality isn't real, just virtue signaling by autistic males ;-)

For QM, the first simple QT comes from just trying to figure out an incandescent light bulb (Planck) not even the photoelectric effect.  A regular light bulb doesn't behave according to Newton or Maxwell.  Almost everyone has experience with that quantum mechanics (but not experience with accelerated muon particles).

And many people have experience with GPS ... though don't appreciate how "parts per billion" accuracy is required to get sub-kilometer accuracy.  Not only satellites are required, but also atomic clocks.  One part in 40 million or 250 parts per billion to get 1 meter accuracy.

Advice ... don't listen to hacks and quacks.  And yes, the actual story of how these ideas developed, is "saving of appearances" and opponents of early 20th century physics had the same objection as the quacks you listen to.  But only lay people bother to listen to them.
Title: Re: Time dilation, length contraction, Relativity and the Bible!
Post by: Mousetrap on September 02, 2018, 10:23:15 AM
Quote from: Baruch on September 02, 2018, 08:45:08 AM
...

Advice ... don't listen to hacks and quacks.  And yes, the actual story of how these ideas developed, is "saving of appearances" and opponents
I think Ron Hatch can not be classified as a Quack.
I met him a few years ago when my company needed GPS locators on our farming equipment, and I was astounded at his personal practical experiments.

He actually informed fellow NASA scientists to drop any formulations of SR and GR in Space and Time due to the incorrect implementation thereof.
This guy can make a tractor run on a barren land, plowing with numerous others, without leaving the line with more than 15 mm over 20 Km.
I think he should be listened to.
Title: Re: Time dilation, length contraction, Relativity and the Bible!
Post by: Hakurei Reimu on September 02, 2018, 07:03:16 PM
Quote from: Mousetrap on September 02, 2018, 03:50:16 AM
.HR, I always find references to experiments done that proves SR.
Mousie, you seem not to understand that if SR is "proved," then the basic assumptions and the conclusions that naturally follow from those assumptions also follow.

Quote
However, once I started questioning just one claim, (The claim that an observer inside a space ship will see his flashing light touch the Back and front of the space ship at the same time, travelling from the center), I realized that this is an assumption, and not fact.
It is not an "assumption," it is a conclusion directly following from the principle of relativity and the constancy of the speed of light in all frames. If you contest the former, then you need to find some experiment that gives you different answers depending on the speed of the aparatus. If you contest the latter, then you need to find some circumstance where light will be measured going at any other speed but c in a vacuum. If you have neither, you don't have a leg to stand on to challenge the above claim.

Quote
I knew that even though we are travelling through space, the one thing that will be observed as a point in space, will be a flash of light.
You, or the whole universe, can travel at whatever speed, the exact point where light was flashed, will be its source of measurement for it's velocity.
I can measure the velocity of the light just fine using my own instruments. And that's the point. My instruments are physical aparatuses, using the laws of physics to deduce things like the velocity of light. If they measure the speed of light to be c, then it makes those measurements based on the application of physical law and data â€" the light is really traveling at c.

Quote
Therefore, you can travel in a space ship, at half the speed of light, you will not measure light at c, but at 0.5c.
If that were true, then Michelson and Morley would have detected the velocity of Earth as it orbited the sun at least one point along its orbit. They didn't, even though their equipment was precise enough to detect such a difference. That destroys your claim.

Quote
what will be observed is that the space ship traveled with the light from the lights origin, and light will measure c only from that point.
Who observes this? You're pretending with the clause "what will be observed" is that there's some absolute determination of the light's speed. Such things are always measured by instruments â€"physical objectsâ€" which have their own frame of motion. So "what will be observed" is nonsense unless it is understood who or what is making the observation.

Quote
Now, to come up with non falsification tests on experiments such as "we tested it by atomic clocks on flights around the world, is simply a parrot speech with zero credibility.
You pretend that this was the only "proof" I gave you. You only have to look up "Tests of Special relativity" on Wikipedia to come up with a slew of experiments and families of experiments that demonstrate, measure and verify every major prediction of Einstein, and so far, Einstein has passed all tests.

I for one gave you also at least one other proof of time dilation in particular, that of the fact that a particular family of subatomic particles created in our atmosphere were surviving longer than they should. You have yet to answer that.

You seem to think that the H-K experiment is the only test of time dilation. We have a whole list of them, from accelerating subatomic particles and watching for doppler shift that is not that of classical doppler shift, from noting that unstable muons created in the atmosphere survive far longer than they should without time dilation, to the simple observation that the Standard Model would fall into complete shambles without SR and all of its implications (including time dilation and length contraction) at its core.

Quote
I will give you a small bit of facts I already studied and collected.

Look at Dr. Borchardt on SR proven by around the world flights (http://thescientificworldview.blogspot.com/2011/02/time-dilation-and-hafele-and-keating.html)

Please note on the sloppy scientific work, and the assumptions made by SR scientists.

Luckily there are scientists who do scrutinize claims made by ones that want to become famous in being the first to prove SR true.
Bullshit. The Standard Model is currently the most accurately scrutinized and measured model of particle physics and it has as its core assumption the veracity of SR and all the time and space wierdness coming along with it. The amount the phase of a particle advances as it goes from point to point is based on the concept in SR called the interval, and from there everything in relativity follows. Knowing how particles propagate through the system is a key calculation in the Standard Model, and if this is wrong, then no prediction of the Standard Model would play out.

If the core of our model is wrong, then we are unreasonably lucky as the Standard Model has not only passed all tests that it should fail if SR is wrong in its implications, such as time dilation, but also the fact that we've based our entire technology base on it. The continued well-functioning of computers and other devices that owe their very existence to the veriacity of the Standard Model is continued testiment to the correctness of SR.

The original Hafele and Keating experiment is more demonstration than it is serious experiment. Your source points out that the drifts are all over the place, but like he himself said, they can be corrected for. And when we extrapolate those drifts, we find that at the end of the flights there are unexplained jumps in where the clocks should be at given classical physics. Now, yes, not all showed the same offset or any at all, but that's why there were three of them. After all, the drift reveals that those clocks can be at times unreliable due to random reasons.

Your Dr. Borchardt charges that "of my 320 publications, only one was repeated by others in any detail." In the case of Hafele and Keating, we know that this is incorrect. The University of Maryland reproduced it more preciesely through the period of Sept 1975 thru Jan 1976, and this time (unlike H-K) results were available real-time, where the shift due to relativistic effects could be watched by those on board the plane and on the ground. The upshot is that this experiment was in line with predictions of SR and GR to 1.6%. The H-K experiment was repeated in 1996 with more accurate atomic clocks by the National Physics Laboratory, on a round trip from London to Washington, D.C. to London, giving a time gain of 39±2 ns (39.8 ns predicted), and again in 2010 with a round-the-world flight that gave a measurement of 230±20 ns (246±3 ns predicted). So, I can confidently mark this claim busted; H-K was replicated and vindicated. Even if the original was crude and poor, it was probative enough to be replicated more carefully.

As to other experiments, I similarly charge that your Dr. Borchardt has his head up his ass. An experiment doesn't have to be "repeated in detail" to be replicated. Indeed, such precise replications are of limited worth, as mere repeats are prone to the same systematic errors as the original, a fact that limits their usefulness. Testing the same phenomenon using a different method is much preferred, because they would have different systematic errors, and the Anna Karenina principle would ensure that errors would be all over the place in different directions. It is unlikely that a phenomenon detected by multiple, disparate methods is nonexistent.

Quote from: Mousetrap on September 02, 2018, 10:23:15 AM
I think Ron Hatch can not be classified as a Quack.
I met him a few years ago when my company needed GPS locators on our farming equipment, and I was astounded at his personal practical experiments.
So what if you were impressed? As far as I can determine, Ron Hatch has no presence at all in his impact factor. That means none of his papers has had very many citations â€" which is a sign more that he really doesn't have anything interesting to say, rather than he is some kind of persecuted scientist with the truth on his side.

Quote
He actually informed fellow NASA scientists to drop any formulations of SR and GR in Space and Time due to the incorrect implementation thereof.
So? I'm certain that Hatch did inform NASA. That doesn't mean that he'd been heeded. To my knowledge, none of the implementations have been changed on his account.

See, the drift of the GPS satellites is something you can measure. It's an 'experiment' of sorts that has been in the running for well over forty years. If the effects of SR and GR weren't real, then it would have been plainly obvious by now.

Quote
This guy can make a tractor run on a barren land, plowing with numerous others, without leaving the line with more than 15 mm over 20 Km.
I think he should be listened to.
Pfft. Even if I were to believe that (how did he figure out that it was only 15 mm, and how did he figure out he really plowed 20 km?), that figure is only one part in just over a million. That's child's play. If he were to calculate the g-factor of the electron to one part in 2.6e-13 (a lot harder to do) without appealing to SR, then he'd be talking.

I also notice a definite lack of calculations on your part. For instance, the relativistic doppler effect necessitates time dilation.

Let's go back to our rocket example. Instead of an almost-c supership, let's consider a half-c semisupership. Of course, it takes two seconds Earth-Moon time to travel this distance. Naively, we suppose that if we place a strobe light that flashes every half-second and set it to strobe the first time upon launch, then this strobe will flash five times along the trip, once at launch, three times en route, and once as it arrives. The frequency of the flashes is f = 2 Hz.

The Doppler effect for light is verified by experiments to be

   f′/f = sqrt(c-v/c+v)

where f is the frequency at the source, f′ is the frequency observed, and v is the relative velocity of the source away from the observer.

First, the Moon's perspective. Since our rocket has velocity of v = -½c (approaching at half-c), we have f′/f = sqrt(c+½c/c-½c) = sqrt(1+½/1-½) = √3 = 1.732050808. So, at half the speed of light, the frequency has increased by a factor of √3 = 1.732050808, and it seems like that rather than time slowing down on the rocket, time has sped up instead. And so Einstein's notion of time dilation is faâ€"

Not so fast. This is just the frequency the incoming flashes are arriving at our observer on the Moon. If there were no time dilation, then we'd expect the rocket to emit a total of five flashes over the course of its two second trip, the first being just as it launches from the Earth, and the last being just as it arrives on the Moon. Since it takes light one second to get from Earth to the Moon, this first flash arrives at 12:00:01, one second after the rocket launches. Of course, the fifth and last flash would have to arrive just as the rocket arrives on the Moon, at 12:00:02. So, all of the flashes have to arrive within that 1 second window between the first flash and the last flash. But this is half the time that the rocket took to emit those same five flashes, so were we to observe five flashes between 12:00:01 and 12:00:02 as we expect from the classical case, f′/f = 2 > √3.

So, although the flashes are coming in faster than the two per second of the strobe as expected, they are not coming in quite fast enough for classical physics to explain. Indeed, the rocket only gets off four flashes before it arrives. But if the rocket only emitted four flashes in a time when it should have emitted five, then its strobe must have been flashing slower than... expected.

Oh dear.

Let's consider the duration between the first and second flashes, the one emitted at launch and the one immediately following. This first flash was emitted at x = 0 at time t = 0 in our familiar S, the Earth-Moon frame. This flash was detected on the moon at x = 1 ls at time t = 1 s in S, on the Moon. Its successor flash was recieved 1/f′ = 1/(f√3) = (1 s)/(2√3) = 0.2886751346 second later, or at t = 1.2886751346 s. The world line of this flash of light is described by x = c(t - 1/f′). The rocket's world line is described by x = ½ct. Equating the two and solving for t, we get

   Â½ct = c(t - 1/f′)
      â‡"
   t = 2/f′ = (1 s)/√3
     = 0.5773502692 s > 0.5 s

So the shipboard strobe emits its second flash about 0.07735 s later than it should have if its clocks are ticking at the same speed as they were in S. But of course, we set the strobe to flash at 0.5 second intervals, so when the shipboard clock reads 12:00:00.5, it emits a flash of light. So, at the same time the S clocks read 12:00:00.5773502692, the shipboard clock reads 12:00:00.5. According to the Earth-Moon frame, S, the shipboard clock is running slower than the Earth-Moon clocks by a factor of 1.154700538, and if we equate this to 1/sqrt(1-v²/c²) and solve for v we get... v = (0.5)c! Of course, this is the time dilation that SR predicts.

While the strobe does indeed appear to speed up from the Moon's point of view, it does not speed up enough to compensate for a mere apparent time dilation you think is the true picture. Instead, locates the actual event (coordinates in space and time) whence the flash was emitted in that frame of reference, and it emitted the flash late as if it had been slowed down. The ship emitted its second flash 0.07735 s late, even though we set it up to be emitted at precisely 0.5 s into the flight.
Title: Re: Time dilation, length contraction, Relativity and the Bible!
Post by: trdsf on September 02, 2018, 11:14:28 PM
Mousie, I haven't taken you off ignore, but it's pretty clear what you're trying to say, via Hakurei's replies, and I am morally obliged by my commitment to truth and reality to respond, even if I couldn't conceivably care less how or whether you reply.

It really is so simple that even you can grasp it: the whole point of Einsteinian relativity is that no matter where you are and how you are moving, you always observe the speed of light to be constant, and it is always measured as 299,792,458 meters per second in a vacuum without any regard to how the observer is moving.  If your spaceship is moving at 299,792,457 meters per second, you will still measure light to be moving at 299,792,458 meters per second in a vacuum.  No one nowhere at no time has ever observed anything other than that.  That's all there is to it.  No, you don't have any evidence to the contrary, and neither does anyone else.

If anyone ever does come up with something that doesn't fit Einstein's predictions, it's going to be well attested, confirmable, and probably out of one of the big institutes like CERN or Fermilab.  I don't care what Ron Hatch has to say, regardless of what he's done with GPS (which, by the bye, depends fundamentally on Special and General Relativity in order to work) -- Newton believed in alchemy, that didn't make alchemy correct.  This is an 'appeal to authority' fallacy.  Although given your regular misuse of the strawman fallacy, I have little hope that you understand that.
Title: Re: Time dilation, length contraction, Relativity and the Bible!
Post by: Mousetrap on September 03, 2018, 01:53:27 AM
Quote from: Mousetrapon September 02, 2018, 10:23:15 AM
I think Ron Hatch can not be classified as a Quack.
I met him a few years ago when my company needed GPS locators on our farming equipment, and I was astounded at his personal practical experiments.
Quote from: Hahurei ReimuSo what if you were impressed? As far as I can determine, Ron Hatch has no presence at all in his impact factor. That means none of his papers has had very many citations â€" which is a sign more that he really doesn't have anything interesting to say, rather than he is some kind of persecuted scientist with the truth on his side.
Persecuted?
Not at all.
He does not care what the rest of the world does.
one of his patents are a GPS receiver with added calculations removing GR and SR.
He's making money on the accuracy of his inventions.
Furthermore,
Why do you think someone is wrong if they actually disprove Einsteins' theory when they have workable and operational evidence?
You go on the premise that supporters of Special and General relativity should judge someone who produced evidence that it is wrong.

It is as if you ask Pelosi and Waters to decide if Trump is correct in his economical reforms.
Pal, I suggest you get out of your brainwashed comfort zone, and question what others are telling you.
Or, at least go and find out for yourself.
Those peers who you run to to tell you what the truth is, will never agree with anything about erroneous Relativistic calculations.
Why?
Because, it will change a big part of scientific BS where they can get billions to build more accelerators.
Money!!!
Grants!!!
Quote from: Mousetrap
He actually informed fellow NASA scientists to drop any formulations of SR and GR in Space and Time due to the incorrect implementation thereof.
Quote from: Hahurei ReimuSo? I'm certain that Hatch did inform NASA. That doesn't mean that he'd been heeded. To my knowledge, none of the implementations have been changed on his account.
Yeh! Go and ask Pelosi and Waters.

Money!!!
Grants!!!

Quote from: Hahurei ReimuSee, the drift of the GPS satellites is something you can measure. It's an 'experiment' of sorts that has been in the running for well over forty years. If the effects of SR and GR weren't real, then it would have been plainly obvious by now.
No one spoke about drifts HR.

Title: Re: Time dilation, length contraction, Relativity and the Bible!
Post by: Mousetrap on September 03, 2018, 02:15:04 AM
Quote from: trdsfIf anyone ever does come up with something that doesn't fit Einstein's predictions, it's going to be well attested, confirmable, and probably out of one of the big institutes like CERN or Fermilab.  I don't care what Ron Hatch has to say, regardless of what he's done with GPS (which, by the bye, depends fundamentally on Special and General Relativity in order to work) -- Newton believed in alchemy, that didn't make alchemy correct.  This is an 'appeal to authority' fallacy.  Although given your regular misuse of the strawman fallacy, I have little hope that you understand that.

And so did Hatch.
He came up with attested, confirmed, and unfortunately not out of one of the big institutes like CERN or Fermilab, experiments, but with his GPS systems.

But what do you think the scientists at Cern and Fermilab is doing when they learn their future funding will be jeopardized and they have to go back to the drawing board to get another explanation on the origins of Matter, space and Time.

They will have to work on trucks and tractors.
And that sounds too much like work to them.
:)





Title: Re: Time dilation, length contraction, Relativity and the Bible!
Post by: Mousetrap on September 03, 2018, 02:29:58 AM
Quote from: trdsf on September 02, 2018, 11:14:28 PM
Mousie, ... -- Newton believed in alchemy, that didn't make alchemy correct.  This is an 'appeal to authority' fallacy.  Although given your regular misuse of the strawman fallacy, I have little hope that you understand that.
Wow, this is a hugely destorted view of Newton and alchemy.
Newton did believe in alchemy, ONLY UNTIL HE DISPROVED IT AND FOUND EVIDENCE THAT ALCHEMY IS A WASTE OF TIME!!!!

If Newton would be able to check Einstein's relativity claims of Time dilation, and Length contraction, he will remove this alchemy also.

I love it that you would build a Straw man, the Newton alchemist one, and then tell me I did it.
Title: Re: Time dilation, length contraction, Relativity and the Bible!
Post by: aitm on September 03, 2018, 01:29:28 PM
 

Yet she increased her prostitution, remembering the days of her youth when she engaged in prostitution in the land of Egypt. She lusted after their genitals as large as those of donkeys, and their seminal emission was as strong as that of stallions.

Ezekial 23:19-20

See, now THAT is biblical dilation and length contraction all in one. But then the babble with all its god "science" is full of such wonderful head scratchers. I mean.....why do we allow handicapped..mentally and physical into churches when daddy god forbides it.....or the poor guy what lost his nuts to cancer or a hungry hooker. Meh.......lets ignore the nonsense of the babbling babble and use science to prove that this idiot actually exists. Boo-yah.
Title: Re: Time dilation, length contraction, Relativity and the Bible!
Post by: Hydra009 on September 03, 2018, 07:50:03 PM
Quote from: Mousetrap on September 03, 2018, 02:29:58 AMI love it that you would build a Straw man, the Newton alchemist one, and then tell me I did it.
Newspaper headline when Mousie was born:  scientists create first analogy-proof human.
Title: Re: Time dilation, length contraction, Relativity and the Bible!
Post by: Hydra009 on September 03, 2018, 07:54:00 PM
Quote from: Mousetrap on September 03, 2018, 02:15:04 AMBut what do you think the scientists at Cern and Fermilab is doing when they learn their future funding will be jeopardized and they have to go back to the drawing board to get another explanation on the origins of Matter, space and Time.

They will have to work on trucks and tractors.
And that sounds too much like work to them.
:)
Soo...the researachers at CERN and Fermilab are concealing the truth that special relativity is bogus because otherwise they'd have to do automotive repair?  That's the narrative you're going with?

Do you even reality, bro?
Title: Re: Time dilation, length contraction, Relativity and the Bible!
Post by: Baruch on September 03, 2018, 08:25:46 PM
It is how the story is told, and how it is analogized to non-physics subjects ...

https://www.youtube.com/watch?v=Y-AiqCp7Vlc

An otherwise good documentary, ruined by materialist atheism.  Fact is, atoms aren't atomic, because nuclei and elementary particles, which are then reduced to quantum fields, which are then attempted to be reduced to superstrings.  A good documentary sticks to the experiments, and doesn't preach.  Materialism, is a simplifying abstraction made by humans or other sentients.  Plato will claim that this is absolute truth, found by secular prophets.  Except it isn't absolute, there are multiple definitions of truth in English.  And I am just as skeptical of secular prophets as I am of the theistic kind.

Just how mass-energy relates to space-time, is still controversial.  And the idea that space-time isn't continuous, but is itself quantized (as opposed to mass-energy) is also controversial.  Which is why the relationship between the two pragmatic metaphors ... space-time and mass-energy ... is still partially understood.
Title: Re: Time dilation, length contraction, Relativity and the Bible!
Post by: Hakurei Reimu on September 03, 2018, 09:08:07 PM
Quote from: Mousetrap on September 03, 2018, 01:53:27 AM
Persecuted?
Not at all.
He does not care what the rest of the world does.
one of his patents are a GPS receiver with added calculations removing GR and SR.
Dude, the receiver doesn't use either. The assumptions of GR and SR are built into the satellites' atomic clocks â€" they are built to tick slower than they would on the ground to compensate for both the effects of gravitational redshift and time dilation from the satellite's orbital speed. On balance, the speedup from being higher in the Earth's gravity well is the bigger effect than the slowdown from the satellite's orbital speed. There's nothing to remove.

Quote
He's making money on the accuracy of his inventions.
Furthermore,
Why do you think someone is wrong if they actually disprove Einsteins' theory when they have workable and operational evidence?
Because that's exactly the place you find yourself. Our theories of how the universe works is based on Einstein's relativity as its cornerstone. Quantum field theory couldn't work without it, and QFT is the best description of particles moving under the electromagnetic force bar none. QFT is what enables you to build all those fancy new electronic gadgets you buy in the store, and at the core of it is the assumption of special relativity. People are making money off of SR, too.

Only your hero doesn't require specific knowledge of GR and SR to make his devices work. In fact, the only thing he would need is differential GPS in order to get results that good, and while his implementation may be really nifty to produce figures that good, it's not a new trick.

Quote
You go on the premise that supporters of Special and General relativity should judge someone who produced evidence that it is wrong.
What evidence? You haven't presented any "evidence." You've produced a claim that some dude is making money off a device that supposedly "adds" calculations to remove GR and SR when the whole system is already compensated such that ground-based devices don't need either to produce their answers. Don't pretend you know this stuff better than me.

Quote
It is as if you ask Pelosi and Waters to decide if Trump is correct in his economical reforms.
Pal, I suggest you get out of your brainwashed comfort zone, and question what others are telling you.
You first. You're the one who seems to take the words of the bible, the words of men, over the very universe your god supposedly created. You need to start listen to what the universe is telling you, not what the bible is telling you.

And you are hilarious when you talk about brainwashing and comfort zones, when you haven't shown me a single calculation of your own to prove that what I'm saying is wrong, using your own understanding and knowledge, and instead appealing to people who are clearly cranks. It's as if you don't want to really learn about special relativity in fear that it will break your little fantasy of yours.

For every paper your boys produce against relativity, I guarantee you that ten thousand other papers have been published that use relativity as one of its core assumptions, and get their predictions right. This is unreasonable success for a theory that is supposedly wrong.

Quote
Or, at least go and find out for yourself.
You have so far produced such a poor case for your side that I sincerely don't believe that "finding out for myself" is worth a single second of my time. It's also symptomatic of someone who doesn't really understand what they're talking about. You rely on others' understanding rather than your own. You might as well have the word "sucker" tattooed on your forehead.

Quote
Those peers who you run to to tell you what the truth is, will never agree with anything about erroneous Relativistic calculations.
Why?
Because, it will change a big part of scientific BS where they can get billions to build more accelerators.
Money!!!
Grants!!!
Those accelerators couldn't even WORK without relativity, you moron. You don't get to TeV-level energies without noticing that your particles do not travel at many times the speed of light (which they would under classical mechanics). That's going to make a big difference in the timing of your circuits. Without mass-energy equivalence, another relativistic prediction, you wouldn't get the shower and an entire bestiary of particles which all have rest masses of more than the particles originally smashed together.

How does your classical mechanics explain that?

I'll give you the short answer: it can't.

And an accelerator's budget is peanuts compared to the fruits of QFT, which gives you stuff like computers and microelectronics. Do you not think that microelectronics are a multi-billion dollar industry? Naw, QFT produces money.

Classical physics was abandoned grudgingly by the scientific community. There were many resisters â€"hell, by your account, there still areâ€" but none of them could find purchase because evidence kept mounting that when dealing with high velocities, only Einstein gets it right. And it's not as if the full edifice was Einstein's alone. Einstein himself built on the work of Poincare. Schwartzchild first described the gravitational field of a spherical body. Physicists take the theories and make it their own.

Quote
Yeh! Go and ask Pelosi and Waters.

Money!!!
Grants!!!
Yes, NASA tends to give grants to shit that works, not to crazy twits who go against the mainstream counteracting well-established principles like relativity.

Let me give you a hint on how science progresses and how big changes like quantum mechanics and relativity occur. Usually, old paradigms get replaced with something completely new and never seen before. New paradigms never get replaced by what had worked before but had fallen out of favor due to new evidence. If there is going to be a successor to Einstein, it's not going to look like anything you've ever seen before. Classical physics was dropped precisely because it doesn't give you the right answers for the very fast, the very small, or in very strong gravity fields. It still doesn't. The replacement for SR and Gr and QM are going to be wierder than all of them.

Quote
No one spoke about drifts HR.
Oh? Does not a clock that is running faster than another clock drift out of synch with that clock? That's what we're talking about. That is the continuous experiment. If GR and SR were wrong, then the GPS satellites' atomic clocks would drift out of synch with Earth time. The workings of GPS depend on those clocks being extremely accurate.
Title: Re: Time dilation, length contraction, Relativity and the Bible!
Post by: Hakurei Reimu on September 03, 2018, 09:34:18 PM
I like it how Mousie doesn't seem to know how grants work. The money for a grant doesn't go to the scientist. It goes towards the experiment. In a grant, you have to provide an accounting of where the money goes. Unless there is an item that says "stipend," which would be kept to reasonable levels, none of the money goes to the scientist. It all goes towards lab equipment, materials, etc. The money outlaid for the accelerators went mostly towards building the accelerators. Most of the remainder goes towards running the accelerators. Most of the grants used for the accelerators goes towards paying for time on the accelerators. When the grant runs out of money, what you're left with is shiny new lab equipment, a series of papers, and maybe a pittance stipend.

Ask any scientist who has actually had a grand proposal granted, and he'll tell you you don't make any real money on grants.
Title: Re: Time dilation, length contraction, Relativity and the Bible!
Post by: Hydra009 on September 03, 2018, 10:35:35 PM
But...but grants are such a convenient propagandists' stump.  It lets cranks cast doubt on the scientific consensus by claiming that their conclusions are driven by money.

NASA scientists are paid to say that the Earth is a sphere.
Climate scientists are paid to say that global warming is real.
Physicists are paid to say that special relativity is real.

You can cast doubt on anything by saying the magic word:  grants!
Title: Re: Time dilation, length contraction, Relativity and the Bible!
Post by: Mousetrap on September 04, 2018, 02:41:06 AM
Quote from: aitm on September 03, 2018, 01:29:28 PM


Yet she increased her prostitution, remembering the days of her youth when she engaged in prostitution in the land of Egypt. She lusted after their genitals as large as those of donkeys, and their seminal emission was as strong as that of stallions.

Ezekial 23:19-20

See, now THAT is biblical dilation and length contraction all in one. But then the babble with all its god "science" is full of such wonderful head scratchers. I mean.....why do we allow handicapped..mentally and physical into churches when daddy god forbides it.....or the poor guy what lost his nuts to cancer or a hungry hooker. Meh.......lets ignore the nonsense of the babbling babble and use science to prove that this idiot actually exists. Boo-yah.
And your post displays your ignorance on what the Bible says.
You see, this is why I will never become atheist again.

You had to go to your Atheist websites (Annotated Bible for instance) to get hold of something from the Bible which you think is vulgar and I should be ashamed of.

Well, lets see how you fare with this strategy.

The verses that you refer to is YHWH telling Judah exactly what the 10 tribes of Israel did.
They whored after the religions of Moloch and Ba'aal.
These religions are sex cults, if you dont know, and it was customary for these worshipers to burn their eldest child in fire as a human sacrifice to Moloch.
Furthermore, once a year the women worshipers had to engage in temple prostitution for a full day. No matter if she was married or not.
They were sexually used by the male crowds in huge sex orgies.

Now, from your pompous hypocrite mind, do you have a better way of describing this scenario than what YHWH did when he warned Judah that He will also sent Judah into captivity for doing the exact thing?

And guess what!
YHWH did exactly what He promised.
You should see the archaeological evidence showing how Shalmanezzer and Nebuchadnezzar took these cities in Israel and Judah. The people were impaled in their thousands, beheaded, flayed alive, and the reminants were taken away as slaves.

I expect God to be a truthful being, that calls a spade a spade, and not some watered down sexual lustful fairy you would like him to be.

Now, lets see what Atheism holds dear about sex and values of morality.
Nietzsche and Bertrant Russel was clear about the following.
There is no difference between animal and man.
in Atheism, the strongest are the winner. Women are there as possessions for men to ensure that the race survives.
Sex should be free of religious views where marriage does not exist and adultery is natural.
If a lion clears his territory of rivals, and he kills off cubs not his, collects all the females, this is nature.

If a man walk down the street and gets rid of all his male neighbors, take the wives, kills of the offspring of the rivals, he is also doing what is natural.
Why should he not do this?
This will ensure that the human race gets the strongest and cleverest genes to ensure its survival.
This was exactly what Hitler wanted to practice after the studies he made on Nietzsche.
Go and find out what the Nazis did to children born with syndromes.
Pure Atheistic practices pal!

no child born with defects will be allowed to live, such weaknesses should die!

CS Lewis, upon thinking about this, realized that this was the reason that God exists.
Sin!
if we as a human mind with a conciseness, knows such behavior is wrong, what prohibits us to live that way anyhow.
If a world is filled with actions to allow the fittest to survive, why not live atheism to the fullest?

Lewis realized that just by realizing something is immoral and wrong, we have FREE CHOICE!

Free choice can only be ascribed to a Creator allowing such, for if it is up to men, nothing can be wrong.

Anyhow, you did not read the Bible, and you did not know what the hell YHWH was telling you.
Now you know!

Title: Re: Time dilation, length contraction, Relativity and the Bible!
Post by: Mousetrap on September 04, 2018, 03:08:40 AM
Quote from: Hydra009 on September 03, 2018, 07:54:00 PM
Soo...the researachers at CERN and Fermilab are concealing the truth that special relativity is bogus because otherwise they'd have to do automotive repair?  That's the narrative you're going with?

Do you even reality, bro?
And you continue to hang onto scientists which you prefer.
And ignore those they dont.
Do you even know who Ron Hatch is?
have any scientist at NASA and CERN looked at what he does?
No?
Why?
Perhaps we are back in the dark ages where the scientists of the world wanted to burn Copernicus for the true events in nature, and not because what they believed was true.
Title: Re: Time dilation, length contraction, Relativity and the Bible!
Post by: Mousetrap on September 04, 2018, 03:13:18 AM
Quote from: Hakurei Remu
You go on the premise that supporters of Special and General relativity should judge someone who produced evidence that it is wrong.
What evidence? You haven't presented any "evidence." You've produced a claim that some dude is making money off a device that supposedly "adds" calculations to remove GR and SR when the whole system is already compensated such that ground-based devices don't need either to produce their answers. Don't pretend you know this stuff better than me.
But I dont.
I also do not have to produce any evidence anymore that SR is wrong when we have a much better scientist who already done it.
Title: Re: Time dilation, length contraction, Relativity and the Bible!
Post by: Mousetrap on September 04, 2018, 03:25:21 AM
Quote from: HR
Quote from: MTIt is as if you ask Pelosi and Waters to decide if Trump is correct in his economical reforms.
Pal, I suggest you get out of your brainwashed comfort zone, and question what others are telling you.
You first. You're the one who seems to take the words of the bible, the words of men, over the very universe your god supposedly created. You need to start listen to what the universe is telling you, not what the bible is telling you.

My dear HR,
If you think for one moment that Nature is in contrast with the Creator, or that only one of them can be true, it is you that are making the biggest apologetic error for atheism's existence.
All I did was to discover that not only did the Bible lay the foundation of all scientific discoveries in the last 500 years, but that it was available for over 3000 years.

I also learned that what atheists regard as the evidence against a Creator, was actually evidence they plagiarized from the Bible in the first instance from Newton and Kant!

It will for ever be a fallacy that atheists are somehow busy discovering the origins of the Universe.
they are attempting to find something that can disprove the need for a Creator of everything, but every time they do come up with some sop story, They create this hydra with 7 heads that can not be destroyed due to per pressure.

Anyhow, you can believe in Length contraction and Time dilation.
No problem.
Science will just overtake you and man will become more informed, and I know it will not be long before Einstein is proven incorrect, and SR abd GR gets posted to the past with Time dilation.


Title: Re: Time dilation, length contraction, Relativity and the Bible!
Post by: Jason78 on September 04, 2018, 03:43:56 AM
Quote from: Mousetrap on September 04, 2018, 03:25:21 AM
It will for ever be a fallacy that atheists are somehow busy discovering the origins of the Universe.
they are attempting to find something that can disprove the need for a Creator of everything, but every time they do come up with some sop story, They create this hydra with 7 heads that can not be destroyed due to per pressure.

Then where exactly did your creator(s) do their work?  We've got a very good idea of what the early universe must have been like to produce the phenomena we see today and none of it requires the extra step of a creator.

Quote from: Mousetrap on September 04, 2018, 03:25:21 AM
Anyhow, you can believe in Length contraction and Time dilation.

You don't need to believe in it.  It's born out by every experiment.   
Title: Re: Time dilation, length contraction, Relativity and the Bible!
Post by: Baruch on September 04, 2018, 04:14:43 AM
Quote from: Hydra009 on September 03, 2018, 10:35:35 PM
But...but grants are such a convenient propagandists' stump.  It lets cranks cast doubt on the scientific consensus by claiming that their conclusions are driven by money.

NASA scientists are paid to say that the Earth is a sphere.
Climate scientists are paid to say that global warming is real.
Physicists are paid to say that special relativity is real.

You can cast doubt on anything by saying the magic word:  grants!

Who is buried in grants tomb?  The guy on the gassy knoll.
Title: Re: Time dilation, length contraction, Relativity and the Bible!
Post by: Mousetrap on September 04, 2018, 05:26:16 AM
Quote from: Jason78 on September 04, 2018, 03:43:56 AM
Then where exactly did your creator(s) do their work?  We've got a very good idea of what the early universe must have been like to produce the phenomena we see today and none of it requires the extra step of a creator.

You don't need to believe in it.  It's born out by every experiment.
Hey Jason 78.
I was waiting for you my whole life mate.
So, how did the universe look like to produce a phenomena we see today.

Wait, Wait Wait!!!
I know how it looked!

Like nothing!

It was nothing!

So, how did we get from nothing to something?

When did science find out that everything came from nothing?
Who was first with this scientific claim?

Guess?
Dont know?

Mmmmmm...
I know.
Title: Re: Time dilation, length contraction, Relativity and the Bible!
Post by: trdsf on September 04, 2018, 06:25:21 AM
Quote from: Hydra009 on September 03, 2018, 07:54:00 PM
Soo...the researachers at CERN and Fermilab are concealing the truth that special relativity is bogus because otherwise they'd have to do automotive repair?  That's the narrative you're going with?

Do you even reality, bro?
Reality is one of the Seven Dirty Words (https://en.wikipedia.org/wiki/Seven_dirty_words) in Mouseworld.  Reality, Logic, Reason, Sense, Repeatability, Mathematics and Tits... er, Data.
Title: Re: Time dilation, length contraction, Relativity and the Bible!
Post by: Unbeliever on September 04, 2018, 01:27:49 PM
"I found a solution, but it only works for spherical chickens in a vacuum." ;-)
Title: Re: Time dilation, length contraction, Relativity and the Bible!
Post by: Hakurei Reimu on September 04, 2018, 11:15:16 PM
Quote from: Mousetrap on September 04, 2018, 03:13:18 AM
But I dont.
I also do not have to produce any evidence anymore that SR is wrong when we have a much better scientist who already done it.
What makes you think that your scientist is "better?" Seriously, the only thing I've seen you claim for your boy is a feat that is a mere one order of magnitude improvement of a technique in common use for fine positioning when selective availability was in force.

Furthermore, your boy has an accuracy of only one part in 1.333 million, while the GPS satellites, with an orbit of ~20,000 km and can locate you within a 4.8 m sphere, have an accuracy of one part in ~4 million. That's why I called it "child's play."

Not only that, your "scientist" didn't tell NASA anything, much less that GR and SR need to be removed from GPS, because if he did, NASA would have told him that they didn't have anything to do with managing the GPS constellation and tell him to address the Pentagon. That's right. GPS is a military asset, which your scientist would have been clued in on if he had actually done what you claim he'd done. But he didn't because your story, that your scientist "told NASA" anything about GPS, is a lie.

In other words, your own story paints your "scientist" as a con man. You don't need a working device to run a con.

I also note that you have no comment that all the corrections to GPS are applied satellite-side. The relativity corrections are applied to keep the GPS clocks in synch to a similar atomic clock on the ground. There's no need to bother the GPS receiver with that, and as such, a GPS receiver can (and does) treat the signals it receives as if it were in the classical world, without taking into account SR and GR, because those corrections have already been applied.

No, no. Your boy "removing" the SR and GR calculations don't prove jack shit. They're not there to remove.

Quote from: Mousetrap on September 04, 2018, 03:25:21 AM
My dear HR,
If you think for one moment that Nature is in contrast with the Creator, or that only one of them can be true, it is you that are making the biggest apologetic error for atheism's existence.
What I'm saying, you moron, is that if you really believed that the Creator created the universe, then it is simply insane to believe that the Bible should be believed over the observations of said universe.

After all, the universe is a lot harder for man to rewrite than the Bible.

The only reason you are trying to fit the universe to the bible is because the universe is at odds with what the bible says, and you are trying to hold onto your favorite book of fairy tales. If you really were serious about figuring out what God had in mind in creating the universe, scientific conclusions being at odds with the Bible wouldn't bother you in the slightest, because it is the scientific conclusions that are exhaustively tested against the very universe you think is the purest expression of God's will.

Quote
All I did was to discover that not only did the Bible lay the foundation of all scientific discoveries in the last 500 years, but that it was available for over 3000 years.
Any book that claims to be the end-all and be-all of knowledge is not the "foundation" of science. Science is primarily a method of finding things out, by testing your ideas against reality, not some book. Christianity has been holding the primacy of the Bible since day one of the Renaissance.

Quote
I also learned that what atheists regard as the evidence against a Creator, was actually evidence they plagiarized from the Bible in the first instance from Newton and Kant!
You still sticking with the story that Newton and Kant got their inspiration from the Bible? The great inspiration for Kant that he was oddly silent about? The great inspiration for Newton that he also doesn't credit? Sure, guy, pull the other one. We know that Newton and Kant were Christians, but that doesn't say anything about what they can contribute to science.

Like I said before, you can scrape any source at all for any inspiriation you want, so long as you test those ideas against reality. That's what makes something science and not mere assertion. If an idea doesn't hold up to experiment, it's wrong, even if it comes from the Bible.

Quote
It will for ever be a fallacy that atheists are somehow busy discovering the origins of the Universe.
they are attempting to find something that can disprove the need for a Creator of everything, but every time they do come up with some sop story, They create this hydra with 7 heads that can not be destroyed due to per pressure.
I like it how you think you have this great insight into the minds of atheists. No, Christian scientists are trying to find out the origin of the universe, too, even if they believe in God, because 'Goddidit' doesn't satisfy even the Christian scientist's thirst for knowledge.

This notion that you can only seek a godless origin of the universe if your a godless atheist needs to die, because it's simply not true. It's just a story you tell yourself to give you the excuse not to search further and learn deeper.

Quote
Anyhow, you can believe in Length contraction and Time dilation.
No problem.
Science will just overtake you and man will become more informed, and I know it will not be long before Einstein is proven incorrect, and SR abd GR gets posted to the past with Time dilation.
If there comes a day when Einstein is struck down, I will be happy because that means we can have Warp drives and all the coolness that comes with it. Further, if it happens, I will know long before you do when the day comes and what finally does Einstein in. This is because I understand the theory and know where to look for the cracks in its foundation. I'll be way ahead of you.

It is a statement of supreme arrogance to think that YOU know that your boys have the right mavrick idea, out of all the fringe thinkers that are in every scientific field. The only reason you would be right is if you are lucky, not because you "know" anything more than I do. You've never been able to articulate WHY classical mechanics has to be right other than assertion, assertion, assertion. An assertion is worth next to nothing. A specific experiment that cracks the foundation is needed, but you've never been able to give me a reference to any paper that does this. You don't have a Michelson-Morley experiment of your own. All you have is an alleged improvement in accuracy by some chump who thinks that NASA runs the GPS constellation.

So yeah, you may be right, and winged monkeys may come flying out my butt, but I'm not holding my breath.
Title: Re: Time dilation, length contraction, Relativity and the Bible!
Post by: Mousetrap on September 05, 2018, 02:42:29 AM
Quote from: HR
Quote from: MT
I also learned that what atheists regard as the evidence against a Creator, was actually evidence they plagiarized from the Bible in the first instance from Newton and Kant!
You still sticking with the story that Newton and Kant got their inspiration from the Bible? The great inspiration for Kant that he was oddly silent about? The great inspiration for Newton that he also doesn't credit? Sure, guy, pull the other one. We know that Newton and Kant were Christians, but that doesn't say anything about what they can contribute to science.

Like I said before, you can scrape any source at all for any inspiriation you want, so long as you test those ideas against reality. That's what makes something science and not mere assertion. If an idea doesn't hold up to experiment, it's wrong, even if it comes from the Bible.

Was Kant silent about his Nebular theory?
Well, I tried to count the times he referred to the Biblical God, the Biblical narratives, the Flood, etc.
If I were to remove these references from his Natural History of the Universe based on Newtonian principles, It will be unintelligible.

On Newton, a man who claims that he could not find any errors in the Bible in his lifetime, even when compared with theoretical and experimental science, says something about you and Newton and Kant.

It says, Mouse trap, who should you believe?

Newton and Kant...or this scientists that  claims the Bible is a mere fairy tale?

Now, my advice to you is this.
Go back and question your foolish scientists that propose the Bible is wrong, AND FIND OUT FOR YOURSELF.
GET OUT OF YOUR COMFORT ZONE AND PROVE NEWTON WRONG BEFORE YOU CRITICIZE THE BIBLE.

And if you claim Newton was wrong on the authentication he underwrote to the Bible, you can just as well delete all his scientific work also.

Oh, I forgot, you and Einstein did just that.
Title: Re: Time dilation, length contraction, Relativity and the Bible!
Post by: Mousetrap on September 05, 2018, 02:50:52 AM
Quote from: trdsf on September 04, 2018, 06:25:21 AM
Reality is one of the Seven Dirty Words (https://en.wikipedia.org/wiki/Seven_dirty_words) in Mouseworld.  Reality, Logic, Reason, Sense, Repeatability, Mathematics and Tits... er, Data.
You obviously got it all wrong sir.
Reality is something you call Relativity.
Logic is something you call creation ex nihil.
Reason is something you call, Communism.
Sense is something you call, evolutionary Instinct.
Repeat-ability is something you call Atheist Propaganda.
Mathematics is something you call 0=+1-1 (Matter and anti matter)
Tits is something you call whore, forgetting you have them too.
Title: Re: Time dilation, length contraction, Relativity and the Bible!
Post by: Shiranu on September 05, 2018, 03:33:52 AM
God, you are dull.
Title: Re: Time dilation, length contraction, Relativity and the Bible!
Post by: Baruch on September 05, 2018, 04:25:43 AM
Quote from: Mousetrap on September 05, 2018, 02:50:52 AM
You obviously got it all wrong sir.
Reality is something you call Relativity.
Logic is something you call creation ex nihil.
Reason is something you call, Communism.
Sense is something you call, evolutionary Instinct.
Repeat-ability is something you call Atheist Propaganda.
Mathematics is something you call 0=+1-1 (Matter and anti matter)
Tits is something you call whore, forgetting you have them too.

See, mixing politics with the search for objective knowledge.  Isn't that Lysenkoism?  Isn't that Reich culture?  Mixing emotion with reason is ... crankiness.
Title: Re: Time dilation, length contraction, Relativity and the Bible!
Post by: Hakurei Reimu on September 05, 2018, 11:36:16 AM
Quote from: Mousetrap on September 05, 2018, 02:42:29 AM
Was Kant silent about his Nebular theory?
Kent was pretty silent about his Nebular theory coming from the bible. I read his work on the topic. He never credits the bible for his Nebular hypothesis. (Because a hypothesis is just a hypothesis until you support it with evidence.) Instead, he credits Thomas White.

Quote
Well, I tried to count the times he referred to the Biblical God, the Biblical narratives, the Flood, etc.
If I were to remove these references from his Natural History of the Universe based on Newtonian principles, It will be unintelligible.
Like I said before, I think of the two of us, I'm the only one who actually read Kant's paper. I even noted that he makes lots of references to the Creator. I also never read that he makes any assertion one way or another about the Flood. The Flood, after all, occurred long after the Earth was formed. So, yeah, I think you can drop all the references to the Creator, the biblical narratives (which are hardly mentioned at all, so no prob), the Flood and all that and it will still be quite intelligible.

Quote
On Newton, a man who claims that he could not find any errors in the Bible in his lifetime, even when compared with theoretical and experimental science, says something about you and Newton and Kant.
Newton was not a perfect man and neither was Kant. They both built upon the work of others, and others built upon their work. Some of their work is going to be proved wrong, and anyone who is uncomfortable with that fact shouldn't be in science.

Newton was an asshole. When his peers argued against him, instead of taking it with calm and poise, he threw a hissy-fit and started keeping his research to himself, including his work on alchemy and biblical chronology. (Yeah, he believed in alchemy, too. Both of those particular works are largely forgotten.) He eventually got a job working as Warden and then Master for the Royal Mint, which he considered his most important work.

Quote
It says, Mouse trap, who should you believe?

Newton and Kant...or this scientists that  claims the Bible is a mere fairy tale?
Even Isaac Newton, asshole he was, and Immanuel Kant acknowledged that they built upon the work of others, and that others will build upon them. I think they always had the sense that their work may be superceeded. (Though, again, Newton was an asshole.)

Quote
Now, my advice to you is this.
Go back and question your foolish scientists that propose the Bible is wrong, AND FIND OUT FOR YOURSELF.
GET OUT OF YOUR COMFORT ZONE AND PROVE NEWTON WRONG BEFORE YOU CRITICIZE THE BIBLE.
I've read enough of Ronald Hatch's stuff to figure out that all he has done is built an elaborate mathematical reformulation of GR and SR and insisting that they're not GR and SR and that everything observed is merely "apparent." He does not contest that the effects of SR and GR are observed, he merely insists that they are only a seeming. But his physics is no more Newtonian than Einstein's relativity.

Furthermore, such mathematical wizardry not interesting from a scientific perspective, which explains why his citations on such articles are very low, in the single or double digits for each article. After all, if you're attacking one of the main pillars of physics with any kind of veracity, you're going to be talked about, even if you are only talked about to be rubbished. Also, his own works account for a sizable chunk of those citations.

I think physicists can see through the little mathematical games he's playing and are thinking, "How cute!"

I've already addressed your "He told NASA!" canard: it's a fabrication, on either your part or his. Because otherwise, your canard would be "He told the Pentagon!"

Now, this doesn't mean that Hatch can't produce good science despite his crankery. His being able to plow a row with 15 mm accuracy over 20 km is impressive. However, it is over a range where both SR and GR effects would vanish into the noise â€" a field doesn't ascend the gravity well very far and farm equipment makes snails look speedy. It's also not that good compared to what could have been done a few decades ago.

As to proving Newton wrong, here is a list of things that are impossible in Newton's mechanics:


So, yeah, Newton was wrong, and if you want to tie Newton's mechanics to the Bible so badly, I guess that makes the Bible wrong, too. That's what you get for idolotry.

About that Bible, though. I have one more thing to say here. The Bible makes the observation that (in a world where the fastest things were horses, light not yet being identified as something with speed) the world does seem to not care what man does to change times and lengths. A particular road to Cairo from Jerusalem would measure the same regardless who is traveling it (discounting measurement inaccuracies), and time would march regardless of what man did. Thus, even if Newton were strictly true at all speeds, it would be an obvious observation that makes it uninteresting. Thus, even if Einstein was struck down, the Bible would be true, but not interesting, and keeping with my mantra. As it happens, the current state of science has the Bible as neither true nor interesting here.

Quote
And if you claim Newton was wrong on the authentication he underwrote to the Bible, you can just as well delete all his scientific work also.

Oh, I forgot, you and Einstein did just that.
Einstein, I, and the entire scientific community did not claim that the Newton was wrong because he was underwriting the bible. We claim that Newton was wrong because he didn't have the full picture. It wasn't even until 1862 when the speed of light was definitively measured to be definitely somewhat less than 300 thousand kilometers per second. Newton had no access to speeds that would make the effects of relativity measurable to his instruments, so that he didn't foresee the effects of relativity is understandable.

It's also no detriment on Newton that he was ultimately proved wrong. (His personal behavior, however, is another story.) He did the best job he could and came up with a paradigm that lasted three hundred years, and only fell when we had access to speeds and phenomena that were unavailable to Newton.

Even today, Newton's physics is still a good approximation of physics for ordinary speeds (even up to orbital speeds), which is why you're taught Newton's mechanics in high school physics. Newton worked in a world where absolute time and space was obvious, and his work reflected that. Trouble is, just because it's obvious doesn't mean its true.

You would do well to keep this in mind.
Title: Re: Time dilation, length contraction, Relativity and the Bible!
Post by: Hydra009 on September 05, 2018, 11:48:57 AM
Quote from: Mousetrap on September 05, 2018, 02:42:29 AM
Go back and question your foolish scientists that propose the Bible is wrong
This statement right here says a lot about how you think (and not think) and none of it is good.

"Your scientists."

As if they are owned by a certain ideological group.  It also hasn't escaped my attention how you talk about "Christian scientists" as if those terms have much significance to each other.  It's all reminiscent of Nazi-era grouping of science into "Jewish science" and "Aryan science", with Aryan scientists celebrated while Jewish scientists are vilified.

And lastly, your conclusion is that these scientists - the ones who belong to us heathens exclusively - are foolish.  Why?  Because they propose that the Bible is wrong (heaven forbid).  Here we see an ideology where everything is built around the assumption that Bible is inerrant and everything else judged accordingly.  Naturally, atheists are the great villains of this ideology.
Title: Re: Time dilation, length contraction, Relativity and the Bible!
Post by: Baruch on September 05, 2018, 01:05:52 PM
If one sees everything as politics, then even American football events are just conspiracies against the R party.  Not everything is politics, thought it seems so to one kind of psychopath.
Title: Re: Time dilation, length contraction, Relativity and the Bible!
Post by: trdsf on September 05, 2018, 02:18:41 PM
Newton wasn't even wrong, really -- not within the limits of 17th century measurement.  All Newton was, was incomplete, and to this day it's sufficiently accurate to get astronauts, cosmonauts and taikonauts to orbit and back, as nothing meaningfully relativistic is going on there.

The difference with GPS is that time has to be measured phenomenally accurately, accurately enough that the tiny differences caused by relativity does matter.  You can't do GPS with Newtonian physics... but there was a certain scarcity of satellites in the 1600s and 1700s, so that's hardly his fault.
Title: Re: Time dilation, length contraction, Relativity and the Bible!
Post by: Unbeliever on September 05, 2018, 03:04:17 PM
Mumpsimus: a stubborn person who insists on making an error in spite of being shown that it is wrong.
Title: Re: Time dilation, length contraction, Relativity and the Bible!
Post by: Hakurei Reimu on September 06, 2018, 12:00:41 AM
Quote from: Hakurei Reimu on September 05, 2018, 11:36:16 AM
I've read enough of Ronald Hatch's stuff to figure out that all he has done is built an elaborate mathematical reformulation of GR and SR and insisting that they're not GR and SR and that everything observed is merely "apparent." He does not contest that the effects of SR and GR are observed, he merely insists that they are only a seeming. But his physics is no more Newtonian than Einstein's relativity.

...

I think physicists can see through the little mathematical games he's playing and are thinking, "How cute!"
The above judgement was based on snippets gleaned from abstracts behind a paywall (I refuse to shell out money for an internet battle). Since then, I've managed to track down some equations that our friend Hatch is using from a slideshow presentation from 2000.

The first thing to remark is that Hatch does not understand GR on its own terms. This is clear from his own attempts to characterize the "gravitational scale factor" of GR. In GR, the metric plays a central role in the physics of spacetime. Instead, what he expresses as the scale factor is just the gravitational redshift for a Newtonian potential, z = exp(-GM/rc²)-1, plus 1. But we don't use this expression or any similar one for deriving the kinematics of free-falling particles; we derive it explicitly from the metric.

Speaking of the weak field, Hatch's scale factor for his "revision" looked awfully familiar to me, so I checked in my copy of A First Course on General Relativity by Bernard Schultz, and lo and behold, I found it (or rather, its square, but it's in a d_² term) in the metric expression for the weak field approximation. Specifically, in the time component, which is the dominant component of the weak field approximation for a Newtonian potential, Ï• = -GM/r. From this, you can recover Newton's gravitational force law, dp⃗/dÏ,, = F = -m∇ϕ = -GMm/r², for a particle free-falling in this metric.

No, Dr. Hatch, GR does NOT derive F = -GMm(z+1)/r² as you claim!

Thing is about the weak field approximation, it's been kicking around since Einstein, and is derivable from GR with the assumptions that the gravitational field is weak (with escape velocities well below light) and for small velocities (again, well below light). Under the assumptions of the weak field/small velocities, the metric can be treated as straight Minkowskian perturbed by a very small matrix, h, and under the Lorentz transformation, the metric transforms in such a way that our h transforms as if it were a tensor itself, and leads to what Schultz calls a "convenient fiction" that h is a background field on flat spacetime. Hatch claims his is an "ether theory," and this is part of the reason why it works after a fashion. The other thing is that we can derive the weak field approximation for the Newtonian potential completely in GR. I don't know how Hatch derived his expression for his gravitational scale factor, but if his calculations are in any way sound, I wouldn't be surprised that he somehow recapitulated the result of GR's weak field approximation.

As such, I'm able to put a finer point on that little thing that physicists say when they see Hatch's stuff: "How cute! He re-derived Einstein's gravitational weak field approximation!" So it's no wonder physicists aren't taking him seriously. He's literally coming up with stuff that has been known for almost a century and treating it as if he's come up with a brand new interpretation. It'll even sorta work for weak fields.

Addendum: I have just now learned that Hatch's scale factor, s, comes exactly from the weak field approximation. Just straight rips it off. It's called an "approximation" for a reason. So, yeah, it really is just a recapitulation of an approximation that physicists have known about for almost a century. â'¨

Oh, and he's derived the full spherically symmetric external field in GR wrong. We know what it is. It's the Schwartzschild metric, which does NOT have exponentials in its expression. â'¨ /Addendum

Now, his alternate Lorentz transformation? That's more difficult to fathom, because I have yet as this writing to track down an explicit expression. He does make reference to an absolute frame, though how one's supposed to find the absolute frame (and thus get the "really correct" value of c) is beyond me. However, Hatch does reference the apparent speed of light in tranverse and parallel (along-side) directions as ct = c/γ; cp = c/γ², and a length contraction along the parallel direction is lp = l/γ, while transverse length is unaffected. (As always, γ = sqrt(1-v²/c²).)

Let's work out a Michelson-Morley type experiment here. Figuring the lengths of the two arms in Earth's frame, E, and translating to the "true frame" will allow us to avoid parallel and transverse speed of light shenannigans. The parallel moving tube is measured to be L = l'p in E, so it's "true length" is lp = γL, so it takes light tp = 2γL/cγ² = 2L/cγ to travel this distance. For the transverse moving tube, measured at L = l't in E, its "true length" is lt = L, so it takes light tt = 2L/cγ*. The difference is ∆t = tt - tp = 2L/cγ - 2L/cγ = 0, so it appears that it replicates the Michelson-Morley experiment and no fringing is detected.

But * indicates an incorrect derivation: the light does NOT travel lt = L along the transverse path, because the ends of that tube are MOVING forward at v â€" the light that gets reflected from the far mirror has to be traveling along a diagonal outward, and takes a similar diagonal back. In fact, the actual path traveled by the light can be shown to be γL each leg, not L. This gives us tt = 2γL/cγ = 2L/c, which gives us a ∆t = 2L/c - 2L/cγ = 2L/c (1-1/γ) ≠ 0. Ergo, Hatch's derivation of transverse and parallel length contraction does NOT produce an equal time along both tubes.

Taking it from E, this becomes t'p = 2L/cp = 2Lγ²/c; t't = 2L/ct = 2Lγ/c, so ∆t' = γ∆t = 2Lγ/c - 2Lγ²/c = 2Lγ/c (1-1/γ), which gives us ∆t = 2L/c (1-1/γ) the same magnitude of fringe shifting.

But of course, Michelson and Morley and other experiments of its type show no such fringing. Hatch's equations make the wrong predictions. Too bad, so sad.

And the nail in this coffin comes in the form of his conclusion, which predict that "LIGO experiments should indicate failure of GRT model within 10 years." Except that LIGO detected their first gravitational wave on 14 September 2015. While five years late, it does mean that Hatch's prediction is wrong, and six have been detected since. Womp, womp.
Title: Re: Time dilation, length contraction, Relativity and the Bible!
Post by: Mousetrap on September 06, 2018, 03:17:23 AM
The claims you make about you reading Kant's work, and telling me I am a liar is really a frustration to any intellect.
All you are doing is taking everything I say, and write pages of opposing claims.
I do not have a problem with any of that, for you have a right to speak your mind.
But, I thought I really need to show you that you are not truthful in your so called facts you present here.
You say: I also never read that he (Kant) makes any assertion one way or another about the Flood.
This is again a derailment strategy telling me my claims are lies, sending me out to get the sources, which you said you read!
Why do you act this way?
Well, I decided to not ignore you, but to prove to you what I claimed was true.

All you did is to demonstrate that You did not read Kant's essay!
And we already established you also never read the Bible.

Here is Kant on ONE of many REFERENCES OF THE FLOOD!

Quote from: Hakurei Reimu
Quote from: Mousetrap
Well, I tried to count the times he referred to the Biblical God, the Biblical narratives, the Flood, etc.
If I were to remove these references from his Natural History of the Universe based on Newtonian principles, It will be unintelligible.
Like I said before, I think of the two of us, I'm the only one who actually read Kant's paper. I even noted that he makes lots of references to the Creator. I also never read that he makes any assertion one way or another about the Flood. The Flood, after all, occurred long after the Earth was formed. So, yeah, I think you can drop all the references to the Creator, the biblical narratives (which are hardly mentioned at all, so no prob), the Flood and all that and it will still be quite intelligible.
And here is one page on Kant and the flood.
Now who is lying?

Universal natural history and theory of the heavens or essay on the constitution and the mechanical origin of the whole universe according to Newtonian principles
Could we not imagine that the Earth once had a ring like Saturn? It could have risen from its surface just as Saturn’s did and have remained for a long time while the Earth was slowed down by who knows what cause from a much faster rotation to its present rate, or that we can consider that universal basic material falling sideways was capable of having formed it in accordance with the rules explained above, which we do not have to take completely seriously if we want to indulge our penchant for oddities. But what a stock of lovely explanations and consequences such an idea presents us with! A ring around the Earth! What a beautiful sight for those created to inhabit the Earth as a paradise; what comfort for those on which nature smiles from all sides! But this is nothing compared with the confirmation such a hypothesis can borrow from the chronicle of the story of creation and which is no small recommendation for applause for those who believe they are not desecrating but rather confirming the honour of revealed religion when they make use of it to give the excesses of their wits some prestige. The water of the firmament mentioned in Moses’ description has already caused the interpreters some effort. Could one not use this ring to help to get oneself out of this difficulty? Without a doubt this ring consisted of watery vapours, and in addition to the advantage it was able to provide the first inhabitants of the Earth, there is the additional one of having it break when required so that floods could punish the world which had made itself unworthy of such beauty. Either a comet, whose attraction brought confusion into the regular motions of its parts, or the cooling of the area of its location unified its dispersed vaporous particles and hurled it down onto the earth in one of the most gruesome cloudbursts. It is easy to know what the consequences of this were. The whole world disappeared under the water and in the strange and volatile vapours of this unnatural 1:304 rain also absorbed that slow poison which brought all creatures closer to death and destruction. Now the figure of a pale and light arc had disappeared from the horizon and the new world, which could never remember this sight without feeling terror in the face of this terrible tool of divine revenge, perhaps saw, with not a little consternation, in the first rain that coloured arc that appeared to copy the first in shape but, through the assurance of the reconciled heavens, was to be a sign of grace and a memorial of a continuing preservation of the Earth, changed as it now was. The similarity of the shape of this memorial sign with the event it signified could commend such a hypothesis to those who are devoted to the dominant tendency of bringing the miracles of revelation into the same system as the ordinary laws of nature. I consider it more advisable completely to forgo the fleeting applause such correspondences might arouse for the true pleasure that arises from the perception of regular
Title: Re: Time dilation, length contraction, Relativity and the Bible!
Post by: Hakurei Reimu on September 06, 2018, 09:55:05 AM
Quote from: Mousetrap on September 06, 2018, 03:17:23 AM
The claims you make about you reading Kant's work, and telling me I am a liar is really a frustration to any intellect.
All you are doing is taking everything I say, and write pages of opposing claims.
Well, isn't that was a debate is about?

Quote
This is again a derailment strategy telling me my claims are lies, sending me out to get the sources, which you said you read!
Why do you act this way?
Because you're an asshole. You come here and ruin our wa with these pointless threads about the Bible forsaw science and blah blah blah. You pretend that this stuff is scientifically justified, but when I show it isn't, you instead get your panties in a twist and devolve into the typical Christian spew-fest.

Quote
Well, I decided to not ignore you, but to prove to you what I claimed was true.

All you did is to demonstrate that You did not read Kant's essay!
And we already established you also never read the Bible.
I've never read it cover to cover, yes. I don't pretend otherwise. But in that respect, I'm not much different from most Christians. Even if I had, you'll find something else to complain about. It's simply a waste of time to read the whole Bible on your behalf.

Quote
Here is Kant on ONE of many REFERENCES OF THE FLOOD!
----
And here is one page on Kant and the flood.
Now who is lying?

Universal natural history and theory of the heavens or essay on the constitution and the mechanical origin of the whole universe according to Newtonian principles (http://users.clas.ufl.edu/burt/spaceshotsairheads/Kantuniversalnaturalhistory.pdf)
Could we not imagine that the Earth once had a ring like Saturn? It could have risen from its surface just as Saturn’s did and have remained for a long time while the Earth was slowed down by who knows what cause from a much faster

<snip>

dominant tendency of bringing the miracles of revelation into the same system as the ordinary laws of nature. I consider it more advisable completely to forgo the fleeting applause such correspondences might arouse for the true pleasure that arises from the perception of regular
It has been over a month since I read this, and now that I see it again, I do remember reading this section. In fact, here was my comment to you about this very passage:
Quote from: Hakurei Reimu on August 02, 2018, 05:24:26 PM
Uh, no you haven't. Even Kant himself doesn't say that scripture is where his description of his nebular theory came from. He, in fact, references Thomas Wright. And lots of other people, but only one brief mention of the actual bible and that's in reference to the waters above the firmament, and how a ring around Earth could resolve that problem. He, however, arrived at his nebular theory by his own work and building on the work of others.
So, did I forget? Yes, I'll cop to that. Did I lie? No. Did I read Kant's essay? Certainly, because the only reason I knew about the ring similar to Saturn in the first place to comment about it a month ago is because I read it in Kant. Who's lying? YOU ARE. Why is it that, in all of the "many" references to the flood in Kant you boast about, why did you pick out the VERY SAME reference that I picked up on a month ago? The reason why is that this is the only reference that you could find to the Flood. There are not "many;" there is one. Singular.

Furthermore, Kant's passage on the Flood is all fluff and no crunch. Where is the justification for anything Kant says here? Without that justification, it's just an assertion and can be dismissed as such. No wonder I forgot about it.

I accept your correction. Now, what about the rest of it?

What about mass-energy equivalence?

What about the Michelson-Morley experiment, which according to your own boy Hatch's "theory," would should have detected something but we know produces a null result?

What about the relativistic Doppler effect?

What about the lengthening of the half-life of relativistic unstable particles?

What about the sheer number of theories that use SR as a foundation that successfully predict results?

What about the fact that we cannot accelerate particles past the speed of light even though Newton's mechanics say they should be going many times the speed of light?

What about the fact that it doesn't matter what frame you work in, their results apply across frames via the Lorentz transformation?

What about the fact that GR, another successful theory by Einstein, and SR can be formulated in a coordinate-free way?

What about the fact that, at best, Hatch has recapitulated the weak field approximation of GR and toting it as an alternative?

What about the fact that Hatch predicted no gravity waves, yet LIGO and other detectors have now detected them?

Don't try to use my faulty memory on Kant (which is not the subject of this thread) to derail us from the echoing absence of response to any of the above. You attribute to malice on my part what is reasonably explainable by a forgetful brain. You are a very disingenuous person, to turn an honest mistake into a reason to claim some sort of malice on my part.
Title: Re: Time dilation, length contraction, Relativity and the Bible!
Post by: Mousetrap on September 07, 2018, 02:25:00 AM
Quote from: Hakurei Reimu on September 06, 2018, 09:55:05 AM
Quote from: Mouse Trap
Well, I decided to not ignore you, but to prove to you what I claimed was true.

All you did is to demonstrate that You did not read Kant's essay!
And we already established you also never read the Bible.
I've never read it cover to cover, yes. I don't pretend otherwise. But in that respect, I'm not much different from most Christians. Even if I had, you'll find something else to complain about. It's simply a waste of time to read the whole Bible on your behalf.
Oh, but you did say that between you and I you are the only one who read Kant's Natural history.
Did you forget?
Let me show you your exact words!

Quote from: HRLike I said before, I think of the two of us, I'm the only one who actually read Kant's paper. I even noted that he makes lots of references to the Creator. I also never read that he makes any assertion one way or another about the Flood. The Flood, after all, occurred long after the Earth was formed. So, yeah, I think you can drop all the references to the Creator, the biblical narratives (which are hardly mentioned at all, so no prob), the Flood and all that and it will still be quite intelligible.

And you have the audacity to tell me I am a liar!

I am leaving for Ghana tonight for 3 weeks.
When I return, I am going to take you up on the Mickelson Morley experiment, and Lorenz transformation.
But this time I hope you will be more liberal in your approach on what I will show you, and will not just post pages of arguments.
If we can dissect with minimalism, every fact on what I will work on, It will be possible that either I will show you what I found and understand what SR and GR, and you can perhaps show me exactly where I went wrong in my understanding.
Or, I will be able to show you something you never thought about.
I think such a proposition is reasonable, and I hope you will agree to such a brain linking.
Title: Re: Time dilation, length contraction, Relativity and the Bible!
Post by: Baruch on September 07, 2018, 06:06:34 AM
Have a good business trip ;-)
Title: Re: Time dilation, length contraction, Relativity and the Bible!
Post by: Mike Cl on September 07, 2018, 08:50:20 AM
Quote from: Mousetrap on September 07, 2018, 02:25:00 AM

And you have the audacity to tell me I am a liar!


You are a liar proven by your words.  It does not take audacity to say so, only honesty--which you lack.  You are all stripes of a liar from simply being disingenuous to bald-faced lying' not an honest bone in your body.  And you lie to everybody, but mostly to yourself.  And seemingly, you are too stupid to notice or care.  Audacity?  No, just saying it like it is. 
Title: Re: Time dilation, length contraction, Relativity and the Bible!
Post by: Hakurei Reimu on September 07, 2018, 02:26:47 PM
Quote from: Mousetrap on September 07, 2018, 02:25:00 AM
Oh, but you did say that between you and I you are the only one who read Kant's Natural history.
Pull your head out of your ass. I was clearly talking about the Bible, not Kant, when I said I never read "it" cover to cover, which should have been clear from "in that respect, I'm not much different from most Christians." Last time I looked, it's the BIBLE that Christians claim to follow, not the works of Immanuel Kant. I read all of the Kant's Allgemeine.

Quote
Did you forget?
Let me show you your exact words!

And you have the audacity to tell me I am a liar!
Yes, I am calling you a liar, and it's only audacity if it's not true. I already copped to forgetting that Kant mentioned the Flood. Haven't you ever read something and then forgotten it three weeks later? Of course not. Moustrap is the perfect superman, incapable of forgetting. Nuts to that! I'm not accepting being called a liar because some internet goober thinks that reading Kant causes spontaneous photographic memory retention.

However, I call you a liar because â€"among many other things you have lied aboutâ€" you are saying that there are "many" references to the Bible in Kant's Allgemeine, but at the time of my reading I had found only one mention of the Bible or the Flood â€"yes, that entire passage only counts as one reference, because Kant's talking about one thingâ€" and interestingly, it's the exact same reference you cite. If there were "many" references as you claim, you would have likely picked one of the others. But you picked the only one that I have even a vague memory reading. What a coincidence. No, it's not a coincidence, you are lying by way of gross exaggeration. There are not "many" references to the bible; there's only one, rather easily-forgetable mention.

But even your cited passage does not demonstrate that Kant is drawing inspiration from the Bible. All I see is Kant doing exactly what you're doing: trying to make the science fit the Biblical account. He was trying to explain the Flood by way of inspiration from seeing or reading about the rings of Saturn. Saturn was his actual inspiration here.

As stated before, science need not explain what is not observed. There was no Flood that drowned the whole of the Earth, at any point, let alone a geologically recent one. No observation of geology supports one. So, this is just another point on which Kant is wrong. But being wrong doesn't make you a bad person. You're just wrong. When one gets better information, one tends to throw out old ideas, and indeed, one should reexamine their old ideas and throw them out if they prove inadequate in the face of better information. Kant isn't a bad person or even a bad Christian because he happened to be wrong in saying that Earth had a Saturn-like ring that caused the Flood; he's just not correct.

Anyway, we're done with Kant. We've been done with Kant since you abandoned your Cosmology thread. Kant has nothing to do with time dilation, length contraction, or anything this thread's about.

Quote
I am leaving for Ghana tonight for 3 weeks.
When I return, I am going to take you up on the Mickelson Morley experiment, and Lorenz transformation.
But this time I hope you will be more liberal in your approach on what I will show you, and will not just post pages of arguments.
If you want to demolish one of the foundational pillars of modern physics, then you have a big task ahead of you. It essentially means that you have to explain what is observed in our experiments better than when using SR, and there is a lot to explain.

Quote
If we can dissect with minimalism, every fact on what I will work on, It will be possible that either I will show you what I found and understand what SR and GR, and you can perhaps show me exactly where I went wrong in my understanding.
Or, I will be able to show you something you never thought about.
I think such a proposition is reasonable, and I hope you will agree to such a brain linking.
Take my advice: Stick with SR. GR is even more mind-warping than SR because SR can be conducted in flat geometries where the coordinate systems are nice and calm, while GR involves space itself warping by the presence and motion of mass-energy. GR is highly nonlinear, which is basically mathematics-talk for "get a computer to do that bullshit, because that's really nasty-ass brain-fuckage."

Given that I've never seen you use the Lorentz transformation even once, much less successfully, I'm going to have to demand you pass a test of skill. If you don't even grasp that basic concept, then you're not in a position to argue about SR. The following exercise should be easy if you understand the Lorentz transformation:

Show that, for any two points P and Q with arbitrary coordinates in S and for any arbitrary boost v < c, that the Lorentz transformation Λ(v) from S into S' leaves the quantity ∆s² = ∆t²c² - (∆x² + ∆y² + ∆z²) unchanged.
Title: Re: Time dilation, length contraction, Relativity and the Bible!
Post by: Cavebear on September 08, 2018, 06:36:38 AM
Quote from: Mousetrap on September 02, 2018, 05:00:34 AM
Ron Hatch proves GPS calculations will be 100% correct, only if the relativity calculations is removed! (https://www.gps.gov/governance/advisory/members/hatch/)

Oh, Hahurei Reimu.
It seems as if the 2 evidences for SR and GR, actually proves SR and GR is one big error!

Planes flying around the world with Cesium clocks, as well as Global positioning systems does not prove Relativity at all.
On the contrary, when there are scientists who would like to get the calculations correct, they realized....
Einstein was wrong, and he fed us BS!!!

Thanks for helping me make up my mind about your long arguments.  When you denied the cesium clocks as proving relativity, all your arguments went out the window.

:)
Title: Re: Time dilation, length contraction, Relativity and the Bible!
Post by: Baruch on September 08, 2018, 08:56:11 AM
Quote from: Cavebear on September 08, 2018, 06:36:38 AM
Thanks for helping me make up my mind about your long arguments.  When you denied the cesium clocks as proving relativity, all your arguments went out the window.

:)

Unfortunately, science deniers, find they have more and more to deny, once they go down that road.  If they would stick to what is currently speculative (superstrings) then they wouldn't drive into a ditch.
Title: Re: Time dilation, length contraction, Relativity and the Bible!
Post by: Hakurei Reimu on September 08, 2018, 06:32:11 PM
Quote from: Cavebear on September 08, 2018, 06:36:38 AM
Thanks for helping me make up my mind about your long arguments.  When you denied the cesium clocks as proving relativity, all your arguments went out the window.

:)

All Mousie proves in showcasing Hatch is that a crank is a crank, even if he works for the government. It's still argument from irrelevant authority, not argument from his own understanding. That, right there, is a severe problem. You can find anyone with a degree and credentials who disagrees with any subject matter under the sun, but if there were really a problem to turn up with SR and GR, it's not going to be found by Ronald Hatch, whose own papers show a criminal misunderstanding of both. It's going to come from people like Steve Weinberg, or Kip Thorne, or any one of a thousand experts on GR and/or SR, as they know the subject better than anyone. They know where to look for the cracks in the foundation. So far, none has turned up.
Title: Re: Time dilation, length contraction, Relativity and the Bible!
Post by: trdsf on September 09, 2018, 02:01:31 PM
Quote from: Hakurei Reimu on September 08, 2018, 06:32:11 PM
All Mousie proves in showcasing Hatch is that a crank is a crank, even if he works for the government.
And Hatch is far from the first, even at NASA.  If you want some real crankery, look up Jack Parsons: founder of JPL, responsible for many aspects of both liquid- and solid-fueled rocketry including the principles behind the Shuttle's SRBs which are based fundamentally on Parson's work... and also enthusiastic occultist, Thelemite, and follower of Alaister Crowley.

We'll all have a nice cup of coffee waiting for our little church mouse to extol the virtues of the Ordo Templi Orientis because one scientist believed in it, since he's set that as his standard for demonstrating something.

(crickets chirp)

(crickets continue chirping)

(crickets' legs fall off from all the incessant chirping)
Title: Re: Time dilation, length contraction, Relativity and the Bible!
Post by: Baruch on September 09, 2018, 02:23:55 PM
Face on Mars fooled quite a few in the 1980s.  I knew engineers quite enthused about it, because scifi has to be true, right?
Title: Re: Time dilation, length contraction, Relativity and the Bible!
Post by: Mike Cl on September 09, 2018, 02:39:45 PM
Quote from: Baruch on September 09, 2018, 02:23:55 PM
Face on Mars fooled quite a few in the 1980s.  I knew engineers quite enthused about it, because scifi has to be true, right?
And what of the canals?  They fooled many for many years.
Title: Re: Time dilation, length contraction, Relativity and the Bible!
Post by: Baruch on September 09, 2018, 10:36:11 PM
Quote from: Mike Cl on September 09, 2018, 02:39:45 PM
And what of the canals?  They fooled many for many years.

Early on, scifi ran away with the imagination.  Cyrano De Bergerac ... did the first scifi story about the Moon.  One hundred years ago, a shield against gravity was imagined, allowing a much less impactful trip to the Moon than a giant cannon in Florida.  I have visited the observatory in Flagstaff, and honored Percival Lowell's grave.  But it was his student Clyde Tombaugh, who made the real discover, Pluto.
Title: Re: Time dilation, length contraction, Relativity and the Bible!
Post by: trdsf on September 10, 2018, 09:27:22 PM
Quote from: Mike Cl on September 09, 2018, 02:39:45 PM
And what of the canals?  They fooled many for many years.
And the sad thing about that, is that the 'canals of Mars' thing was entirely due to mistranslation.  The astronomer Giovanni Schiaparelli described what he saw as canali, which isn't the Italian for 'canals'.  It means 'channels'.  A channel is a natural formation; a canal is not.
Title: Re: Time dilation, length contraction, Relativity and the Bible!
Post by: Cavebear on September 12, 2018, 05:35:48 AM
Quote from: trdsf on September 10, 2018, 09:27:22 PM
And the sad thing about that, is that the 'canals of Mars' thing was entirely due to mistranslation.  The astronomer Giovanni Schiaparelli described what he saw as canali, which isn't the Italian for 'canals'.  It means 'channels'.  A channel is a natural formation; a canal is not.

And even the channels weren't really there.  Those telescopes at ground level with ripples in the atmosphere were REALLY bad.  You could see anything you wanted to see.
Title: Re: Time dilation, length contraction, Relativity and the Bible!
Post by: Mike Cl on September 12, 2018, 09:02:29 AM
Quote from: Cavebear on September 12, 2018, 05:35:48 AM
And even the channels weren't really there.  Those telescopes at ground level with ripples in the atmosphere were REALLY bad.  You could see anything you wanted to see.
Just like the bible.
Title: Re: Time dilation, length contraction, Relativity and the Bible!
Post by: Cavebear on September 12, 2018, 09:09:34 AM
Quote from: Mike Cl on September 12, 2018, 09:02:29 AM
Just like the bible.
Interesting.  Didn't Satan show Jesus the whole world from a mountaintop?  Obviously the world was flat back then.  Oh but wait, it isn't.  So the bible must have been corrected, right?
Title: Re: Time dilation, length contraction, Relativity and the Bible!
Post by: Mike Cl on September 12, 2018, 11:20:04 AM
Quote from: Cavebear on September 12, 2018, 09:09:34 AM
Interesting.  Didn't Satan show Jesus the whole world from a mountaintop?  Obviously the world was flat back then.  Oh but wait, it isn't.  So the bible must have been corrected, right?
How can you correct perfection???
Title: Re: Time dilation, length contraction, Relativity and the Bible!
Post by: Cavebear on September 12, 2018, 11:28:20 AM
Quote from: Mike Cl on September 12, 2018, 11:20:04 AM
How can you correct perfection???

Logic and thought and study.  And little things in the religious texts.  Like pi is not exactly 3.  The Earth is not flat.  There is no deity.  For theists, one small step at a time.  ;)
Title: Re: Time dilation, length contraction, Relativity and the Bible!
Post by: trdsf on September 12, 2018, 12:40:23 PM
Quote from: Cavebear on September 12, 2018, 05:35:48 AM
And even the channels weren't really there.  Those telescopes at ground level with ripples in the atmosphere were REALLY bad.  You could see anything you wanted to see.
Well, it's possible that at the very least, the spokes and lines he recorded on Venus were a real observation...  but that what Lowell was actually seeing was the shadow of his blood vessels on his own retina (https://www.skyandtelescope.com/astronomy-news/venus-spokes-an-explanation-at-last/?c=y), because he had his telescope stopped down so far.  It's possible â€" and I suspect probable â€" that the Martian lines have a similar source, especially coupled with the apparent changes in the face of Mars due to its own weather... and of course Lowell's own wishful thinking.
Title: Re: Time dilation, length contraction, Relativity and the Bible!
Post by: Baruch on September 12, 2018, 12:59:25 PM
Quote from: Cavebear on September 12, 2018, 11:28:20 AM
Logic and thought and study.  And little things in the religious texts.  Like pi is not exactly 3.  The Earth is not flat.  There is no deity.  For theists, one small step at a time.  ;)

And America, nor any other place, is the Promised Land, nor with there ever  be utopia, no matter how much scifi you read.
Title: Re: Time dilation, length contraction, Relativity and the Bible!
Post by: Mike Cl on September 12, 2018, 02:44:20 PM
Quote from: Cavebear on September 12, 2018, 11:28:20 AM
Logic and thought and study.  And little things in the religious texts.  Like pi is not exactly 3.  The Earth is not flat.  There is no deity.  For theists, one small step at a time.  ;)
Cavebear, I was being factious.
Title: Re: Time dilation, length contraction, Relativity and the Bible!
Post by: Cavebear on September 12, 2018, 02:59:12 PM
Quote from: trdsf on September 12, 2018, 12:40:23 PM
Well, it's possible that at the very least, the spokes and lines he recorded on Venus were a real observation... 

No way.  Venus was an opaque blank.  As Sagan said in Cosmos (here I go again with Sagan), earlier astronomers looked at Venus and just saw clouds.  Well, what are clouds?  Clouds are water.  So Venus must be all swamp.  What grows in swamps?  Ferns.  What eats ferns?  Dinosaurs.  Observation: you couldn't see anything on Venus.  Conclusion?  Dinosaurs!

LOL!